CMN 577 - Exam Review

¡Supera tus tareas y exámenes ahora con Quizwiz!

You note that your 11-year-old female patient is at Tanner Stage II. You would advise her mother that menarche will probably start in: A) 1 to 2 years B) 2.5 to 3 years C) 4 years D) It is dependent on the girl's genetic makeup

A) 1 to 2 years

Females with polycystic ovarian syndrome are at higher risk for: A) Heart disease and breast cancer B) Uterine fi broids and ovarian cancer C) Premature menopause D) PID (pelvic infl ammatory disease)

A) Heart disease and breast cancer

Which of the following is the best method for diagnosing candidiasis in the primary care setting? A) Wet smear B) Tzanck smear C) KOH (potassium hydroxide) smear D) Clinical fi ndings only

A) Wet smear

What does positive serum acid phosphatase suggest?

Associated with malignancy of the prostate gland with bone metastasis.

Which are the cardioselective beta blockers?

Atenolol. Esmolol. Metoprolol. Bisoprolol.

After being diagnosed with a minor respiratory illness, a nurse practitioner advises a parent about over the counter medication to treat the child's fever and discomfort. The child is 4 months old and has no pre-existing conditions. The child's parent should be advised to select: A. Aspirin B. Acetaminophen 15mg/kg every 4-6hours C. Children's Acetaminophen 15mg/kg every 4-6hours And/Or Children's Ibuprofen 10 mg/kg every 6-8 hours D. Children's Ibuprofen 10 mg/kg every 6-8 hours

B. Acetaminophen 15mg/kg every 4-6hours For a child over 2 months and under 6 months, the only appropriate over the counter medication for fever is Acetaminophen, given at a dosage of 15mg/kg every 4-6hours. *Ibuprofen is not indicated until the age of 6 months.* Parents should be reminded that dosage is weight based and their appropriate dose calculated for them. Parents should be advised against use of Aspirin due to the risk of Reye's Syndrome

Your 37 yo patient is 17 weeks pregnant with her second child. The patient voices concern related to her age with this pregnancy after refusing earlier screenings. Of the following diagnostic tests, which should be discussed to evaluate for abnormalities like Trisomy 21? When should this be performed? A. Anatomy Ultrasound; 19 weeks B. Amniocentesis; 19 weeks C. Nucal translucency; 19 weeks D. Chorionic villus sampling; 19 weeks

B. Amniocentesis; 19 weeks The patient has already surpassed the time for NT (1st trimester) and CVS (10-14 weeks); an anatomy ultrasound would not give this information

S4 is a sign of

LVH

Why should you avoid nitrofurantoin and sulfa drugs during the third trimester?

They can increase the risk of hyperbilirubinemia Leik

Treatment plan for allergic rhinitis

Treatment Plan ■ Nasal steroid sprays daily (i.e., fluticasone/Flonase) BID. ■ Decongestants (i.e., pseudoephedrine or Sudafed) PRN. Do not give to infants/young children. ■ Zyrtec 10 mg daily or PRN or combined antihistamine with decongestants. ■ Dust mite allergies: Avoid using ceiling fans, no stuffed animals or pets in bed, use a HEPA filter, and the like.

Which statin has highest incidence of rhabdo?

Zocor (simvastatin)

Expected ankle jerk (achilles) reflext when checking DTRs in a patient with severe sciatica or diabetic peripheral neuropathy?

may be absent (score of 0) or hypoactive (score of 1) Leik

Risk factors for asthma fatality

x of ED SABA use, nocturnal awakenings, increased dyspnea and wheezing, respiratory viral infections

Patient characteristics associated with chronic bronchitis?

"Blue bloaters" Overweight, cyanosis and normal or slightly increased respiartory rate

Presentation of rash related to cutaneous larva migrans?

"Creeping eruption" rashes are shaped like red raised wavy lines (serpinginous or snakelike) that are alone or few. They are red and very pruritic, and become excoriated from scratching (appears maculopapular). The areas of the body that are commonly exposed directly to contaminated soil and sand, such as the soles of the feet, extremities, or the buttocks, are the most common locations for larva migrans. Systemic treatment with either ivermectin once a day (for 1 to 2 days) or albendazole (for 3 days) is the preferred therapy for larva migrans. Leik

Patient characteristics associated with emphysema?

"Pink puffers" Underweight, pink skin, and increased respiratory rate.

S3 heart sound is...

"normal" in later pregnancy, children, and some young adults < age 35 to 40 years. S3 can also be a sign of heart failure​ (dyspnea, cough, wheeze, rapid weight gain, edema, fatigue, exertional dyspnea).

CHADS2VASc scoring

*C*HF *H*TN *A*ge >75 years *D*iabetes *S*troke/TIA/Systemic embolism *V*ascular disease *A*ge 65-74 years *S*ex, female Age > 75 and stroke count for 2 points each. Others are 1 point each. 2 points or more total = candidate for anticoagulation

Scales to assess symptoms of COPD

+ CAT (COPD Assessment Test) -- Cough -- Presence/amt of phlegm -- Chest tightness -- Breathlessness with stairs -- Activity limitations at home -- Confident to leave home -- Sleep quality -- Energy level + mMRC (Modified British Medical Research Council) Dyspnea Test -- Dyspnea with strenuous exercise (0) -- Dyspnea walking up small hill (1) -- Walking slower than people my age (2) -- Stop for breath after 100 yards (3) -- Too dyspneic to leave house (4)

Correcting dehydration in pediatric patients

+ Can try oral rehydration treatment (ORT) for mild & moderate dehydration + Mild: 3-5% (no loss of turgor) usually 50ml/kg plus losses over 4 hrs plus losses - if vomiting, try 5 ml every 3 min + Moderate: 6-9% (decreased turgor and urine below 1 ml/kg/h) usually 100 ml/kg over 4 hrs plus losses + Severe: 10% (decreased turgor with tenting and urine below 1 ml.kg/h). Admit for IV therapy! Add half to 2/3 of deficit to maintenance in first 8 hrs if isotonic or hyponatremic. If hypernatremic, give 25% of deficit every 12 hrs.

Indications for the use of beta blockers

+ Hypertension + post-myocardial infarction (first line), + angina, + arrhythmias, + migraine prophylaxis. Leik

Signs/symptoms of digoxin overdose?

+ Initial symptoms are gastrointestinal (anorexia, nausea/vomiting, abdominal pain). Others are arrhythmias, confusion, and visual changes (yellowish green tinged-color vision, scotomas). + Severe toxicity is treated with digoxin-binding antibodies (Digibind). Leik

Symptoms of primary hypothyroidism

+ Lethargy + Weight gain + Intolerance to cold + Constipation + Menstrual irregularities + Course, dry skin + Hair loss

Uses for the following penicillins: + Penicillin V PO QID + Amoxicillin BID to TID + Augmentin PO BID + Benzathine penicillin G IM + Dicloxacillin PO QID

+ Penicillin V PO QID - strep throat (1st line!) + Amoxicillin BID to TID - AOM (1st line) + Augmentin PO BID - AOM/sinusitis (1st/2nd line) + Benzathine penicillin G IM - Syphillis (1st line) + Dicloxacillin PO QID - Cellulitis (not caused by MRSA), impetigo, erysepelas

GOLD assessment of COPD

+ Symptoms (mMRC or CAT) + Airflow Limitation (1-4) + Exacerbations and/or hospitalizations per year + Spirometry

The most common symptoms of TIA include:

+ Weakness in an extremity + Abruptly slurred speech or partial loss of vision + Sudden gait changes

Adverse effects of beta blockers

+ bronchospasm, + bradycardia, + depression, fatigue, + erectile dysfunction, + blunts hypoglycemic resonse

Drugs/substances that interact with statins

- Grapefruit juice - Other fibrates (except fenofibrate) - Antifungals (intraconazole, ketoconazole) - Macrolides (erythromycin, clarithromycin, telithromycin) - Amiodarone (Cardorone), some CCBs (diltiazem, amlodipine, verapamil)

1. ADA Guidelines for Diabetes 2017

...

Normal PSA level

0-4 ng/mL = Normal 4 - 10 ng/mL = BPH ≥ 10 ng/mL = Suggestive of prostate CA

ADA dignosis of T2DM

1) 2 hour 75 g GTT ≥ 140 mg/dL 2) Fasting glucose ≥ 126 mg/dL on 2 separate occasions.

Treatment for rocky mountain spotted fever?

1) Early treatment is important and empiric treatment should be started early if RMSF is suspected. Refer the patient to the closest ED as soon aspossible. 2) It may be difficult to distinguish RMSF from meningococcemia before the blood culture results and the CSF culture results are available. 3) RMSF: ■ Dog/wood tick bite; spirochete called Rickettsia rickettsia. ■ Treat with doxycycline 100 mg PO/IV for a minimum of 7 days. Leik

According to the most recent American Heart Association guidelines, what would be the most appropriate treatment for the following patient? 40-year-old white male with history of type 2 diabetes whose blood pressure for the last 3 consecutive office visits has been greater than 140/90. 1. ACE inhibitor 2. Calcium Channel Blocker 3. Beta Blocker 4. Thiazide

1. ACE inhibitor According to ACC/AHA 2017 guidelines and their meta-analysis reviewed, patients presenting with the comorbidities of HTN and DM benefit most from an ACE inhibitor, due to the renal protective benefits of preventing complicated kidney disease.

When auscultating a patient's heart, you notice a harsh, rough murmur located at the apex and the murmur radiates to the carotid arteries. Which valvular heart disease does the patient mostly likely have? 1. Aortic Stenosis 2. Mitral Valve Prolapse 3. Mitral Stenosis 4. Aortic Regurgitation

1. Aortic Stenosis In aortic stenosis, an ejection click may come before the murmur, which is heard at the apex. The murmur is characterized by a harsh, rough sound that can radiate to the carotid arteries and upper interscapular area. The murmur occurs after S1 and before S2.

What type of immunity is less active in the elderly: 1. Cell-mediated immunity 2. Humoral immunity 3. Total immunity 4. Passive immunity

1. Cell-mediated immunity

Of the conditions listed below which are long term sequelae of eating disorders? 1. Osteoporosis 2. Constipation 3. Pericardial effusion 4. Early satiety

1. Osteoporosis Early satiety, constipation and pericardial effusion of short term consequences of eating disorders.

Tanner Stages Girls

1. Preadolescent breasts 2. Breast buds with areolar enlargement 3. Breast enlargement without separate 4. Areola and nipple project as secondary mound 5. Adult breast: Areola recedes, nipple retracts

A male with HIV infection has a CD4 count of less than 200 cells/mm3. Which of the following is an appropriate action: 1. Start patient on Bactrim DS daily 2. Schedule pentamidine nebulizer treatments 3. Recheck CD4 count 4. Recommend vaccination against Herpes zoster.

1. Start patient on Bactrim DS daily Tip: CD4 count of 200 or less, start prophylaxis against "PCP" pneumonia (new name organism is P. jirovicii). ​Zoster vaccine (Zostrix) is contraindicated in immune-compromised patients.

Most children can understand abstract concepts (early abstract thinking) and are better at logical thinking at what age? How to test?

11 years. You can ask about proverbs to assess abstract thinking Leik

4) A 35 year old male with a BMI of 28 is seen for a routine PE. Vital signs are: BP 134/80, Pulse 80/min., Resp. 20/min. The lipid profile results are: HDL 35, LDL 120. FBG is 120 mg/dL. What is the best initial treatment plan to follow for this patient: 1) Advise the patient that he needs to lose 40 pounds 2) Recommend moderate exercise for at least 30 minutes for most days of the week 3) Initiate a prescription of Zocor 20 mg daily 4) Refer the patient to a cardiologist for a stress test

2) Recommend moderate exercise for at least 30 minutes for most days of the week

Which of the following statements regarding adolescent substance abuse is a true statement? 1. A family history of drug addiction is poorly correlated with adolescent substance abuse 2. A CAGE questionnaire score of 2 or greater is highly suggestive of substance abuse in an adolescent 3. The AAP recommends urine and serum screening for drug use in the adolescent as routine screening. 4. Tertiary level prevention of adolescent substance abuse is aimed at those with increased risk for substance abuse.

2. A CAGE questionnaire score of 2 or greater is highly suggestive of substance abuse in an adolescent Using the CAGE questionnaire is an effective screening tool to determine risk of substance abuse in the adolescent.

6 year old boy presents with complaints of fatigue, tea colored urine, and bilateral periorbital edema. The symptoms started 3 days ago. His mother reports that he had a fever and sore throat 10 days ago. His urinalysis shows proteinuria, hematuria, and negative for leukocytes and nitrites. He has elevated serum antistreptococcal antibodies and decreased serum C3 complement. What is most likely his diagnosis? 1. Lupus Glomerulonephritis 2. Acute Post infectious Glomerulonephritis 3. Acute Cystitis 4. IgA Nephropathy

2. Acute Post infectious Glomerulonephritis Acute Post infectious Glomerulonephritis cannot be diagnosed with elevated antistreptococcal antibodies alone, but in combination with depressed serum C3 complement. +Lupus Glomerulonephritis will have depressed serum C3 complement, but will not have elevated antistreptococcal antibodies. +Acute Cystitis would present different clinically, and would have positive leukocytes and nitrites in the urinalysis. +IgA Nephropathy will not have a depressed serum C3 complement.

Which of the following medications is considered a high intensity statin? 1. Pravastatin 80mg 2. Rosuvastatin 40mg 3. Simvastatin 40mg 4. Atorvastatin 20mg

2. Rosuvastatin 40mg high intensity statins can lower LD cholesterol by about 50%. These include rosuvastatin 20-40mg and atorvastatin 40-80mg. Moderate intensity statins include atorvastatin 10-20mg, rosuvastatin 5-10mg, lovastatin 40mg, pravastatin 40-80mg, and simvastatin 20-40mg.

1) Which of the following laboratory tests are recommended for an adult male with a podagra who is on furosemide (Lasix) 20 mg BID: 1. CBC with differential 2. Serum uric acid 3. Serum creatinine 4. 24-hour urine for protein and creatinine clearance

2. Serum uric acid

A 70 year old ex-smoker has a diagnosis of COPD, hypertension, and DJD. The color of the patient's phlegm has changed to a green color the past few days. The patient is not febrile, but has been lethargic for the past few days. Which of the following can treat the source of the patient's new symptoms: 1. Oxygen at 2/L by nasal cannula 2. Start patient on an antibiotic 3. Albuterol: saline nebulizer treatments 4. Increase fluid intake because many elderly patients become dehydrated when ill

2. Start patient on an antibiotic

2) The cones of the retina are responsible for what type of vision: 1. peripheral vision and night vision 2. central vision and 20/20 vision 3. peripheral vision 4. night vision

2. central vision and 20/20 vision

target INR for warfarin

2/3

Which of the following are co-morbid conditions are often associated with substance abuse in adolescents? 1. Attention deficit hyperactivity disorder and bipolar disorder 2. Anxiety and depression 3. All of the above 4. None of the above

3. All of the above All of the above conditions are associated with an increased susceptibility to substance abuse in the adolescent.

Treatment/intervention in eating disorders is most effective when it is: 1. Outpatient management only 2. Inpatient approach first, then outpatient management. 3. Initiated after assessing multiple factors including severity and duration of illness 4. None of the above

3. Initiated after assessing multiple factors including severity and duration of illness Treatment and intervention of eating disorders may take place in outpatient or inpatient settings or both depending on several factors including both the severity and duration of illness

3) All of the following are correct statements about G6PD anemia except: 1. It is classified as a hemolytic anemia 2. The intake of sulfa-containing drugs should be avoided 3. These patients are at higher risk of autoimmune disorders 4. Foods such as fava beans can precipitate hemolysis in these patients.

3. These patients are at higher risk of autoimmune disorders

Patients with community-acquired pneumonia (CAP) or who have a history of smoking or have COPD are are at higher risk for CAP that is caused by: 1. Strep pneumoniae 2. Mycoplasma pneumoniae 3. Pseudomonas aeroginosa 4. Hemophilus influenzae

4. Hemophilus influenzae Not as common but often seen in patients with underlying lung disease (like smokers)

Patient presents to clinic with complaint of erectile dysfunction, which of his current medications would prohibit him from receiving a phosphodiesterase inhibitor? 1. amphetamine salt (Adderall) BID 2. Atorvastatin at bedtime 3. Monthly IM testosterone injections 4. PRN sublingual nitroglycerin

4. PRN sublingual nitroglycerin According to Kloner's study referenced above, the concomitant use of phosphodiesterase inhibitor and nitrates have proven to be a fatal combination resulting in episodes of hypotension severe enough to result in hypotensive injury to the brain, cardiac shock, and death. The avoidance of concurrently prescribing/using these drugs is essential to patient safety.

CDC BMI for age growth charts to track child's weight status

95 - 100%: obese 85 - 95%: overweight 5 - 85%: normal <5%: underweight

A five-year-old patient comes into the clinic for a well-child visit and his visual acuity is 20/50. How should the NP proceed? a. Refer b. tell parents this is normal for his age c. perform a fundoscopic exam with an ophthalmoscope d. assess for nystagmus

A - REFER! Children ages 3-5 years of age should be referred for visual acuity of less than 20/40 or 10/20 in either eye or a two-line difference between the eyes. Question pool

The 72 year old patient presents with severe eye pain with onset this morning, seeing halos around lights, and blurred vision. Assessment reveals an erythematous hard eye, cloudy cornea, and nonreactive dilated pupil. The NP suspects acute angle-closure glaucoma and knows the next course of action to be? a. Refer urgently to ophthalmology b. Prescribe Prostaglandin eye drops. c. Betamethasone eye drops and return if no improvement in 3 days. d. Discontinue Spiriva for existing COPD due to anticholinergic effects.

A - angle closure glaucoma is a medical emergency. Patient will present with severe acute eye pain and blurred vision described as halos around lights. There is usually a preexisting narrow anterior chamber angle that may or may not be known to patient. The physical exam will reveal reddened eye hard on palpation with cloudy cornea. Treatment is immediate reduction of intraocular pressure by immediate referral to ophthalmology to prevent severe or total visual loss if untreated within 2-5 days after onset Question pool

What is the preferred treatment for erosive esophagitis?

A PPI Leik

17 y/o male patient presents with sore throat that started abruptly 2 days ago. He reports "feeling hot" but has not taken his temperature. Vitals: 100/65 Pulse: 98 R: 18 Temp: 101.1 F / 38.4 C (oral). Exam reveals bilateral tonsillar exudate and cervical lymphadenopathy. What medication will you prescribe? a. Penicillin V b. Nitrofurantoin c. ciprofloxacin d. doxycycline

A Pen V Acute bacterial pharyngitis is diagnosis with the presence of a sore throat and at least one: fever > 38.3 C, tonsillar exudates, or a positive group A β-hemolytic streptococcus culture. Group A strep infections are should be treated with Pen V PO or Benzathine PCN IM. Alternative treatments include: amoxicillin, cephalexin, clindamycin and azithromycin.

Hidradentis suppurativa: Presentation and treatment

A bacterial infection of the sebaceous glands of the axilla (or groin) by Staphylococcus aureus (gram positive) that frequently becomes chronic. It is marked by fl are-ups and resolution. Usually both axillae are involved. Chronic episodic infection eventually leaves sinus tracts and heavy scarring. Classic Case Patient complains of an acute onset of painful, large, dark-red nodules and papules under one or both axillae that become abscessed. Ruptured lesions drain purulent greencolored discharge. Pain resolves when the abscess drains and heals. History of recurrent episodes on the same areas in the axilla results in sinus tracts and multiple scars. Labs C&S of purulent discharge. Treatment Plan ■ Amoxicillin/clavulanate (Augmentin) PO BID or dicloxacillin TID × 10 days. ■ Mupirocin ointment to lower third of nares and under fingernails BID × 2 weeks. ■ Use antibacterial soap (e.g., Dial), especially on axilla and groin areas. ■ Avoid underarm deodorants during acute phase.

What is torus palatinus?

A bony overgrowth at the midline of the hard palate. Leik

Common clinical presentation of lupus or SLE, and patient teaching?

A malar rash (butterfl y rash) is present in most patients with lupus. These patients should be advised to avoid or to minimize sunlight exposure (photosensitivity). LEIK

Dubowitz Score

A method of estimating the gestational age of a Neonate based on 21 defined clinical signs. This method provides the correct gestational age +/- 2 weeks in 95% of infants. A score of 40 indicates a gestational age of 40 weeks.

The span of the normal adult liver is: A) 15 to 18 cm in the midclavicular line B) 12 to 16 cm in the right midclavicular line C) 2 to 6 cm in the midsternal line D) 4 to 8 cm in the midsternal line

A) 15 to 18 cm in the midclavicular line

The posterior fontanel should be completely closed by: A) 3 months B) 4 months C) 5 months D) 6 months

A) 3 months The posterior fontanelle normally closes by 3 months of age. The anterior fontanelle closes between 12 and 18 months of age.

Which of the following would be classified as a second-degree burn? A) A severe sunburn with blistering B) Burns that involve the subcutaneous layer of skin C) A reddened fi nger after touching a hot iron D) Burns that involve eschar

A) A severe sunburn with blistering Burns: First degree: erythema without blistering, painful; second degree: red skin with superficial blisters, painful; third degree: entire skin layering, involving the subcutaneous tissue and may include soft tissue facia, painless.

Cullen's sign is most commonly associated with which of the following? A) Acute pancreatitis B) Myocardial infarction C) Acute pyelonephritis D) Preeclampsia

A) Acute pancreatitis Cullen's sign is commonly seen in acute pancreatitis. It is a yellowish-blue skin color change around the umbilicus. It is thought to occur due to the pancreatic enzymes that run along the ligament and subcutaneous tissues around the umbilicus.

Clara is a 20-year-old college student who reports to the student health clinic with a laceration in her left hand. She tells the nurse practitioner that she cut her hand while working in her garden. Her last Td booster was 5.5 years ago. Which of the following is correct? A) Administer a booster dose of the Td vaccine B) Administer the Td vaccine and the Td immunoglobulin (HyperTet) C) Administer Td immunoglobulin (HyperTet) only D) She does not need any Td immunoglobulin (Hypertet) or a Td booster

A) Administer a booster dose of the Td vaccine Td booster vaccine is recommended for recent injuries if it has been more than 5 years since the last Td.

A 14-year-old female student complains of pain and fullness in her left ear that is getting steadily worse. She has a history of allergic rhinitis and is allergic to dust mites. On physical exam, the left tympanic membrane is red with cloudy fluid inside. The landmarks are displaced in the same ear. The student denies frequent ear infections, and the last antibiotic she took was 8 months ago for a urinary tract infection. She is allergic to sulfa and tells the NP that she will not take any erythromycin because it makes her very nauseated. Which of the following is the best choice of treatment for this patient? A) Amoxicillin 500 mg PO three times a day for 10 days B) Pseudoephedrine (Sudafed) 20 mg PO as needed every 4 to 6 hours C) Fluticasone (Flonase) nasal inhaler 1 to 2 sprays each nostril every 12 hours D) Biaxin (clarithromycin) 500 mg PO two times a day for 10 days

A) Amoxicillin 500 mg PO three times a day for 10 days Best Clues 1) Red tympanic membrane with cloudy fl uid inside and displaced landmarks. 2) Last antibiotic taken was 8 months ago and ear infections were infrequent (lack of risk factors for beta-lactamase resistant bacteria). Notes 1) You must first arrive at the correct diagnosis, which is AOM. 2) Amoxicillin is the preferred first-line antibiotic for both AOM and acute sinusitis in both children and adults (for patients with no risk factors for resistant organisms). 3) The ideal patient is someone who has not been on any antibiotics in the past 3 months and/or does not live in an area with high rates of beta-lactam resistant bacteria. 4) If the patient is a treatment failure, or was on an antibiotic in the previous 3 months, then a second-line antibiotic such as Augmentin BID or cefdinir (Omnicef) BID should be given. 5) If penicillin-allergic, an alternative is azithromycin (Z-Pack) and clarithromycin (Biaxin) BID. 6) Pseudoephedrine (Sudafed) is for symptoms only. Do not use for infants, young children, or patients with hypertension. 7) Nasal steroid spray BID is a good adjunct treatment for this patient because of allergic rhinitis, which causes the eustachian tube to swell and get blocked. Leik

The Jarisch-Herxheimer reaction is best described as: A) An immune-mediated reaction precipitated by the destruction of a large number of spirochetes due to an antibiotic injection B) Severe chills and elevated blood pressure C) Caused by infection with either Chlamydia trachomatis or gonorrheal infection of the liver capsule D) Associated with certain viral illnesses

A) An immune-mediated reaction precipitated by the destruction of a large number of spirochetes due to an antibiotic injection The Jarisch-Herxheimer reaction is defined as an immune-mediated reaction precipitated by the destruction of a large number of spirochetes due to the antibioitic injection.

A 55-year-old male patient describes to you an episode of chest tightness in his substernal area that radiated to his back while he was jogging. It was relieved immediately when he stopped. Which of the following conditions does this best describe? A) Angina pectoris B) Acute myocardial infarction C) Gastroesophageal refl ux disease D) Acute costochondritis

A) Angina pectoris Symptoms of angina pectoris include a pressing, squeezing, or crushing pain, usually in the chest or back, arms, neck, or ear lobes. Weakness, fatigue, and shortness of breath and pain radiating in the arms, shoulders, jaw, neck, and/or back may also occur. Angina pectoris is most commonly brought on by physical exertion, but can also be triggered by stress and other stressors to the body.

You can determine a pulse deficit by counting the: A) Apical and radial pulses at the same time, then finding the difference between the two B) Apical pulse first, then the radial pulse, and subtracting to find the difference between the two C) Apical pulse and the femoral pulse at the same time and finding the difference between the two D) Radial pulse first, then counting the femoral pulse, and subtracting to find the difference between the two

A) Apical and radial pulses at the same time, then finding the difference between the two

Patients who are being screened for TB and are immunocompromised should be evaluated for anergy. Which of the following is the best description of anergy testing? A) Apply Candida or mumps antigen to the right forearm and the PPD on the left forearm and read results in 48 to 72 hours B) Apply Candida or mumps antigen and PPD on left forearm only and check for a reaction in 24 hours C) Mix the Candida or mumps antigen with the PPD and apply it to both forearms D) Apply the Candida or mumps antigen 24 hours before the PPD on the left forearm

A) Apply Candida or mumps antigen to the right forearm and the PPD on the left forearm and read results in 48 to 72 hours Anergy is when the immune system is unable to perform a healthy, normal immune response when the body is challenged with a particular antigen.

While doing a cardiac exam on a 45-year-old male, you note an irregular rhythm with a pulse rate of 110 beats per minute. The patient is alert and is not in distress. What is the most likely diagnosis? A) Atrial fibrillation B) Ventricular fibrillation C) Cardiac arrhythmia D) First-degree right bundle branch block

A) Atrial fibrillation Atrial fibrillation is chaotic electrical activity of the heart, caused by several ectopic foci in the atria without any signs of distress.

Which of the following antibiotics is preferred treatment for healthy adults diagnosed with uncomplicated CAP? A) Azithromycin (Zithromycin) 500 mg on day 1, then 250 mg daily for 4 days. B) Dextromethorphan with guaifenesin (Robitussin diabetes mellitus [DM]) 1 to 2 teaspoons PO QID as needed C) Cephalexin (Kefl ex) 500 mg PO QID × 10 days D) Levofloxacin (Levaquin) 500 mg PO daily × 7 days

A) Azithromycin (Zithromycin) 500 mg on day 1, then 250 mg daily for 4 days.

A chest radiograph shows an area of consolidation on the lower lobe. Which of the following conditions is most likely? A) Bacterial pneumonia B) Acute bronchitis C) COPD D) Atypical pneumonia

A) Bacterial pneumonia Consolidation is visible on x-ray when bacterial pneumonia is present. Consolidation is not present in the lungs with bronchitis, COPD, or atypical pneumonia

Which of the following antihypertensive medications has beneficial effects for an elderly White female with osteoporosis? A) Calcium channel blocker B) Angiotensin-converting enzyme (ACE) inhibitor C) Beta-blocker D) Diuretic

A) Calcium channel blocker Calcium channel blockers act by blocking the calcium channels in the heart muscle and the blood vessels, thereby keeping more calcium in the bones.

The cones in the retina of the eye are responsible for: A) Central vision B) Peripheral vision C) Night vision D) Double vision

A) Central vision

An 18-year-old female has a positive throat C&S for Strep pyogenes (Group A beta streptococci). The patient reports a history of an allergic reaction to penicillin with "swollen lips" accompanied by urticaria. Which of the following is the most appropriate treatment? A) Clarithromycin (Biaxin) 250 mg PO BID × 10 days B) Gargle with salt water 3 times a day C) Cephalexin (Keflex) 250 mg PO QID × 10 days D) Doxycycline 100 mg PO BID × 10 days

A) Clarithromycin (Biaxin) 250 mg PO BID × 10 days Best Clues ■ Positive C&S for strep. ■ Report of a penicillin allergy. ■ Rule out the following options because: - Option B (gargling with salt water is for symptoms and will not eradicate strep). - Option C (penicillin-allergic patients may also be allergic to cephalosporins). - Option D (doxycycline not effective for gram-positive infections).

All of the following findings are associated with the secondary stage of an infection by the organism Treponema pallidum except: A) Condyloma acuminata B) Maculopapular rash of the palms and soles C) Lymphadenopathy D) Condyloma lata

A) Condyloma acuminata Findings consistent with the diagnosis of syphilis, caused by the Treponema pallidum organism, include maculopapular rash of the palms and soles, lymphadenopathy, and condyloma lata. Condyloma acuminata (genital warts) are caused by the human papilloma virus (HPV) and spread to others by skin-to-skin contact.

Which of the following viral infections is associated with occasional abnormal forms of lymphocytes during an acute infection? A) Cytomegalovirus (CMV) B) Epstein-Barr virus (EBV) C) Human papilloma virus (HPV) D) Coxsackie virus

A) Cytomegalovirus (CMV) A test for an acute infection of the Epstein-Barr virus will show abnormal forms of lymphocytes.

Which of the following is recommended treatment for erythema migrans or early Lyme disease? A) Doxycycline (Vibramycin) 100 mg PO BID x 21 days B) Ciprofl oxacin (Cipro) 250 mg PO BID x 14 days C) Erythromycin (E-mycin) 333 mg PO TID x 10 days D) Dicloxacillin 500 mg PO BID x 10 days

A) Doxycycline (Vibramycin) 100 mg PO BID x 21 days The CDC-recommended treatment for erythema migrans or Lyme disease is doxycycline 100 mg PO BID x 21 days.

The mother of an 8-year-old boy reports the presence of a round red rash on the child's left lower leg. It appeared 1 week after the child returned from visiting his grandparents, who live in Massachusetts. During the skin exam, the maculopapular rash is noted to have areas of central clearing making it resemble a round target. Which of the following is best described? A) Erythema migrans B) Rocky Mountain spotted fever C) Meningococcemia D) Larva migrans

A) Erythema migrans Erythema migrans is an annular lesion that enlarges and with time has central clearing. This is commonly caused by a bite from an infected tick and can cause Lyme disease if not treated Appropriately. Lyme disease is most commonly seen in the northeastern United States.

The majority of serum alpha fetoprotein is produced by the: A) Fetal liver B) Mother's liver C) Placenta D) Fetal neural tube

A) Fetal liver

Erythromycin 200 mg with sulfisoxazole 600 mg suspension (Pediazole) is contraindicated in which of the following conditions? A) G6PD deficiency anemia B) Lead poisoning C) Beta thalassemia minor D) B12 deficiency anemia

A) G6PD deficiency anemia Glucose-6-phosphate dehydrogenase (G6PD) deficiency is a hereditary condition that occurs when the red blood cells break down, causing hemolysis, due to a missing or lack of a sufficient enzyme that is needed to help the red blood cells work efficiently. Certain foods and medications may trigger this reaction and hemolysis will occur. Some of these medications include antimalarial drugs, aspirin, nitrofurantoin, NSAIDs, quinidine, quinine, and sulfa medications.

Garlic is a medicinal herb most commonly used to treat? A) Hypertension B) Migraines C) Peptic Ulcers D) Diabetes

A) Hypertension * Garlic is one of the most popular medicinal herbs in Europe and the United States. It has been used since antiquity for the treatment of cardiovascular and infectious diseases. Several mechanisms for garlic's beneficial effects on cardiovascular disease have been proposed: decreased platelet aggregation and adhesion, increased fibrinolysis, antioxidant activity, blood pressure lowering, and decreased low-density lipoprotein (LDL) cholesterol synthesis.

All of the following are considered selective serotonin reuptake inhibitors (SSRIs) except: A) Imipramine (Elavil) B) Fluoxetine (Prozac) C) Sertraline (Zoloft) D) Paroxetine (Paxil CR)

A) Imipramine (Elavil) Prozac, Zoloft, and Paxil CR are selective serotonin reuptake inhibitors. Elavil is a tricyclic antidepressant.

A 44-year-old female who is undergoing treatment for infertility complains of not having a menstrual period for a few months. The night before, she started spotting and is now having cramp-type pains in her pelvic area. Her blood pressure (BP) is 160/80, the pulse rate is 110, and she is afebrile. Her labs reveal a mild anemia with mild leukocytosis. On pelvic exam, the uterine fundus is found above the symphysis pubis. The cervical os is dilated at 3 cm. Which of the following is most likely? A) Inevitable abortion B) Threatened abortion C) Incomplete abortion D) Acute pelvic infl ammatory disease

A) Inevitable abortion Inevitable abortion is defined as vaginal bleeding with pain and cervical dilation and/or effacement. Threatened abortion is defined as vaginal bleeding with absent or minimal pain and a closed, long, and thick cervix. Incomplete abortion is moderate to diffuse vaginal bleeding, with the passage of tissue and painful uterine cramping or contractions. Acute pelvic inflammatory disease is a sudden onset of infl ammation and pain that affects the pelvic area, cervix, uterus, and ovaries, which is caused by infection.

You are reviewing the bilirubin level on a 3-day-old full-term neonate. You note that it is 10 mg/dL. The infant has a slight yellow color to his skin, mucous membranes, and sclera. The infant is feeding well, is not irritable, and has 8 to 10 wet diapers per day. Which of the following is a true statement? A) Keep on monitoring the infant's bilirubin level until it returns back to normal in about 1 week B) Recommend that the infant be treated with phototherapy 10 minutes a day until the bilirubin level is back down to a normal range C) Refer the infant to a neonatologist as soon as possible D) Refer the infant to the neonatal intensive care unit

A) Keep on monitoring the infant's bilirubin level until it returns back to normal in about 1 week Bilirubin is excreted through the urine and feces. Increased fluids and wetting 8-10 diapers a day is sufficient fluid intake/excretion to help bring down the bilirubin level. Levels should continue to be monitored and should improve in approximately 1 week.

A patient who recently returned from a vacation in Latin America complains of a severe headache and stiff neck that are accompanied by a high fever for the past 12 hours. While examining the patient, the nurse practitioner fl exes both the patient's hips and legs and then tells the patient to straighten them against resistance. The name of this test is: A) Kernig's maneuver B) Brudzinski's maneuver C) Murphy's sign D) Homan's sign

A) Kernig's maneuver Kernig's maneuver is performed by having the patient flex both hips and legs and then straighten the legs against resistance, testing for meningitis. Flexion of hip/knees is a positive sign for meningitis. Brudzinski's maneuver is performed by placing the patient's hands behind his head, and gently tucking chin to chest. If pt flexes knees, it is a positive sign for meningitis. Murphy's sign is elicited by having the patient inspire with the tips of the examiner's fingers placed on the right upper quadrant, at the liver border, under the ribs. Pain on inspiration is suggestive of cholecystitis. Homan's sign is flexion of the foot, causing pain in the posterior calf area, suggestive of a DVT.

Human papilloma virus (HPV) infection of the larynx has been associated with: A) Laryngeal neoplasia B) Esophageal stricture C) Cervical cancer D) Metaplasia of esophageal squamous cells

A) Laryngeal neoplasia

The following abnormal lab results may be seen in patients with acute mononucleosis EXCEPT: A) Lymphocytosis and/or atypical lymphocytes B) Positive EBV titers for immunoglobulin (Ig) M and Ig G C) Elevated liver function tests D) Elevated creatinine and BUN

A) Lymphocytosis and/or atypical lymphocytes Common symptoms of mononucleosis include positive EBV titers, elevated liver enzymes, and elevated creatinine and BUN.

A previously healthy 30-year-old complains of an acute onset of fever and chills accompanied by a productive cough with purulent sputum and a loss of appetite. The patient denies receiving an antibiotic in the previous 3 months. The NP diagnoses community-acquired pneumonia (CAP). The Infectious Diseases Society of America (IDSA) and the American Thoracic Society (ATS) treatment guidelines recommend which of the following as the preferred first-line treatment for this patient? A) Macrolides B) Antitussives C) Cephalosporins D) Fluoroquinolones with gram-positive bacteria activity

A) Macrolides According to the IDSA and the ATS treatment guidelines, outpatient treatment of CAP in healthy patients (no comorbidities) are the macrolides (azithromycin, clarithromycin, or erythromycin). Leik

A 25-year-old woman complains of dysuria, severe vaginal pruritis, and a malodorous vaginal discharge. Pelvic examination reveals a strawberry-colored cervix and frothy yellow discharge. Microscopic exam of the discharge reveals mobile organisms that have flagella. The correct pharmacologic therapy for the condition is: A) Metronidazole (Flagyl) B) Ceftriaxone sodium (Rocephin) C) Doxycycline hyclate (Vibramycin) D) Clotrimazole (Gyne-Lotrimin)

A) Metronidazole (Flagyl) Trichomoniasis symptoms include dysuria, severe vaginal pruritis, and malodorous vaginal discharge. Wet prep will show trichomonads that are pear-shaped and have several flagella (whiplike tails) at one end. CDC recommendation for treatment is metronidazole.

Stella works in the housekeeping department of a hospital. She presents to the employee health clinic with a complaint of a needlestick to her left thumb. The needle was in one of the garbage bags from the emergency room. The patient had a little bleeding that stopped spontaneously. Which of the following is the next step? A) Order an enzyme-linked immunosorbent assay (ELISA) test as soon as possible B) Recommend a tetanus booster in 1 week C) Offer the patient hepatitis B immunoglobulin D) Order a chest x-ray

A) Order an enzyme-linked immunosorbent assay (ELISA) test as soon as possible Employee health clinic protocols for needlesticks recommend ordering an ELISA test as soon as possible to establish baseline blood work for the employee.

A second triple screen on a 35-year-old primigravida reveals abnormally low levels of the alpha fetoprotein and estriol and high levels of human chorionic gonadotropin. Which of the following interventions is the best choice for this patient? A) Order an ultrasound B) Order a computed tomography (CT) scan of the abdomen C) Order a 24-hour urine for protein clearance D) Assess for a history of illicit drug or alcohol use

A) Order an ultrasound Abnormally low levels of alpha fetoprotein and estriol and high levels of human chorionic gonadotropin are abnormal during pregnancy. An ultrasound should be ordered to further evaluate the fetus for characteristics of Down syndrome and/or fetal demise.

The positive signs of pregnancy include: A) Palpation of the fetus and auscultation of the fetal heart tones by the nurse practitioner B) Palpation of the fetus and a positive quantitative serum pregnancy test C) Fetal heart tones and a positive quantitative serum pregnancy test D) Fetal heart tones and feeling of movement of the baby by the mother

A) Palpation of the fetus and auscultation of the fetal heart tones by the nurse practitioner + *Presumptive signs* of pregnancy are symptoms experienced by the woman, such as amenorrhea, breast tenderness, nausea/vomiting, fatigue, and increased urinary frequency. + *Probable signs* of pregnancy are signs detected by the examiner, such as an enlarged uterus. + *Positive signs* of pregnancy are direct evidence of pregnancy such as audible fetal heart tones or cardiac activity on ultrasound.

Which of the following medications is indicated for the treatment of obsessive-compulsive disorder? A) Paroxetine (Paxil CR) B) Haldoperidol (Haldol) C) Lorazepam (Xanax) D) Imipramine (Elavil)

A) Paroxetine (Paxil CR) Antidepressants are the most common medications used for OCD. Those antidepressants that are approved for OCD by the Food and Drug Administration (FDA) include clomipramine (Anafranil), fluvoxamine (Luvox), fluoxetine (Prozac), paroxetine (Paxil, Pexeva), and sertraline (Zoloft).

Which of the following is most likely to be found in patients with a long-standing case of iron-deficiency anemia? A) Pica B) Fatigue C) Pallor D) Irritability

A) Pica Notes 1) If you are guessing, use common sense. Fatigue and irritability are found in many conditions. 2) Pallor is also seen in many disorders such as shock, illness, and anemia. 3) By the process of elimination, you are left with option A, the correct choice. 4) Another specific clinical finding in iron-deficiency anemia is spoonshaped nails (or koilonychia). Do not confuse this finding with pitted nails (psoriasis). Leik

Human chorionic gonadotropin (hCG) is produced by the: A) Placenta B) Hypothalamus C) Anterior pituitary D) Ovaries

A) Placenta

A 55-year-old woman who has type 2 diabetes is concerned about her kidneys. She has a history of 3 urinary tract infections within the past 8 months. She denies dysuria and frequency at this visit. Which of the following is the best initial course to follow? A) Recheck the patient's urine and order a urine for culture and sensitivity B) Order an IVP (intravenous pyelogram) C) Advise the patient to follow up with a urologist D) Evaluate the patient for a possible kidney infection

A) Recheck the patient's urine and order a urine for culture and sensitivity A urinary tract infection is defined as the presence of 100,000 organisms per mL of urine in asymptomatic patients or greater than 100 organisms per mL or urine with pyuria (more than 7 WBCs/mL) in a symptomatic patient.

Joe, a 20-year-old Asian male, reports pain in his right knee after twisting it playing soccer. The injured knee locks up when he attempts to straighten his leg. Which of the following is the best course of treatment for this patient? A) Refer him to an orthopedic specialist B) Refer him to a chiropractor C) Advise him that the clicking noise will resolve within 2 to 4 weeks D) Advise him to use an Ace bandage wrap during the first 2 weeks for knee support and to see you again for reevaluation

A) Refer him to an orthopedic specialist Referral to orthopedics is advised for evaluation of the need for treatment and surgery. Delaying referral and treatment can be detrimental if the diagnosis is not correct.

A 28-year-old male nurse tells the employee health nurse practitioner that he was treated for a UTI twice the previous year. The patient denies fever, fl ank pain, or urethral discharge during the visit. Which of the following is the best follow-up for this patient? A) Refer the patient to a urologist B) Prescribe the patient ofloxacin (Floxin) for 2 weeks instead of 1 week C) Advise the patient that he needs to void every 2 hours when awake D) Refer the patient to the local ED, because he has a very high risk of sepsis

A) Refer the patient to a urologist It is recommended that frequent UTIs in males be referred to a urologist for evaluation and treatment.

An adolescent male reports the new onset of symptoms 1 week after returning from a hiking trip in North Carolina. He presents with complaints of high fever, severe headache, muscle aches, and nausea. The symptoms are accompanied by a generalized red rash that is not pruritic. The rash initially appeared on both ankles and wrists and then spread toward the patient's trunk. The rash involves both the palms and the soles. Which of the following is the best treatment plan to follow? A) Refer the patient to the hospital ED. B) Refer the patient to an infectious disease specialist. C) Initiate a prescription of oral glucocorticoids. D) Collect a blood specimen for culture and sensitivity.

A) Refer the patient to the hospital ED. Best Clues 1) Knowledge of the emergent nature of RMSF (can cause death if not treated within the first 8 days of symptoms). Leik

The bell of the stethoscope is best used for auscultation of which of the following? A) S3 and S4 and low-pitched tones B) S3 and S4 only C) S1 and S2 and high-pitched tones D) S1 and S2 only

A) S3 and S4 and low-pitched tones

Which of the following drug classes is indicated as first-line treatment of both major depression and OCD? A) Selective serotonin reuptake inhibitors (SSRIs) B) Tricyclic antidepressants (TCAs) C) Mood stabilizers D) Benzodiazepines

A) SSRIs Best Clues 1) Rule out benzodiazepines, which are used to treat anxiety or insomnia (process of elimination). 2) Mood stabilizers such as lithium salts are used to treat bipolar disorder (process of elimination). 3) The stem is asking for the "first-line treatment" for depression, which is the SSRIs. Notes 1) TCAs are considered second-line treatment for depression. 2) TCAs are also used as prophylactic treatment of migraine headaches, chronic pain, and neuropathic pain (i.e., tingling, burning) such as postherpetic neuralgia. Example of TCA: amitriptyline (Elavil), nortriptyline (Pamelor). 3) Do not give suicidal patients a prescription for TCAs because of the high risk of hoarding the drug and overdosing. Overdose of TCAs can be fatal (cardiac and CNS toxicity). 4) SSRIs are also first-line treatment for OCD, generalized anxiety disorder (GAD), panic disorder, social anxiety disorder (extreme shyness), and premenstrual mood disorder (fluoxetine or Prozac). Examples of SSRIs: citalopram (Celexa), escitalopram (Lexapro), fluoxetine (Prozac), sertraline (Zoloft), paroxetine (Paxil). 5) Anticonvulsants such as carbamazepine (Tegretol) are also used for chronic pain and trigeminal neuralgia. Leik

Which drug class is the first line treatment for a 75 yr old male recently diagnosed with major depressive disorder? A) Selective Serotonin Reuptake Inhibitors (SSRIs) B) Benzodiazepines C) Antipsychotics D) Acetylcholinesterase Inhibitors

A) Selective Serotonin Reuptake Inhibitors (SSRIs) SSRIs particularly Sertraline are often used as first line agents because of their relatively benign side-effect profile. Benzodiazepines are avoided in the elderly.

Which of the following laboratory values may be found elevated alone on the liver function panel of patients who are alcohol abusers? A) Serum GGT (gamma glutamyl transaminase) B) Serum ALT (alanine aminotransferase) C) Serum bilirubin D) Blood urea nitrogen

A) Serum GGT (gamma glutamyl transaminase)

An elderly male of Mediterranean descent has a routine complete blood count (CBC) done for an annual physical. The following are his lab test results: hemoglobin of 13.0 g/dL, a hematocrit of 39%, and an MCV of 72 fL. His PSA result is 3.2 ng/mL. The urinalysis shows no leukocytes and few epithelial cells. Which of the following laboratory tests are indicated for this patient? A) Serum iron, serum ferritin, total iron binding capacity (TIBC), and the red cell differential width (RDW). B) Serum B12 and folate level with a peripheral smear. C) CBC with white cell differential and urinalysis. D) Urine culture and sensitivity with microscopic exam of the urine

A) Serum iron, serum ferritin, TIBC, and the red cell differential width (RDW). Best Clues 1) Low hemoglobin and hematocrit for gender (male) and age (abnormal CBC result). 2) An MCV of 72 fL is indicative of microcytic anemia (assessment). 3) The ethnic background of the patient (demographics). 4) Ignore the urinalysis and PSA tests since they are not necessary to solve the problem. Notes: In iron-deficiency anemia, the following results are found: ■ Decreased (serum ferritin and serum iron levels). ■ Elevated (TIBC/transferrin levels and the RDW). In alpha or beta thalassemia trait or minor, the following results are found: ■ Normal to high (serum ferritin and serum iron levels) ■ Normal (TIBC/transferrin levels) Leik

If left untreated, Zollinger-Ellison syndrome can cause which of the following? A) Severe ulceration of the stomach or duodenum B) Toxic megacolon C) Chronic diarrhea D) Malabsorption of fat-soluble vitamins

A) Severe ulceration of the stomach or duodenum Untreated Zollinger-Ellison syndrome can lead to severe ulceration of the stomach or duodenum. Zollinger-Ellison syndrome occurs when tumors (gastrinomas) in the intestine, pancreas, or lymph nodes near the pancreas produce excessive amounts of gastrin, which in turn will increase the amount of acid produced by the stomach. High amounts of acid in the stomach will produce ulceration of the stomach or duodenum.

Carol, a 30-year-old type 2 diabetic, is on regular insulin and lente insulin in the morning and in the evening. She denies changes in her diet or any illness, but recently started attending aerobic classes in the afternoon. Because of her workouts, her blood sugars have dipped below 50 mg/ dL very early in the morning. Her fasting blood sugar before breakfast is now elevated and higher than normal. Which of the following is best described? A) Somogyi phenomenon B) Dawn phenomenon C) Raynaud's phenomenon D) Insulin resistance

A) Somogyi phenomenon The Somogi phenomenon is when nocturnal hypoglycemia (2 a.m to 3 a.m.) stimulates the liver to produce glucagon to raise the blood sugar. The fasting blood glucose levels will be elevated from this glucagon production.

The most common type of skin malignancy is: A) Squamous cell cancer B) Basal skin cancer C) Melanoma D) Dysplastic nevi

A) Squamous cell cancer Squamous cell cancer is the most frequent type of skin cancer. It can occur in normal skin or infl amed skin. Common areas involved are those which are exposed to the sun or UV light. The earliest form of squamous cell cancer is known as Bowen's disease, in which the cancer has not involved the tissue nearby.

Erysipelas is an infection of the skin most commonly caused by which of the following class of organisms? A) Streptococci B) Staphylococi C) Gram-negative bacteria D) Fungi

A) Streptococci Erysipelas is a skin infection commonly caused by group A betahemolytic streptococci. This infection is usually more superfi cial than other bacterial infections of the skin, such as cellulitis.

A 14-year-old female who is sexually active is brought in by her mother for an immunization update. According to the mother, her daughter had 1 dose of hepatitis B vaccine. Which of the following vaccines would you administer at this visit? A) Td and hepatitis B B) DTaP (diphtheria, tetanus, acellular pertussis) and hepatitis B C) Hepatitis B only D) MMR (measles, mumps, rubella) and Td

A) Td and hepatitis B The CDC recommends Td and hepatitis B for 14-year-old females. It is recommended for her to continue the hepatitis series, which includes a total of 3 injections.

Which of the following clinical fi ndings can mimic a case of testicular torsion but is not considered an emergent condition? A) The "blue dot" sign B) One swollen testicle with yellow-colored penile discharge C) An acute onset of dysuria and frequency D) A varicocele

A) The "blue dot" sign The "blue dot" sign is located underneath the skin of the testicle and appears as a round, blue to purple mass. It is not an emergent condition. Also known as the torsion of testicular appendage.

A middle-aged nurse is having his PPD or Mantoux test result checked. A reddened area of 10.5 mm is present. It is smooth and soft and does not appear to be indurated. During the interview, the patient denies fever, cough, and weight loss. Which of the following is a true statement? A) The PPD result is negative. B) The PPD result is borderline. C) The PPD should be repeated in 2 weeks. D) A chest x-ray and sputum culture are indicated.

A) The PPD result is negative. Best Clues 1) Knowledge that skin induration, not the red color, is the best indicator of a positive reaction. 2) Lack of signs or symptoms of TB. Leik

When evaluating the blood pressure on both the arms and legs of an infant who has a diagnosis of coarctation of the aorta, which of the following is the correct finding? A) The blood pressure is higher in the arms than in the legs B) Only the diastolic blood pressure is higher in the legs than in the arms C) The blood pressure is higher in the legs than the arms D) The blood pressure is lower in both arms than in the legs

A) The blood pressure is higher in the arms than in the legs In coarctation of the aorta, blood pressure is higher in the arms than in the legs due to the narrowing in the aorta. Blood pressure must rise to get adequate blood flow to the lower extremitites; therefore, the blood pressure above the coarctation rises to compensate for this.

RhoGAM's mechanism of action is: A) The destruction of Rh-positive fetal RBCs that are present in the mother's circulatory system B) The destruction of maternal antibodies against Rh-positive fetal RBCs C) The stimulation of maternal antibodies so that there is a decreased risk of hemolysis D) The destruction of maternal antibodies against fetal RBCs

A) The destruction of Rh-positive fetal RBCs that are present in the mother's circulatory system RhoGAM is given to mothers with Rh-negative blood when the fetus has Rh-positive blood. RhoGAM protects the mother from developing antibodies by destroying the Rh-positive fetal RBCs in the mother's blood system.

A positive Coombs test on an Rh-negative pregnant woman means: A) The mother has autoantibodies against Rh-positive red blood cells (RBCs) B) The fetus has autoantibodies against maternal RBCs C) The mother does not have Rh factor against fetal RBCs D) The fetus does not have RBC autoantibodies

A) The mother has autoantibodies against Rh-positive red blood cells (RBCs) The mother's autoantibodies can attack the fetus's Rh-positive red blood cells and cause destruction of these cells, which can cause severe anemia and complications in the fetus. Today this is preventable with the administration of anti-RhD immunoglobulin (Rho(D) immune globulin) to an Rh-negative mother at 28 weeks gestation and after birth if the newborn is Rh positive.

A 92-year-old woman has recently been diagnosed with community-acquired bacterial pneumonia. During the follow-up visit, the son reports that the patient seems to be getting better. Which of the following statements about the patient is not indicative of a serious condition? A) The patient is sleeping less in the daytime and is more alert when awakened (positive/positive). B) The patient complains of double vision and is sleeping more (negative/negative). C) The patient is eating less and she has lost 10% of her previous body weight (negative/negative). D) The patient has become more agitated and confused at night (negative/negative).

A) The patient is sleeping less in the daytime and is more alert when awakened (positive/positive). Best Clues 1) The patient's LOC is "alert when awakened." 2) Both halves of the complex sentence under option A are positive findings. 3) Rule out Options B, C, and D because they are all negative findings. Leik

Which of the following is the correct statement regarding the size of the arterioles and veins on the fundi of the eye? A) The veins are larger than the arterioles B) The arterioles are larger than the veins C) The arterioles are half the size of the veins D) The veins and the arterioles are equal in size

A) The veins are larger than the arterioles

Café-au-lait spots look like tan-to-light brown stains that have irregular borders. They can be located anywhere on the body. Which of the following is a correct statement? A) They are associated with neurofibromatosis or von Recklinghausen's disease B) They may be considered as precancerous after a biopsy C) They are more common in darker-skinned children D) They are associated with Wilson's disease

A) They are associated with neurofibromatosis or von Recklinghausen's disease Café-au-lait spots are caused by an increase in melanin content, often with the presence of giant melanosomes. They have irregular borders and vary in color from light to dark brown. Neurofibromatosis causes tumors to grow in the nervous system, and these tumors commonly cause skin changes that are seen as café-au-lait spots.

A 45-year-old patient complains of a sore throat. Upon examination, the NP notices a bony growth midline at the hard palate of the mouth. The patient denies any changes or pain. It is not red, tender, or swollen. She reports a history of the same growth for many years without any change. Which of the following conditions is most likely? A) Torus palatinus B) Geographic tongue C) Acute glossitis D) Leukoplakia

A) Torus palatinus Best Clues 1) Description of a chronic bony growth located midline in the hard palate. 2) Rule out glossitis, geographic tongue, and hairy leukoplakia because they are all located on the tongue and not on the hard palate (roof of the mouth).

Which of the following drugs can increase the risk of bleeding in patients who are on anticoagulation therapy with warfarin sodium (Coumadin)? A) Trimethoprim/sulfamethoxazole (Bactrim DS) B) Carafate (Sucralfate) C) Losartan (Cozaar) D) Furosemide (Lasix)

A) Trimethoprim/sulfamethoxazole (Bactrim DS)

Jason, an 8-year-old with type 1 diabetes, is being seen for a 3-day history of frequency and nocturia. He denies flank pain and is afebrile. The urinalysis result is negative for blood and nitrites but is positive for a large amount of leukocytes and ketones. He has a trace amount of protein. Which of the following is the best test to order initially? A) Urine for culture and sensitivity B) 24-hour urine for protein and creatinine clearance C) 24-hour urine for microalbumin D) An intravenous pyelogram

A) Urine for culture and sensitivity An 8-year-old male patient with the diagnosis of diabetes has a high risk of urinary tract infections. A large amout of leukocytes in the urinalysis is abnormal and he has been having symptoms of frequency and nocturia for the past 3 days. The urine culture would be ordered because he has a high risk of infection. The urine culture and sensitivity (C&S) is the best evaluation for diagnosing a urinary tract infection.

A 30-year-old primigravida is diagnosed with a possible threatened abortion. The result of the urine pregnancy test is positive. Which of the following statements is true regarding a threatened abortion? A) Vaginal bleeding and cramping are present, but the cervix remains closed B) Vaginal bleeding and cramping are present along with a dilated cervix C) The fetus and placenta are all expelled D) The products of conception and the placenta remain inside the uterus along with a dilated cervix

A) Vaginal bleeding and cramping are present, but the cervix remains closed Threatened abortion is defined as vaginal bleeding and cramping without the presence of cervical dilation.

All of the following are factors important in determining the peak expiratory flow volume except: A) Weight B) Height C) Age D) Gender

A) Weight Peak expiratory flow volume is determined by using height, gender, and age.

A 30 year old female ate at a restaurant and later found out that a worker at the place tested positive for Hep A. She is worried and comes to you for help. What should she do? a. Give her the Hep A vaccine b. Monitor her symptoms c. Give her immunoglobulin

A. What would help YOUR patient the most? She is anxious and worried which is why she came to you with her concerns. Are the potential side effects of giving her the Hepatitis A vaccine worse than not treating at all? You are right, we don't know she was exposed or how long it had been. But you can provide reassurance by giving the vaccine. Forum.

Tourette syndrome is best defined as: a. Characterized by motor tics with at least one phonic tic developed prior age 21 and lasting for at least one year. b. Fast, repetitive, involuntary muscle contractions. c. Acute development of choreiform movements. d. Sudden and brief choreoathetosis or sustained muscular contractions of a limb or torso.

A. + Repetitive, rapid, stereotyped, unwanted muscle contractions that involve discrete muscle groups describes TICS + Sydenham chorea is characterized by an acute onset of choreiform movements + Paroxysmal dyskinesias are sudden-onset, short-duration choreoathetosis or dystonia/ episodes (a sustained muscle contraction of limb or torso, frequently twisting or with abnormal posture) Question pool

The nurse practitioner should suspect child abuse in which of the following scenarios? A. 3 month old with a greenstick fracture of the radius, parent reports the infant rolled off the couch. B. 15 month old with bruises on knees and shins in various stages of healing. C. 4 year old with a buckle fracture of the radius, parent reports she fell while trying to ride her brother's skateboard. D. 7 year old with splash burns on her left shoulder, arm, and hand, parent reports she pulled a pot of soup off the stove.

A. 3 month old with a greenstick fracture of the radius, parent reports the infant rolled off the couch. Any unexplained bruising or fractures on an infant not developmentally mobile should be viewed with concern. A 15 month old with shin bruising is a common finding with accidental injury while learning to walk and run. Buckle fractures are the most common fractures amongst young children and the mechanism of injury matches the diagnosis. Splatter burns are expected with accidental burns in children, burns patterns including stocking or glove burns, buttocks burns, and branding burns from a curling iron or cigarettes should be cause for concern.

You are reviewing the lab results of a 57yo patient that you saw yesterday in clinic for an annual exam. You notice that his ALT is 65 and AST is 195. The NP is aware that this is a common finding in: A. Alcoholic Hepatitis B. Steatosis C. Acute bile duct obstruction D. Hemochromatosis

A. Alcoholic Hepatitis An AST/ALT ratio of >2 is suggestive of alcoholic hepatitis, >3 is strongly indicative. In bile duct obstruction, levels would be >15x normal. Hemochromatosis and steatosis will show mild elevations 5x normal but will be ALT predominant.

A 34-year-old obese male patient arrives in the clinic for his annual wellness exam. He expresses concern that he has a strong family history of HTN and CAD. Which of the following recommendations would not reduce the risks of HTN for this patient? A. Aspirin 81 mg daily B. Weight Reduction C. Exercise D. Smoking Cessation

A. Aspirin 81 mg daily Lifestyle modifications may have modest reduction of blood pressure. Those who have normal or slightly elevated blood pressure with a strong family history should be educated on the lifestyle modifications to reduce their own risks.

1. A social-worker brings in a school-aged adolescent who has a known history of bipolar disorder. Which of the following is TRUE regarding bipolar disorder? A. Bipolar disorder is associated with heightened levels of energy and activity. B. Bipolar disorder is caused by illicit drug use. C. Depressive symptoms are a late finding of bipolar disorder D. Primary care providers should be the first to prescribe treatment for newly diagnosed patients.

A. Bipolar disorder is associated with heightened levels of energy and activity.* Bipolar disorder is associated with heightened levels of energy and activity, irritability, pressured speech, racing thoughts, and impaired judgment (Hay et al., 2016 p. 203). It is not caused by illicit or prescribed drug use. One of the first indications most commonly reported is depressive symptoms. Primary care providers should refer all patients with suspected bipolar disorder to mental health specialist for diagnostic clarification and treatment.

A 45-year-old female presents to the clinic with bilateral ear pain. She states that she has had a small amount of drainage from both ears. She has had no fever. Upon exam, the NP finds both ear canals are red and swollen. The TMs are intact and pearly gray. What is the appropriate treatment for this patient? A. Ciprodex otic drops B. Ciprofloxacin PO C. Amoxicillin PO D. Swim ear otic drops

A. Ciprodex otic drops Otitis externa should be treated with an otic antibiotic solution, such as ciprofloxacin, with or without a corticosteroid (dexamethasone). The steroid will help with the inflammation. Therefore, Ciprodex otic is the most appropriate treatment. + PO cipro is only needed if systemic symptoms are present. + PO amoxicillin is used to treat AOM. + Swim ear otic drops are appropriate for prevention of otitis externa, but not for treatment of infection.

A patient is suspected to have overdosed on opioids. The NP expects all the clinical findings except A. Dilated pupils B. Respiratory depression C. Pulmonary edema D. Peripheral vasodilation

A. Dilated pupils * Overdosage causes respiratory depression, peripheral vasodilation, pinpoint pupils, pulmonary edema, coma, and death.

A 14 year old sexually mature teenager has been diagnosed with cystitis. The patient and her mom has expressed concern of wanting to take an antibiotic for a shorter duration due to the patient having a history of being non-compliant when having to take medicines for longer than five days. When prescribing an antibiotic for the patient, which classification of medicines can be given for three days to treat cystitis for the patient? A. Fluoroquinolone B. Macrolide C. Cephalosporin D. Penicillin

A. Fluoroquinolone The recommended duration of antibiotic choice for uncomplicated cystitis is 7-10 days. However, for sexually mature teenagers with cystitis, fluoroquinolones for three days are effective.

In your pediatric practice you notice a 14-year-old male patient who is slouched in his chair and giggles. When prompted, he admits to using marijuana but no other illegal substances. He states, "I smoke with my friends and it is legal in many states. Nothing bad happens when you smoke marijuana like it does with real drugs." You respond to him "I am not here to judge you, but I want to offer guidance. Marijuana is a real drug and it does have adverse effects. One long term effect that may concern you is... A. Gynecomastia B. Hepatitis C. Tendon rupture D. Paranoid psychosis

A. Gynecomastia*** Long term use of marijuana causes gynecomastia. This is an important to inform patient's in middle adolescence because they are worried about looking normal are intensely self-involved according to the text. Hepatitis is associated with alcohol and intravenous drug abuse. Tendon rupture is a long-term effect of anabolic steroids. Paranoid psychosis is related more to amphetamines, MDMA, and sedative hypnotics but could be misperceived as the correct answer if the reader is not aware that marijuana causes gynecomastia.

A 65-year-old man is in the clinic for a yearly wellness visit. He is without colorectal cancer risk factors and had a colonoscopy done when he was 54 years old. Last year he opted to complete a FIT test because of the difficulty he had with the colonoscopy bowel prep and the FIT result was negative. According to the USPSTF, which of the following colorectal cancer screenings would be an appropriate option to offer today A. He may repeat a FIT test again this year. B. He is up to date and does not need to be screened for colon cancer this year. C. A colonoscopy is his only option since he did the FIT test last year. D. He may choose to do a stool DNA test and repeat in 5 years.

A. He may repeat a FIT test again this year. The USPSTF recommends several options for colorectal cancer screening in men and women between the ages of 50-75. The options include a colonoscopy every 10 years, a yearly fecal immunochemical test (FIT) or guaiac-based fecal occult blood test (gFOBT), a FIT-DNA test every 1- 3 years, a flexible sigmoidoscopy every 5 years, or a CT colonography every 5 years. A yearly FIT test is a good option for someone that has a difficult time with the colonoscopy preparation. If any of the optional test results are positive, a colonoscopy should be performed. Question pool

Mr. Jones is a 74-year-old male coming to see you after a short stay in the hospital for Heart Failure. He has had regular bowel movements and walks with a cane. He is having trouble making it to the bathroom to urinate since he has been home. The most common cause of his incontinence is/are: A. Medications & restricted mobility B. Infection & medications C. Stool impaction & restricted mobility D. Medications

A. Medications & restricted mobility Medications used in heart failure (strong diuretics), and the reliance of a mobility device can lead to transient incontinence. There was no mention of infection, don't assume what you're not told. He has regular bowel movements, so impaction is unlikely and if this was your cause would need to be investigated further. Medications alone may cause the incontinence, however the reliance on a mobility aide further restricts restroom access.

A 53-year-old woman presents to clinic complaining of intermittent vaginal itching and erythema for "months". The patient reports that her symptoms always resolve but return soon after. She reports approximately three episodes in the past 2 months all of which have been treated. The patient has been tested for STI's with negative results in the past and is a monogamous relationship and reports using condoms during intercourse. The patient has taken over-the-counter Monistat, in addition to oral Diflucan. Medical history includes obesity, hypertension, and hypercholesteremia. As her provider what is the best intervention? A. Order a fasting serum glucose level B. Order gonorrhea and Chlamydia swabs C. Refer to obstetrics and gynecology D. Discuss weight loss options with the patient

A. Order a fasting serum glucose level* Women who suffer from chronic candida vulvovaginitis should be suspected for diabetes, thus a fasting serum glucose should be completed. The patient has been evaluated for STIs previously with no prior finding. Referral is not appropriate at this time. Although a discussion of weight loss with the patients is applicable it will not resolve the issues of chronic vulvovaginitis. (Masharani, 2018).

A 77 year old female with history of Hypothyroidism, Hypertension, and Rheumatoid Arthritis presents to the clinic with multiple bullae and blisters over her trunk, and her right elbow flexor. The onset of her symptoms was 6 days ago. She complains of mild itching and some burning over her lesions. She denies fever. She has tried high dose topical corticosteroids and oral antibiotics with no relief. A skin punch biopsy was done and Bullous Pemphigoid was diagnosed. What is the initial treatment of choice? A. Prednisone 0.75mg/kg daily B. High dose topical corticosteroids C. Dapsone 50mg po Daily D. IV IgG

A. Prednisone 0.75mg/kg daily Bullous Pemphigoid is an autoimmune skin disorder that is characterized by large bullae and blisters over flexural surfaces, oral mucosa and skin surface. If the patient has a mild disease then topical steroids may be effective. In this scenario the patient has trialed topical steroids and has not had success. Prednisone at 0.75mg/kg a day is recommended to achieve initial control of the disease. Oral Dapsone is effective when mucous membranes are affected. The use of IV IgG is reserved for refractory cases. This would not be used unless the patient failed oral steroid therapy.

2. A 7-year-old female patient presents to your office today. Her parents report that she has been suffering with severe diarrhea for the past week and has not been wanting to eat or drink very much. Her lab results reveal decreased GFR and an elevated serum creatinine. The Nurse Practitioner correctly identifies her condition as _____ acute kidney injury. A. Prerenal B. Renal C. Postrenal D. Intrinsic renal

A. Prerenal* Acute kidney injury can be prerenal, renal, or postrenal. Common causes of prerenal injury include volume depletion, seen in GI disturbances (nausea/vomiting), loss of volume through the skin, diminished fluid intake, or with hemorrhage. Cardiac failure, nephrotic syndrome, capillary leakage, cirrhosis, a renal arterial thrombosis, or vessel injury can also lower circulating volume and cause prerenal injury. See Table 24-1 Classification of acute kidney injury for further prerenal, renal, and postrenal cause

Which of the following presents the highest risk for HIV transmission and infection: A. Receptive anal intercourse B. Penile-vaginal intercourse C. Oral intercourse D. All present equal transmission and infection rates

A. Receptive anal intercourse MSM account for 67% of all new HIV infections in the United States, despite representing a small proportion of the country's population. The high burden of HIV infection among MSM stems in part from the efficient transmission of the virus through receptive anal intercourse, which confers a higher risk of HIV infection than other sexual activities, such as penile-vaginal and oral intercourse.

A G2P1 26 year old female, 26.2 weeks gestation, presents to the clinic with fatigue and dyspnea. She is late to care, having only one other pre-natal visit about 7 weeks ago. Her last delivery was vaginal and uncomplicated. The NP understands these symptoms are: A. Signs of anemia, and checks the CBC B. Consistent with third trimester, and warrant no further testing. C. Early warning signs of pre-eclampsia, and checks for proteinuria. D. Prescribes bedrest for the last 4 weeks of pregnancy

A. Signs of anemia, and checks the CBC Anemia is common in pregnancy, and usually becomes evident in the third trimester. Supplemental iron is usually given to avoid this complication, but this patient is late to care, and has not been taking any supplements. Anemia must be ruled out. B. Although fatigue and dyspnea are common findings in the 3rd trimester, periodic CBC must be performed to rule out anemia, since this mom is late to care, her symptoms cannot be ignored. C. Early warning or pre-eclampsia is HBP, proteinuria, stomach pain, and blurry vision, not fatigue and dyspnea. D. Bedrest is not warranted, treatment of underlying anemia is the correct plan.

1. A 34 year old woman presents to the clinic complaining of shortness of breath, dizziness, and tremors. She has a one year history of panic attacks that are worsening and occurring at least once a week to where she passes out a work. She has been dealing with financial difficulty for the last month. She is currently taking clonazepam 0.5 mg po bid prn anxiety. Which medication would be the next appropriate therapy to start? A. Venlafaxine XR 37.5 mg po qd B. Clorazepate 15 mg po qhs C. Imipramine 10 mg po qd D. St. John's wort 450 mg po bid

A. Venlafaxine XR 37.5 mg po qd : Clorazepate is considered a benzodiazepine and she is already on this type of medication. Imipramine is a tricyclic antidepressant and should be a second line to SSRI's or SNRI's to treat depression, anxiety, and panic disorders due to extensive potential side effects. St. John's wort has several drug interactions and potential tyramine food interactions. Venlafaxine is considered first line treatment for generalized anxiety disorder and panic disorders.

A 34-year-old female presents to your clinic with a chief complaint of generalized malaise with gradual onset. She has a pale appearance and reports to you that she follows a strict vegetarian diet and is transitioning to becoming vegan. She stopped consuming dairy products 6 months ago. As the NP, you decide to order lab work for further investigation. You include what as a differential diagnosis? A. Vitamin B-12 deficiency B. Folic Acid Deficiency C. Hereditary Spherocytosis D. G6PD Deficiency

A. Vitamin B-12 deficiency* Vitamin B12 deficiency causes a moderate to severe anemia of slow onset. Patients are usually pale and may be mildly icteric or sallow. Since vitamin B12 is present in foods of animal origin, dietary vitamin B12 deficiency is extremely rare but is seen in vegans—strict vegetarians who avoid all dairy products, meat, and fish.

The CDC recommends that all pregnant women receive which two vaccinations during pregnancy: A: Influenza and Tdap B: MMR and Tdap C: Influenza and MMR D: MMR and Varicella

A: Influenza and Tdap * The CDC recommends all pregnant women receive an Influenza vaccine during the "flu season" and all pregnant women should receive a Tdap at 27-36 weeks to protect the infant from pertussis. Live virus vaccinations such as MMR and varicella should be given prior to pregnancy, because they are contraindicated during pregnancy.

What HTN medications are first-line for patients with DM or mild/moderate renal disease?

ACE inhibitors Leik

What are the preferred drugs to treat hypertension in diabetics and patients with mild renal disease because of their renal-protective properties?

ACE inhibitors or ARBs LEIK

Abnormal findings in Hypertensive retinopathy

AV nicking, silver and/or copper wire arterioles Leik

Drugs that need to be tapered after long term use.

Abrupt discontinuation will cause the treated condition to fl are up (exacerbation), rebound, and/or have adverse effects: - Beta-blockers (rebound hypertension or hypertensive crisis). - Benzodiazepines (severe anxiety, insomnia, seizures, tremors). - Oral steroids. - Anticonvulsants (seizures). - Paroxetine or Paxil. - Antiarrythmics (refer to cardiologists). - Antipsychotics and many others.

What constitutes primary amenorrhea?

Absence of menarche by age 15 (with or without secondary sex characteristics).

Precautions for women on accutane

Accutane is a potent teratogen. Reproductive-aged females must use 2 reliable forms of birth control and must have a negative pregnancy test 1 month before, during, and 1 month after Accutane. NOTE: This and all Vitamin A derivatives are teratogenic in animal studies. Also avoid "mega doses" of Vitamin A in pregnancy. Leik

Initial choice for alzheimer's disease treatment for a patient with dysphagia?

Acetylcholinesterase inhibitors including Aricept or Exelon. The patch form would be the best choice because of the dysphagia. Forum

Treatment of shingles

Acyclovir (Zovirax) 5 × per day or valacyclovir (Valtrex) BID × 10 days for initial breakouts and 7 days for flare-ups. Most effective when started within 48 to 72 hours of when rash appears. Leik

USPSTF recommendations for colorectal screening?

Adults aged 50 to 75 years ■ High-sensitivity fecal occult blood test (FOBT) × 3 consecutive stool samples annually. -- Guiac FOBT - test for heme, so there are pre-testing dietary restrictions (no red meat) -- FIT - tests for antibodies, so no dietary restrictions are usually necessary ■ Flexible sigmoidoscopy every 5 years. ■ Colonoscopy every 10 years (unless polyps are removed, then every 3 years).

Patients with WHAT KIND of history are at higher risk for aspirin and NSAID allergies?

An atopic history (asthma, eczema, allergic rhinitis) with nasal polyps Leik

What is the precursor lesion of squamous cell carcinoma: 1. Basal cell carcinoma 2. Actinic keratosis 3. Melanoma 4. Eczema

Answer is option 2. Actinic keratosis is the precursor lesion of squamous cell cancer.

Hep B Core Antibody (anti-HBc)

Appears at the onset of symptoms in acute hepatitis B and persist for life. The presence of anti-HBc indicates previous or ongoing infection with hepatitis B virus in an undefined time frame CDC

GLP-1 medications are approved for use with ________ insulin, but not with _______ insulin.

Approved for use with basal insulin NOT approved for use with rapid/prandial

USPSTF recommendations for lung cancer screening?

Asymptomatic adults aged 55 to 80 years who have a 30 pack-year smoking history and currently smoke or have quit smoking within the past 15 years. Screen annually for lung cancer with low-dose computed tomography. Discontinue screening when the patient has not smoked for 15 years. Grade: B

What is AV nicking?

At sites where the thickened arterioles cross the veins, the veins appear kinked or crimped. Caused by chronic HTN, and is sign of likely TOD elsewhere in body (e.g. heart, brain, kidneys).

Foods to avoid if on anticoagulation therapy

Avoid or limit high intake of vitamin K foods such as green leafy vegetables (kale/collard greens, spinach, cabbage), broccoli. Vitamin K decreases the effectiveness of warfarin sodium. Avoid excessive intake of vitamin K-rich foods. Mayonnaise, canola oil, and soybean oil also have high levels of vitamin K

A 47-year-old female with a history of HTN, PCOS, and OA of the left knee presents for her annual physical exam. The FNP reviews fasting labs obtained prior to this visit and sees K+ value of 5.4 mEq/L. An EKG reveals no peaked T waves, widened QRS, or other ectopy, and is normal compared to a baseline EKG obtained at age 40. The patient states her OBGYN recently started her on a new medication. What medication is most likely contributing to the patient's hyperkalemia? a. Metformin b. Spironolactone c. Fluconazole d. Metronidazole

B Spironolactone is often used in PCOS to combat hirsutism and adult acne and is a drug that can inhibit excretion of potassium, causing serum levels to increase. Metformin, fluconazole, and metronidazole do not inhibit potassium excretion. Question pool

Mother brings child to clinic for 12-month old vaccinations. She is concerned about getting the vaccinations today because he has had a fever of 100 - 100.4 with a little diarrhea for the past 3 days. What is the best action for the NP? A. Reschedule visit after fever and diarrhea subsides. B. Educate mother that minor acute illness are not contraindicated and proceed with the vaccinations. C. Treat the current illness with a stool culture and amoxicillin, then reschedule immunizations. D. Educate mother and give all vaccinations except for MMR and Varicella because they both contain live virus

B Contraindications to vaccinations would include a moderate to severe illness but not low-grade temp with diarrhea.

You recently treated a 3-year-old female with high-dose amoxicillin for AOM. She presents with her parents for f/u. Symptoms have improved, but parents are concerned about recurrent infections. They want to know if there is any way to eliminate the causes of AOMs. While educating her parents, you explain that all the follow are predisposing factors for AOM except: a. smoke exposure b. Swimming c. Daycare attendance d. Bottle Feeding

B - Swimming is a predisposing factor for otitis externa (swimmer's ear). Predisposing factors of AOM include: eustachian tube dysfunction, bacterial colonization, viral URIs, smoke exposure, impaired immune system, bottle feeding, season, day care attendance, genetics, and age.

A 24 yr old female with a history of migraines, comes into the office today for a new prescription for migraines. She is currently 18 weeks pregnant with her first child. The nurse practitioner would choose the preferred medication which is: A. Carfegot B. Acetaminophen C. Sumatriptan D. Valproic Acid

B - acetaminophen The preferred abortive medication during pregnancy include acetaminophen, acetaminophen and codeine or other narcotics, and magnesium. The following are more effective abortive medications but are NOT PREFERRED during pregnancy: butalbital, isometheptene, caffeine, aspirin, naproxen, ibuprofen, and triptan drugs) Rationale: + Ergotamine-containing preparations (cafergot) should be avoided during pregnancy. + Sumatriptan: Triptans should be avoided in women who are pregnant +Valproic Acid or divalproate should be avoided in pregnancy Question pool

A 10 year-old boy presents to the clinic with his mother. The chief complain is abrupt onset of abdominal pain. Physical examination localize the pain over the right iliac area. Pain is present with the movement of the right testicle. Mom indicates that the pain is so severe that it made him vomit last night. A Urinary tract infection B Acute appendicitis C Testicular torsion D Peptic ulcer

B Acute appendicitis Typically, patients with acute appendicitis have a periumbilical pain that localizes in the right lower quadrant. During acute appendicitis, vomiting usually follows the onset of pain.

A nurse practitioner working in a school health clinic is evaluating a new student who has been referred to him by a teacher. David is a 16-year-old male with a history of attention deficit disorder (ADD). He complains that his parents are always fighting and he thinks that they are getting divorced. During the interview, he is staring at the floor and avoiding eye contact. He reports that he is having problems falling asleep at night and has stopped seeing friends, including his girlfriend. Which of the following statements is the best choice to ask this teen? A) "Do you want me to call your parents after we talk?" B) "Do you have any plans of killing yourself or hurting other people?" C) "Do you have any close male or female friends?" D) "Do you want to wait to tell me about your plans until you feel better?"

B) "Do you have any plans of killing yourself or hurting other people?" Best Clues 1) Classic behavioral cues (avoidance of eye contact, insomnia, social isolation) 2) Parents fighting and getting divorced Leik

A nursing home resident reports to his physician that his previous roommate was recently started on tuberculosis treatment. A Mantoux test and chest x-ray are ordered for the patient. What is the minimum size of induration considered positive for this patient? A) 3 mm B) 5 mm C) 10 mm D) 15 mm

B) 5 mm To perform the Mantoux test, a dose of 5 tuberculin units in 0.1 mL solution is injected intradermally into the forearm and read 48 to 72 hours later. Results are measured by the size of the induration, not erythema. Standard results of 10 mm or less are considered negative for a lowrisk population. For a high-risk population, such as a recent exposure, 5 mm of induration or greater is a positive result.

All of the following children are within the parameters of normal growth and development for their age group except: A) A 2-month-old who coos and smiles B) A 14-month-old who understands complex commands C) A 20-month-old who can walk without support D) A 3-year-old who can speak in three- to four-word sentences

B) A 14-month-old who understands complex commands A 14-month-old child should developmentally be able to say "mama" and "dada," know his own name, and know at least 2-4 words. A 2-year-old is able to understand simple commands.

Which of the following individuals is least likely to have an alcohol abuse problem? A) A housewife who gets annoyed if her best friend talks to her about her drinking habit. B) A carpenter who drinks one can of beer nightly when playing cards with friends. C) A nurse who feels shaky when she wakes up, which is relieved by drinking wine. D) A college student who tells his friend that he drinks only on weekends but feels that he should be drinking less.

B) A carpenter who drinks one can of beer nightly when playing cards with friends. A, C and D are positive answers to CAGE questionnaire. Leik

The ELISA and Western Blot tests are both used to test for HIV. Which of the following statements is correct? A) It is a test to detect viral RNA B) A positive ELISA screening does not mean the person has HIV infection C) It is a test to detect viruses D) It is a diagnostic test for the AIDS virus

B) A positive ELISA screening does not mean the person has HIV infection The ELISA test is always followed by a Western blot test to confirm diagnosis.

While performing a routine physical exam on an older White male with a history of cigarette smoking, the NP palpates a pulsatile mass in the patient's midabdominal area. A bruit is auscultated over the soft mass. Which of the following is the recommended imaging method? A) CT scan B) Abdominal ultrasound C) Radiography of the chest D) Plain film of the abdomen

B) Abdominal ultrasound Best Clues 1) Pulsatile mass located in the middle of the abdomen that is associated with a bruit 2) Patient demographics Notes 1) This question describes the classic case of an abdominal aortic aneurysm (AAA). 2) Risk factors for AAA are male gender, elderly, smoker, hypertension, White race, and family history. 3) Signs and symptoms of AAA rupture are abrupt onset of severe abdominal pain with low back pain and abdominal distension with signs and symptoms of shock. 4) The initial imaging diagnostic test to order is the abdominal ultrasound. Leik

You are following up a 65-year-old male who has been on a new prescription of fluvastatin (Lescol) for 6 weeks. During a follow-up visit, he reports feeling extremely fatigued and having dark-colored urine. He denies any generalized muscle soreness. Which of the following diagnoses is most likely in this patient? A) Rhabdomyolysis B) Acute drug-induced hepatitis C) Acute mononucleosis D) A major depressive episode

B) Acute drug-induced hepatitis Liver enzymes, such as ALT and AST, can be elevated with the use of medications, such as statin medications. High liver enzymes can cause an acute drug-induced hepatitis.

A 13-year-old girl complains of an irregular menstrual cycle. She started menarche 6 months ago. Her last menstrual period was 2 months ago. She denies being sexually active. Her urine pregnancy test is negative. Which of the following would you advise the child's mother? A) Consult with a pediatric endocrinologist to rule out problems with the hypothalamus, pituitary, and adrenal (HPA) axis. B) Advise the mother that irregular menstrual cycles are common during the first year after menarche. C) Advise the mother that her child is starting menarche early and has precocious puberty. D) Ask the medical assistant to get labs drawn for TSH, follicle stimulating hormone (FSH), and estradiol levels.

B) Advise the mother that irregular menstrual cycles are common during the first year after menarche. Notes 1) This question describes normal growth and development in adolescents. 2) When girls start menarche, their periods may be very irregular for several months up to 2 years. Leik

Precocious puberty is defined as the onset of secondary sexual characteristics before the age of: A) Age 7 in girls and age 8 in boys B) Age 8 in girls and age 9 in boys C) Age 9 in girls and age 10 in boys D) Age 9 for both girls and boys

B) Age 8 in girls and age 9 in boys

The "gold standard" for the diagnosis of active Helicobacter pylori infection of the stomach or duodenum is: A) A Helicobacter pylori titer B) An endoscopy with tissue biopsy C) An upper GI series D) A urea breath test

B) An endoscopy with tissue biopsy The "gold standard" for diagnosing Helicobacter pylori infection is to perform tissue biopsy via endoscopy.

An 18-year-old female presents in the college health clinic complaining of a strong odor in her vagina. She reports that she had an abortion about 3 weeks ago and recently completed her prescription of antibiotics. The NP performs a vaginal speculum exam and notes a large amount of grayish to off-white discharge coating the patient's vaginal walls. It has a milk-like consistency. During microscopy, the slide reveals mature squamous epithelial cells with numerous bacteria noted on the cell borders. The vaginal pH is at 6.0. Which of the following conditions is most likely? A) Trichomoniasis B) Bacterial vaginosis C) Candida vulvovaginitis D) Hormonal changes

B) Bacterial vaginosis Best Clues 1) The vaginal pH is alkaline (pH of 6.0). 2) Rule out Candida because it is classified as a yeast organism (not a bacteria). 3) Rule out Trichomonas because it is a protozoa or unicellular flagellated organism. 4) The odor and discharge are not due to hormonal changes in an 18-year-old female. Notes 1) Bacterial vaginosis (BV) has an alkaline pH (vagina normally has an acidic pH of 4.0). BV is the only vaginal condition with an alkaline pH for the exam. 2) BV is not considered an STD (it is caused by an imbalance of vaginal bacteria). The sex partner does not need to be treated. It is a vaginosis (not a vaginitis). 3) BV does not cause inflammation (the vulvovagina will not be red or irritated). The microscopy slide will have very few WBCs and a large number of clue cells. 4) The vaginal discharge in Candida infection is of a white color with a thick and curd-like consistency. It frequently causes redness and itching in the vulvovagina due to infl ammation. 5) The microscopy in candidiasis will show a large number of WBCs, pseudohyphae, and spores ("spaghetti and meatballs"). 6) Candida yeast is normal flora of the gastrointestinal (GI) tract and in some women's vaginas. 7) Trichomonas infection (or trichomoniasis) vaginal discharge is copious, bubbly, and green in color. It causes a lot of inflammation resulting in itching and redness of the vulvovagina. It is considered a sexually transmitted infection. The sex partner also needs treatment. 8) Microscopy is the gold standard of diagnosis for BV, candida vaginitis, and trichomoniasis for the exam. Leik

Cathy a 27-year-old patient comes in to your office for her first prenatal visit. While educating her on the Tdap vaccine, you tell her that ideally the vaccine should be given: A) 6 weeks after delivery B) Between 27-36 weeks C) In the first trimester D) Between 35- 40 weeks

B) Between 27-36 weeks The CDC recommends all pregnant women be given the Tdap vaccine during each pregnancy regardless of her previous vaccine history. The optimal timing is between 27 and 36 weeks to maximize the antibody response of the patient and passive antibody transfer to the infant

Which cranial nerve innervates the extraocular muscles of the eyes? A) CN II, CN III, and CN VI B) CN III, CN IV, and CN VI C) CN IV, CN V, and CN VII D) CN V, CN VI, and CN VIII

B) CN III, CN IV, and CN VI

he mother of a 13-year-old male with Down syndrome is in the family nurse practitioner's office and wants a sports physical done for her son. She reports that he wants to join the football team in his school. You would tell the mother that her son: A) Can play a regular football game as long as he wears maximum protective football gear B) Cannot play some contact sports because of an increased risk of cervical spine injury C) Can play certain contact sports after he has been checked for cervical instability D) None of the above

B) Cannot play some contact sports because of an increased risk of cervical spine injury Children with the diagnosis of Down syndrome are at higher risk for atlantoaxial instability, congenital heart defects, and early onset of Alzheimer's disease. Therefore, due to the risk of cervical spine injury, some contact sports are not advised.

Which of the following is the most common cause of nongonococcal urethritis? A) Escherichia coli B) Chlamydia trachomatis C) Neisseria gonorrhoeae D) Mycoplasma

B) Chlamydia trachomatis

Fitz-Hugh-Curtis syndrome is associated with which following infection? A) Syphilis B) Chlamydia trachomatis C) Herpes genitalis D) Lymphogranuloma venereum

B) Chlamydia trachomatis Fitz-Hugh-Curtis is a complication of having pelvic infl ammatory disease that was caused by a vaginal infection, such as gonorrhea or Chlamydia trachomatis. This causes inflammation and infection in the pelvic cavity. Left untreated, this infection can cause adhesions that stretch from the peritoneum to the liver.

There is a higher risk of balanitis in which of the following conditions? A) Renal insufficiency B) Diabetes mellitus C) Graves' disease D) Asthma

B) Diabetes mellitus Balanitis is an yeast infection of the glans of the penis. Men who are not circumcised and who have diabetes mellitus are at higher risk for developing balanitis.

When starting an elderly patient on a new prescription of levothyroxine (Synthroid), the nurse practitioner should keep in mind that the rationale for starting an elderly patient on a lower dose is which of the following? A) Due to its central nervous system effects B) Due to its cardiac effects C) Due to its renal effects D) Due to its hepatic effects

B) Due to its cardiac effects Levothyroxine (Synthroid) should be started on the lowest dose in elderly patients due to the severe side effects that can occur. Side effects include palpitations, tachycardia, anxiety, irritability, elevated BP, flushing, and insomnia.

Which of the following findings is most likely in young primigravidas with pregnancy-induced hypertension? A) Abdominal cramping and constipation B) Edema of the face and the upper extremities C) Shortness of breath D) Dysuria and frequency

B) Edema of the face and the upper extremities Common signs and symptoms of pregnancy-induced hypertension include edema of the face and the upper extremities, weight gain, blurred vision, elevated BP, proteinuria, and headaches.

A 53-year-old crossing guard complains of twisting his right knee while working that morning. The knee is swollen and tender to palpation. The nurse practitioner diagnoses a Grade II sprain. The initial treatment plan includes which of the following? A) Application of cold packs the first 24 hours followed by applications of low heat at bedtime B) Elevation of the affected limb and intermittent applications of cold packs for the next 48 hours C) Rechecking the knee in 24 hours and isometric exercises D) The application of an Ace bandage to the affected knee

B) Elevation of the affected limb and intermittent applications of cold packs for the next 48 hours Elevation of the injured knee above the heart will reduce the amout of swelling that can occur. Use of ice packs immediately after the injury is most effective and will reduce swelling in the tissue. Ice the affected area for 15 minutes at a time intermittently to prevent frostbite and further damage to tissue. Allowing 30-45 minutes between icings of the limb is recommended.

A 57-year-old Hispanic female is seeing you today at the local free clinic. Over the last 2 weeks, she has noticed a change in her stools which were previously light brown, formed, and occurred daily. Now she experiences a bowel movement maybe every 4 days, consisting of what she describes as "a soft, limp straw," and now they are much darker in color. Thinking back, her appetite is not what it used to be and she has lost approx. 20 lbs in the last 6 mths without a change in diet or exercise. She takes no medications regularly. You perform a thorough H&P. VS are stable. BMI is 38. Physical examination today includes a digital rectal exam which is negative, and there is no evidence of fecal impaction. Having limited resources, which of the following would be ordered in your initial evaluation of her condition? A) LFTs B) FIT testing C) CA 125 D) Flexible sigmoidoscopy

B) FIT testing FIT testing requires only one stool sample and does not necessitate fasting, thus is a convenient, sensitive, and relatively economical test used to screen for colorectal CA. A positive FIT test should prompt a referral to Gastroenterology for further evaluation.

A newborn infant who is small for gestational age is noted to have shortened palpebral fi ssures and microcephaly with a small jaw. This infant is most likely to be diagnosed with: A) Down syndrome B) Fetal alcohol syndrome C) Growth retardation D) Hydrocephalus

B) Fetal alcohol syndrome Classic symptoms of fetal alcohol syndrome include small palpebral fissures and microcephaly with a small jaw.

A newborn's mother is discovered to be HBsAg (hepatitis B surface antigen) positive. Which of the following would you recommend for this infant? A) Give the baby hepatitis B immunoglobulin B) Give the baby both hepatitis B vaccine and hepatitis B immunoglobulin C) Give the baby hepatitis B vaccine only D) Send the baby home because he is not infected

B) Give the baby both hepatitis B vaccine and hepatitis B immunoglobulin For a mother who tests positive for HBsAg, the newborn infant should be given hepatitis B vaccine and hepatitis B immunoglobulin for protection.

A 65-year-old female smoker presents with a history of Barrett's esophagus, and gastroesophageal reflux disease (GERD). The patient reports that her gastroenterologist's prescription for esomeprazole (Nexium) 40 mg daily ran out a few days ago. She is complaining of severe heartburn and a sore throat. During the physical exam, the NP notes an erythematous posterior pharynx without tonsillar discharge and mild dental enamel loss on the rear molars. What is the best initial action for the NP to follow? A) Refer the patient to an oncologist for a biopsy to rule out esophageal cancer. B) Give the patient a refill of her proton pump inhibitor (PPI) prescription. C) Recommend that the patient take an OTC ranitidine (Zantac) twice a day until she can be seen by her gastroenterologist. D) Switch the patient's prescription to another brand of PPI because her symptoms are not getting better.

B) Give the patient a refill of her PPI prescription and advise her to schedule an appointment with her gastroenterologist. CLUES 1) Rule out option A because the patient is already under the care of a gastroenterologist. 2) OTC ranitidine (Zantac) is not potent enough to control the symptoms of erosive esophagitis. A PPI is the preferred treatment for erosive esophagitis. 3) Do not switch the patient to another PPI brand. Her worsening symptoms are caused by rebound. 4) The best initial action in this case is to refill the PPI prescription because the patient is fully symptomatic (erosive esophagitis) until she can see her gastroenterologist. NOTES: 1) The patient's severe symptoms are caused by the sudden discontinuation of the high-dose PPI (rebound-type of reaction). 2) Barrett's esophagus is the "precancerous" lesion of esophageal cancer. It is best managed by gastroenterologists (not oncologists). 3) Patients diagnosed with Barrett's esophagus typically have endoscopic examinations with biopsy by a gastroenterologist annually (or every 6 months for high-grade lesions). 4) Patients with Barrett's esophagus are treated with high-dose PPIs for a "lifetime." 5) The first-line treatment of mild, uncomplicated GERD is lifestyle changes (avoid eating 3 to 4 hours before bedtime, dietary changes, weight loss if overweight, etc.). 6) If a patient is at high risk for esophageal cancer (aged 50 years or older, smoker, chronic GERD for decades), consider referral to a gastroenterologist for an upper endoscopy. Leik

A 63-year-old patient with a 10-year history of poorly controlled hypertension presents with a cluster of physical exam findings. Which of the following indicate target organ damage commonly seen in hypertensive patients? A) Pedal edema, hepatomegaly, and enlarged kidneys B) Hepatomegaly, AV nicking, bibasilar crackles C) Renal infection, S3, neuromuscular abnormalities D) Glaucoma, jugular vein atrophy, heart failure

B) Hepatomegaly, AV nicking, bibasilar crackles With long-term, uncontrolled hypertension, organ damage may occur. Organs commonly affected include: brain (stroke), eyes (retinopathy, AV nicking, bleeding, blindness), heart (heart disease, left ventricular hypertrophy, MI, and/or CHF), and kidneys (renal failure, proteinuria).

The mother of a 16-year-old boy is concerned that her son is not developing normally. On physical exam, the patient is noted to have small testes with no pubic or facial hair. What is the most appropriate statement to the mother? A) Her son is developing normally B) Her son's physical development is delayed and should be evaluated by a pediatric endocrinologist C) Her son should be rechecked in 3 months; if he still does not have secondary sexual characteristics, a thorough hormonal workup should be initiated D) Her son's physiological development is slower than normal but is within the lower limit of normal for his age group

B) Her son's physical development is delayed and should be evaluated by a pediatric endocrinologist A 16-year-old male without secondary sexual characteristics should be referred to an endocrinologist. If there is no testicular development by 14 years of age, referral to an endocrinologist is recommended.

A 35-year-old primigravida who is at 28 weeks of gestation is expecting twins. What would you would expect her alpha fetoprotein (AFP) values to be? A) Normal B) Higher than normal C) Lower than normal D) None of the above

B) Higher than normal Alpha fetoprotein is produced in the fetal and maternal liver. Higher levels of alpha fetoprotein are commonly seen in multiple gestations due to the growing fetuses and enlargement of the livers.

1) Which of the following is not true regarding Integrative Medicine? A) Integrative medicine is the integration of complementary therapies (e.g. mind-body therapies, botanical medicines, supplements, acupuncture) with conventional medicine. B) Integrative medicine does not consider the clinician's relationship with the patient as the central therapeutic element. C) Integrative Medicine chooses the safest therapies first. D) Integrative medicine is evidence-based and patient-centered and most of its foundational tenets are shared by conventional medicine.

B) Integrative medicine does not consider the clinician's relationship with the patient as the central therapeutic element. Integrative medicine *does* consider the clinician's relationship with the patient as the central therapeutic element. Options A, C, and D are true statements about Integrative Medicine.

A 72-year-old woman has been on hydrochlorothiazide 12.5 mg for many years to control her Stage II hypertension. Her blood pressure (BP) at this visit is 168/96. She is currently complaining of pain on her right hip and on both knees. She has increased her dose of ibuprofen (Motrin) from 400 mg 3 times day (TID) to 800 mg TID. She is still in pain and would like something stronger. Which of the following statements is the best explanation of the effects of ibuprofen (Motrin) on her disease? A) It increases the chances of adverse effects to her health B) It inhibits the effect of renal prostaglandins and blunts the effectiveness of the diuretic C) It prolongs the therapeutic effects of hydrochlorothiazide and other diuretics D) None of the statements are true

B) It inhibits the effect of renal prostaglandins and blunts the effectiveness of the diuretic NSAIDs and ASA inhibit the vasodilatory effects of prostaglandins, which predisposes the kidney to ischemia. NSAIDs and diuretics can cause acute prerenal failure by decreasing renal blood flow.

A pelvic exam on a woman who is 12 weeks pregnant would reveal that her uterus is located at which of the following areas? A) Between the umbilicus and the suprapubic bone B) Just rising above the suprapubic bone C) Between the suprapubic bone and the xiphoid process D) Between the umbilicus and the xiphoid process

B) Just rising above the suprapubic bone At 12 weeks gestation, the uterus measures approximately the size of a grapefruit, which would be felt just above the suprapubic bone on bimanual exam.

The sentinel nodes (Virchow's nodes) are found at the: A) Right axillary area B) Left supraclavicular area C) Posterior cervical chain D) Submandibular chain

B) Left supraclavicular area The sentinel nodes are found at the supraclavicular area of the chest. They are the first lymph nodes that a cancer lesion will drain into. Therefore, when cancer is diagnosed, these nodes are biopsied to see if the cancer has spread into the lymph system.

A kindergarten teacher is diagnosed with acute pharyngitis. On exam, the throat is a bright red color with no tonsillar exudate, and clear mucus is seen on the lower nasal turbinates. The result of her urinalysis shows a large amount of white blood cells and is positive for nitrites. The patient has a sulfa allergy and thinks she is also allergic to penicillins. Which of the following is the best treatment choice? A) Amoxicillin/clavulanic acid (Augmentin) 500 mg PO BID B) Levoquinolone (Levaquin) 250 mg PO daily C) Trimethoprim sulfamethoxazole (Bactrim DS) 1 tablet PO BID D) Clarithromycin (Biaxin) 500 mg PO BID

B) Levoquinolone (Levaquin) 250 mg PO daily Pharyngitis and urinary tract infection are both covered by using Levaquin, which is a quinolone. Augmentin and Bactrim could not be used due to her allergies.

Which of the following is indicated for initial treatment of an uncomplicated case of Helicobacter pylori negative peptic ulcer disease? A) Omeprazole (Prilosec) B) Misoprostol (Cytotec) C) Ranitidine (Zantac) D) Pepto-Bismol tablets

B) Misoprostol (Cytotec) Misoprostol is recommended for short-term, uncomplicated PUD; it acts by decreasing gastric acid production and enhancing mucosal resistance to injury.

A 28-year-old student is seen in the school health clinic with complaints of a hacking cough that is productive of small amounts of sputum and a runny nose. He does not take any medications, denies any allergies, and has no significant medical history. Physical examination reveals a low-grade temperature of 99.9 degrees Fahrenheit, respirations of 16/min, a pulse of 90 beats per minute, and diffuse fine crackles in the base of the lungs. A chest radiograph (x-ray) shows diffuse infiltrates on the lower lobe of the right lung. The total white blood cell count is 10,500/uL. What is the most likely diagnosis? A) Streptococcal pneumonia B) Mycoplasma pneumonia C) Acute bronchitis D) Legionnaires disease

B) Mycoplasma pneumonia Mycoplasma pneumonia is the organism most commonly seen in children and young adults. It is easily spread from droplets, from sneezing and coughing, in close proximity. Diagnosis is based on symptoms and x-ray results of infiltrates in lower lobes.

A lab technician has a 10.5-mm area of redness and induration in his left forearm after getting a Mantoux test 72 hours ago. The last test, which was done 12 months ago, was negative. He denies cough, night sweats, and weight loss. What is the next best intervention? A) Obtain a sputum culture and a chest x-ray B) Obtain a chest x-ray C) Obtain a sputum for C&S and an acid fast stain D) Obtain a CBC and chest x-ray

B) Obtain a chest x-ray Mantoux results are negative if results show less than 10 mm induration on the forearm for low-risk clients. If results are greater or equal to 10 mm, then a chest x-ray should be ordered for further screening.

A 65-year-old Hispanic woman has a history of type 2 diabetes. A routine urinalysis reveals a few epithelial cells and is negative for leukocytes, nitrites, and protein. Which of the following would you recommend next? A) Order a urine for C&S B) Order a 24-hour urine for microalbumin C) Because it is negative, no further tests are necessary D) Recommend a screening IVP (intravenous pyelogram)

B) Order a 24-hour urine for microalbumin Epithelial cells are cells that are present on the inside lining of the organs. Few epithelial cells can be normal; however, with her history of type 2 diabetes, a 24-hour urine for microalbumin should be ordered to assess kidney function.

A patient is positive for anti-HCV (hepatitis C virus antibody). What is the next step to further evaluate this patient? A) Refer the patient to a gastroenterologist B) Order a hepatitis C PCR (polymerase chain reaction) test C) Order a hepatitis B comprehensive panel D) The patient is immune to hepatitis C and no further testing is indicated

B) Order a hepatitis C PCR (polymerase chain reaction) test When a patient tests positive for anti-HCV (hepatitis C virus antibody), labs ordered should include hepatitis C PCR to further evaluate the patient.

An elderly woman has been on digoxin (Lanoxin) for 10 years. Her electrocardiograph (EKG) is showing a new onset of atrial fibrillation. Her pulse is 64/min. She denies syncope and dizziness. Which of the following interventions is most appropriate? A) Order an electrolyte panel and a digoxin level B) Order a serum thyroid-stimulating hormone (TSH), digoxin level, and an electrolyte panel C) Order a serum digoxin level and decrease her digoxin dose by half while waiting for results D) Discontinue the digoxin and order another 12-lead EKG

B) Order a serum thyroid-stimulating hormone (TSH), digoxin level, and an electrolyte panel Obtaining baseline blood work to evaluate for causes of new onset atrial fibrillation is recommended prior to decreasing or stopping medications. *Thyroid disease is a common cause of new-onset atrial fibrillation.*

A 9-year-old Asian male with no past medical history is brought to clinic by his mother for abnormal behavior at night during sleep. The mother states in the first hour while child sleeps he sweats, screams and sleep walks. Upon waking in the morning, the child has no recollection of the events. Mother states this has occurred 2-3x/week for the past month. The NP diagnoses the patient with what before referring him for a sleep study? A) Nightmares B) Parasomnias C) Psychogenic non-epileptic seizures D) Benign nocturnal myoclonus

B) Parasomnias Nightmares often occur in the early morning hours and can be remembered the next morning. Parasomnias or night terrors occur within 30-90 minutes of falling asleep where the patient has dilated pupils, diaphoresis, sleep walking/talking, sits up in bed, and/or screams for a short amount of time. Psychogenic non-epileptic seizures involve thrashing, pelvic thrusts, bizarre jerking and combativeness in a patient with an audience. Benign nocturnal myoclonus involves focal jerks that occur intermittently throughout the sleep cycle

Hypovolemic shock would most likely occur with fractures of the: A) Spine B) Pelvis C) Femur D) Humerus

B) Pelvis Hypovolemic shock can occur from a fractured pelvis secondary to internal bleeding from a fractured bone fragment that lacerates an artery or vein. The pelvis can accommodate a large amount of fluid.

A 38-year-old multigravida who is at 32 weeks of gestation calls the family nurse practitioner complaining of bright red vaginal bleeding. There is no watery discharge. She complains that her uterus feels hard and is very painful. Which of the following conditions is most likely? A) Placenta previa B) Placenta abruptio C) A molar pregnancy D) An ectopic pregnancy

B) Placenta abruptio + Abruptio placenta symptoms are bright red vaginal bleeding, board-like uterus on palpation, and pain. However, there can be concealed hemorrhage and the patient may not have vaginal bleeding. + Placenta previa is painless bleeding. + Ectopic and molar pregnancy would not progress to 32 weeks gestation.

A 28-year-old multipara who is at 32 weeks of gestation presents to your office complaining of a sudden onset of small amounts of bright red vaginal bleeding. She has had several episodes and appears anxious. On exam, her uterus is soft to palpation. Which of the following is most likely? A) Placenta abruptio B) Placenta previa C) Acute cervicitis D) Molar pregnancy (hydatidiform mole)

B) Placenta previa Placenta previa occurs when abnormal implantation of the placenta occurs. A common symptom of placenta previa is painless, bright red bleeding.

You note the following result on a routine urinalysis of a 37-year-old primigravida who is at 30 weeks of gestation. Leukocyte = trace, nitrite = negative, protein = 2 +, blood = negative. Her weight has increased by 5 lbs during the past week. Which of the following is most likely? A) HELLP syndrome B) Pregnancy-induced hypertension (preeclampsia) C) Eclampsia of pregnancy D) Primary hypertension

B) Pregnancy-induced hypertension (preeclampsia) The classic triad of symptoms of preeclampsia includes hypertension, edema (weight gain), and proteinuria.

A 15 year old boy is seen in the clinic with recurrent depression despite trying cognitive behavioral therapy and intervention modifications at home and school. He denies suicidal ideation but reports he doesn't want to get out of bed. What is the next appropriate plan? A) Advise the boy and his family that this is a normal finding in adolescents. B) Prescribe medication such as a SSRI and educate this will take 4-6 weeks for the optimal effects. C) Refer patient to a Psychiatrist for further management. D) Prescribe a stimulant medication (controlled class II) to improve the boys energy level.

B) Prescribe medication such as a SSRI and educate this will take 4-6 weeks for the optimal effects. * The next appropriate step would be to prescribe a medication such as a SSRI to help with the patient's depression while he is continuing cognitive behavioral therapy.

Jane, a young primigravida, reports to you that she is starting to feel the baby's movements in her uterus. This is considered to be which of the following? A) Presumptive sign B) Probable sign C) Positive sign D) Possible sign

B) Probable sign Probable signs of pregnancy are symptoms of pregnancy that are felt by the woman.

Hegar's sign is considered a: A) Positive sign of pregnancy B) Probable sign of pregnancy C) Presumptive sign of pregnancy D) Problem in pregnancy

B) Probable sign of pregnancy Hegar's sign is softening of the lower portion of the uterus and is considered a probable sign of pregnancy.

You note, during a physical exam on a 6-year-old child, some pitting on the finger nails. This finding is correlated with: A) Iron-defi ciency anemia B) Psoriasis C) Onychomycosis D) Vitamin C deficiency

B) Psoriasis Fingernail pitting is correlated with psoriasis. Psoriasis can cause pitting on all finger and toenails, along with thickening and irregular shape of the nail.

During a routine physical exam of an elderly woman, a triangular thickening of the bulbar conjunctiva on the *temporal* side is noted to be encroaching on the cornea. She denies any eye pain or visual changes. Which of the following is most likely? A) Corneal arcus B) Pterygium C) Pinguecula D) Chalazion

B) Pterygium Pterygium is a triangular growth on the white part of the eye that also extends onto the cornea. Corneal arcus is a ring around the edge of the cornea. Chalazion is a stye in the eye that may cause pain and swelling. Pinguecula is a benign growth on the conjuctiva caused by the degeneration of its collagen fibers. Thick, yellow fibers may be seen.

Which of the following is the best location to auscultate for the S3 heart sound? A) Aortic area B) Pulmonic area C) Tricuspid area D) Mitral area

B) Pulmonic area Notes 1) The best place to listen for the S3 heart sound is the pulmonic area (the pulmonic area is near the ventricles). 2) The S3 heart sound is pathognomic for heart failure. Leik

Which type of exercise would you recommend to a 65-year-old arthritic patient who complains of a new onset of a painful, swollen left knee caused by gardening for 2 days? A) Quadriceps-strengthening exercises of the left knee followed by the application of cold packs for 20 minutes 4 times a day B) Rest the joint and apply cold packs intermittently for the next 48 hours C) Passive range of motion and cold packs D) A cool tub bath with warm packs on the knee to avoid stiffening of the joint

B) Rest the joint and apply cold packs intermittently for the next 48 hours New onset of a painful, swollen left knee caused by unusual activity should be treated with rest and alternating ice packs to rule out other causes of pain first.

An adolescent male reports the new onset of symptoms 1 week after returning from a hiking trip in North Carolina. He presents with complaints of high fever, severe headache, muscle aches, and nausea. The symptoms are accompanied by a generalized red rash that is not pruritic. The rash initially appeared on both ankles and wrists and then spread toward the patient's trunk. The rash involves both the palms and the soles. Which of the following conditions is most likely? A) Meningococcemia B) Rocky Mountain spotted fever (RMSF) C) Idiopathic thrombocytopenic purpura (ITP) D) Lyme disease

B) Rocky Mountain spotted fever (RMSF) Best Clues 1) Location and activity (south central U.S., outdoor activity) 2) Classic rash (red rash on both wrists and ankles that spreads centrally with involvement of the palms and the soles) 3) Systemic symptoms (high fever, headache, myalgia, nausea) Leik

A newly diagnosed middle-aged type 2 diabetic wants to start an exercise program. All of the following statements are true except: A) If the patient is unable to eat due to illness, antidiabetic agents can be continued with frequent glucose monitoring B) Strenuous exercise is contraindicated for type 2 diabetics because of a higher risk of hypoglycemic episodes C) Exercise increases the body's ability to metabolize glucose D) Patients who exercise vigorously in the afternoon may have hypoglycemic episodes in the evening or at night if they do not eat

B) Strenuous exercise is contraindicated for type 2 diabetics because of a higher risk of hypoglycemic episodes Exercise is recommended because exercise helps to use the glucose stores and reduce blood sugar. Blood sugar should be monitored closely, especially if on insulin, when exercising to avoid hypoglycemia.

When an adolescent male's penis grows more in length than width, at which of the following Tanner stages is he classified? A) Tanner Stage II B) Tanner Stage III C) Tanner Stage IV D) Tanner Stage V

B) Tanner Stage III Tanner Stage III in males consists of penis lengthening and darker, coarse pubic hair, which begins to curl.

During a breast exam of a 30-year-old nulliparous female, the nurse practitioner palpates several rubbery mobile areas of breast tissue. They are slightly tender to palpation. Both breasts have symmetrical findings. There are no skin changes or any nipple discharge. The patient is expecting her menstrual period in 5 days. Which of the following would you recommend? A) Referral to a gynecologist for further evaluation B) Tell her to return 1 week after her period so her breasts can be rechecked C) Advise the patient to return in 6 months to have her breasts rechecked D) Schedule the patient for a mammogram

B) Tell her to return 1 week after her period so her breasts can be rechecked Prior to having menses, women will experience breast changes of tenderness with palpation, with symmetrical soft, mobile areas of breast tissue that are benign. Her symptoms should improve after her menses and repeating the exam in 1 week would be recommended.

Which of the following statements is false regarding jaundice in breastfed infants? A) Breastfed infants have a higher incidence of hyperbilirubinemia compared with formula-fed infants B) The mechanism of breast milk jaundice is still not known C) Phytotherapy is usually not indicated for these infants D) It usually starts in the first week of life

B) The mechanism of breast milk jaundice is still not known Jaundice occurs after the first week of life for breastfed infants. Breastfed infants usually have a higher incidence of jaundice, and the condition may last for 2-3 weeks. Bilirubin is broken down and excreted in the urine and stool. The mechanism of breast milk jaundice is still unknown.

A split S2 is best heard at which of the following areas? A) The aortic area B) The pulmonic area C) The tricuspid area D) The mitral area

B) The pulmonic area The split S2 is best heard at the pulmonic area of the heart.

An older woman complains of a new onset of severe pain in her right ear after taking swimming classes for 2 weeks. On physical exam, the right ear canal is red and swollen. Purulent green exudate is seen inside. All of the following are true statements except: A) Pulling on the tragus is painful. B) The tympanic membrane is translucent with intact landmarks. C) The external ear canal is swollen and painful. D) Pain on palpation of the mastoid area.

B) The tympanic membrane is translucent with intact landmarks. Leik

During a sports physical exam, a 16-year-old patient is noted to have a few beats of horizontal nystagmus on extreme lateral gaze that disappear when the eyes move back toward midline. Which statement best describes this clinical finding? A) It is caused by occult bleeding of the retinal artery B) This is a normal finding C) It is a sign of a possible brain mass D) This is a borderline result and requires further evaluation

B) This is a normal finding Horizontal nystagmus is a normal variation on physical exam.

While reviewing some lab reports, the nurse practitioner notes that one of her teenage male patient's lab results is abnormal. The liver function tests are all normal except for a slight elevation in the alkaline phosphatase level. The patient is a member of a soccer team and denies any recent injury. What is the next step in this patient's evaluation? A) Order a liver ultrasound to rule out fatty liver B) This is a normal finding for his age group C) The patient needs to be evaluated further for pancreatic disease D) Refer the patient to a pediatric rheumatologist

B) This is a normal finding for his age group Elevated levels of alkaline phosphatase are commonly seen in growing children. Growth of bone tissue elevates the alkaline phosphatase levels.

The best screening test for detecting and monitoring both hyperthyroidism and hypothyroidism is: A) The total T3-4 B) Thyroid-stimulating hormone (TSH) C) Thyroid profi le D) Palpation of the thyroid gland

B) Thyroid-stimulating hormone (TSH) The best "screening" test of hypothyroidism and hyperthyroidism is the TSH. If this result is abnormal, then further diagnostic tests should be performed.

The best test for diagnosing congenital glaucoma is which of the following? A) Fluorescein staining B) Tonometry C) Snellen vision exam D) The refractive index

B) Tonometry A tonometer is used to measure the intraocular pressure (IOP) of the eye to screen for glaucoma. Normal range IOP is 10-22 mm Hg.

Reccomendation for 24 year old woman with a PAP result of HSIL. a) Reflex HPV DNA testing B) colposcopy c) LEEP d) cryotherapy

B) colposcopy Recommended for adult women 21 - 24 with high grade squamous intraepitheial lesion, atypical glandular cells, high-grade intraepithelial neoplasia and atypical squamous cells.

1. A 45-year old female patient being treated for hepatitis C presents for migraine prophylaxis. Her migraines occur twice monthly when she is treated. She is concerned her migraines will recur weekly since she weened herself off propranolol due to the side effect of fatigue. She tells you today, "I take too many chemical medications, I want something natural" What is the best response by the FNP. A. "Why don't I restart your propranolol for HA prophylaxis. It has the best evidence for migraine prevention." B. "Riboflavin is vitamin B2, it is well studied and efficacious for migraine prevention." C. "Butterbur has strong evidence as migraine prevention. It is the safest option because of your history of hepatitis C". D. "Gingko Biloba is the best choice because it is shown to decrease migraines by 50%."

B. "Riboflavin is vitamin B2, it is well studied and efficacious for migraine prevention." Propranolol does have efficacy for migraine prevention, but the patient has already told you that she weened herself off propranolol and provided a rationale for doing so. Riboflavin is shown in two RCTs to be effective. One RCT found the experimental group taking a 400 mg tab daily had two fewer migraines a month, after a three-month period (Rajapske &Prinsheim, 2016). The goal of prophylactic migraine therapy is to decrease migraines by 50 percent. Although it may not be the most efficacious migraine medication, it is well-studied, fits the patient profile the best, and is the correct answer. Butterbur has strong evidence for migraine prevention, but it has been linked to cases of liver hepatotoxicity. I do not expect FNPs to know extensive information on butterbur, but it is important to know not to prescribe to a patient being treated for Hep. C. Gingko biloba is not given for migraine prevention. It may be alluring to someone with limited knowledge to choose answer D because of the phrase, "decrease migraines by 50%".

A 47-year-old woman with a history of seizure disorder presents to the clinic with an upper respiratory illness. She states that she "really needs to quit smoking" as this is her third respiratory illness this year. As the provider you know that nicotine replacement therapy doubles the chance of successful smoking cessation. All the following are suitable options for this patient EXCEPT: A. Nicotine gum B. Bupropion C. Varenicline D. Nicotine inhaler

B. Bupropion Several Pharmacological therapies have been shown effective in smoking cessation. The nicotine patch, gum, and lozenges are available over the counter while Nicotine nasal spray and inhalers are available by prescription only. Varenicline, a partial nicotinic acetylcholine-receptor agonist, has been shown to improve cessation rates, although its adverse effects including those on mood are not completely understood. The sustained release antidepressant Bupropion is an effective agent used for smoking cessation with minimal weight gain, but seizures are a contraindication. Bupropion lowers the seizure threshold.

A 46-year-old man presents to the office with complaints of acute epigastric pain radiating to the right shoulder. Patient states that the pain began after eating a large meal and has not subsided since. His pain score is 8/10. Physical exam findings reveal a positive Murphy's sign and rebound tenderness. Which of following diagnoses is most consistent with these findings? A. Pancreatitis B. Cholecystitis C. Peptic Ulcer Disease D. Appendicitis

B. Cholecystitis Classic findings of acute cholecystitis include severe, unrelenting epigastric pain that is typically preceded by a large meal. Physical exam findings include rebound tenderness, guarding and a positive Murphy's sign

A 32y/o mother brings her 8y/o daughter to your clinic with complaints of painful hematuria. As a new Family Nurse Practitioner, what should you be investigating for? A. Complete Blood Count B. Direct Injury to the Urinary Tract C. Clue cells D. Abdominal pain with fever

B. Direct Injury to the Urinary Tract Children with painful hematuria should be investigated for Urinary Tract Infections or direct injury to the urinary tract

A mother brings her one year old to his well child visit. At this visit the provider discusses the importance of the application of fluoride varnish. The mother decides to allow the provider to apply the fluoride varnish what should the provider tell the mother once the varnish is applied? A. When you get home brush the child's teeth with fluoride toothpaste B. Do not brush or floss the teeth, and give only soft foods until the next morning C. Do not brush or floss the teeth for one week D. Eat only soft foods for 7 days until the fluoride varnish is all dissolved into enamel.

B. Do not brush or floss the teeth, and give only soft foods until the next morning Fluoride varnish will set quickly to a dull yellow film upon contact with saliva. Caregivers should be instructed to not floss or brush the teeth, and to give the child only soft foods until the next morning. This will provide the fluoride enough time for absorption into the teeth's enamel.

2. A 34 year old woman with worsening panic attacks returns to the clinic one week after starting venlafaxine XR 37.5 mg po daily. She continues to take clonazapam 0.5 mg po bid for her symptoms. She states, "The medication is not working. I am still having shortness of breath, dizziness, and tremors." What should the NP do next? A. Tell the patient the medication may not be working and switch the medication to sertraline B. Educate the patient it can take 2 to 4 weeks to feel some effects of the medication and increase venlafaxine to 75 mg po qd C. Perform an EKG and increase her clonazepam to 1 mg po bid D. Add buspirone and propanolol to her medications

B. Educate the patient it can take 2 to 4 weeks to feel some effects of the medication and increase venlafaxine to 75 mg po qd Antidepressants can take from 2 to 4 weeks to be effective. Educate the patient to continue taking the medication. She will be re-evaluated every two weeks to determine effectiveness and monitor for side effects (Nelson, 2017). Do not change the medication to another anti-anxiety medication until the first medication has time to take effect. Increasing her clonazepam may be reasonable but the patient is already adding a new medication that has not had time to take effect. The patient will have to be tapered down to prevent benzodiazepine dependence. An EKG is not necessary at this time. The question does not imply the patient has a cardiac history

1. A 48 y/o Caucasian female presents to clinic for re-evaluation of blood pressure. Two weeks ago, her BP was 152/96 mm Hg at her annual wellness exam. Today, her BP is 148/94 mm Hg. The Nurse Practitioner expects to do which of the following at this office visit? A. Request a blood pressure log and re-evaluate in two weeks B. Encourage lifestyle modifications, prescribe hydrochlorothiazide C. Encourage DASH diet, prescribe Metoprolol D. Encourage lifestyle modifications and re-evaluate in one month

B. Encourage lifestyle modifications, prescribe hydrochlorothiazide First-line treatment for Hypertension is lifestyle modifications. A diagnosis of HTN can be given when the blood pressure is greater than 140/90 mm Hg on two separate occasions. Medication management in this age group with no comorbidities is hydrochlorothiazide (Papadakis and McPhee, 2017).

A 32-year-old female presents to your clinic complaining of menstrual irregularity over the past two months accompanied by fatigue, palpitations, ten-pound weight loss, and anxiety. She reports that three months ago she was seen and treated for a viral upper respiratory infection and sore throat. Her remaining medical and social history is unremarkable. The patient reports her viral symptoms have resolved, but the sore throat is still present. Your initial first step includes: A. Ordering a strep test B. Examining her neck C. Ordering serum blood tests D. Performing a pelvic exam

B. Examining her neck* Examining the patient's thyroid gland should be the first step. The symptoms are congruent with hyperthyroidism with an initial presentation of a viral infection. The viral infection is typical in the development of subacute thyroiditis (also known as de Quervain thyroiditis). Although the other options may be appropriate the initial step should always be an assessment. (Fitzgerald, 2018, p. 1138).

Mr. Smith is a 36 year old male with T2DM and HTN. He presents to your primary care clinic with complaints of a headache. He states the headache comes on from the base of his skull and wraps around to his eyes, and feels most like a vise gripping his head. He says he gets these headaches 2 times per month at most and he usually goes to work even when he has one. The best medication for this patient would be: A. Triptan B. Excedrin C. Ergotamine D. Amitriptyline

B. Excedrin* Tension type headaches are best treated with simple analgesics. Acetaminophen or Ibuprofen would be best first line medications, however Excedrin is the best choice on this list. Triptans are contra indicated in patients with risk for stroke (DM, HTN). Ergotamine should be avoided in patients with cardiovascular disease as well. Amitriptyline is an antidepressant that can be used for the prophylaxis of migraine that occur 2-3 times per month and cause significant disability with attacks.

A 46 male patient presents to the clinic for a routine checkup with complaints of a chronic cough, sore throat, occasional chest pain, and difficulty sleeping. Cardiac and respiratory workups have been done and ruled out as the cause. What other differential diagnosis should the family nurse practitioner most likely consider: A. Obstructive Sleep Apnea B. Gastroesophageal Reflux Disease C. Cholelithiasis D. Bronchitis

B. Gastroesophageal Reflux Disease Atypical causes of Gastroesophageal reflux disease include asthma, chronic cough, chronic laryngitis, sore throat, non-cardiac chest pain, and sleep disturbances. Respiratory causes such as OSA and bronchitis were ruled out. Chronic cough, sore throat, chest pain, and sleep disturbance are not symptoms associated with cholelithiasis

A 41-year-old African American male is seen in clinic with a blood pressure of 152/96. At his prior wellness visit, his blood pressure was 160/90 and lifestyle modifications were discussed. He reports to you today that he has stopped smoking but that according to his home BP log his systolic blood pressure is still consistently >140 mmHg. You suggest which medication for this patient? A. Losartan B. Hydrochlorothiazide C. Metoprolol D. Lisinopril

B. Hydrochlorothiazide Thiazide diuretics have been studied extensively and is effective in treating HTN. JNC8 suggests that for African American males less than 60 years of age and with a BP > 140/90, either a thiazide diuretic or calcium channel blocker be started either alone or in combination. They suggest that you start with one drug and then add a second one if necessary after the first agent has been titrated to maximum dosage.

When educating a patient over herpes simplex virus, the nurse practitioner shares all of the following with their patient, except: A. Herpes Simplex Virus is a common cause of genital ulcers. B. If the first outbreak is treated appropriately, no more outbreaks will occur. C. Antivirals can reduce the severity and/or duration of an HSV flare. D. Obtaining a viral culture is the gold standard of diagnosing HSV.

B. If the first outbreak is treated appropriately, no more outbreaks will occur. * Treatment of the initial outbreak does not prevent further outbreaks

Microcytic hypochromic cells are commonly seen is which type of anemia? A. Pernicious Anemia B. Iron Deficiency Anemia C. Sickle Cell Anemia D. Anemia of Chronic Disease

B. Iron Deficiency Anemia* Iron deficiency anemia is characterized by the appearance of Hypochromic (pale) Microcytic (Small) red blood cells.

2. A 15-year-old male visits your clinic for a wellness checkup. He states he uses cocaine about once a week. Which statement is true about cocaine? A. It is a depressant that can cause pinpoint pupils, hypotension, and urinary retention. B. It is a stimulant that can cause anxiety, paranoia, dry mouth, and nasal septum ulceration. C. It is a depressant that can cause memory loss, and with chronic use, can cause Wernicke-Korsakoff syndrome. D. It is a stimulant that can lead to paranoid psychosis if used chronically.

B. It is a stimulant that can cause anxiety, paranoia, dry mouth, and nasal septum ulceration.* Answer A describes opioid use. Answer C describes GHB use. Answer D describes ecstasy use

A 35 year old male presents to the office with a chief complaint of chronic headaches. Today he has a unilateral pulsatile headache accompanied by photophobia and nausea. The Nurse Practitioner would most likely diagnose this headache as: A. Tension-type B. Migraine C. Cluster D. Subarachnoid hemorrhage

B. Migraine Typically pulsatile, unilateral, worse with physical activity, with nausea, vomiting, photophobia, phonophobia, osmophobia, cognitive impairment, and blurring of vision. They may/may not have an aura.

Mr. Smith comes to the office with complaints of heartburn after every meal he reports that the heartburn is worse about 30 minutes after a heavy meal. After having the same symptoms for several months, he is referred for an upper endoscopy where he is diagnosed with GERD. What medication will you first prescribe Mr. Smith for his diagnosis of GERD? A. Calcium Carbonate (Tums) B. Omeprazole (protonix) C. Famotidine (pepcid) D. Metoclopramide (reglan)

B. Omeprazole People with known complications of GERD, and troublesome reflux should be treated with a once daily oral proton pump inhibitor. Omeprazole 20mg will achieve adequate control of heartburn in 80-90% of patients. The use of proton pump inhibitors are preferred over H2-receptor antagonist for the treatment of acute and chronic GERD.

A 53 year old male presents to the office with silvery scales overlying bright red, well-demarcated plaques bilaterally on his elbows, and developing on his knees. What dermatologic condition would the Nurse Practitioner identify this as? A. Atopic dermatitis B. Plaque psoriasis C. Pityriasis rosea D. Seborrheic dermatitis

B. Plaque psoriasis The combination of red plaques with silver scales on elbows and knees, with scaliness in the scalp or nail findings, is diagnostic. Psoriasis lesions affect the extensor surfaces and are well demarcated

A 4 year old female presents with vomiting and diarrhea for 2 days with fevers. She has had 7% body weight loss, skin is pale with decreased turgor, and oliguria. She is producing tears when she cries. She is able to tolerate fluids with Zofran administration. The best course of management would be: A. Admit to Pediatrics for 24 hours of IV fluid resuscitation B. Push fluids of 5 ml every 3 min at home until she is able to drink 100ml/kg in 4 hours + losses from vomiting C. Administer Zofran every 4 hours and drink 100ml/kg in 24 hours D. Encourage sleep and rehydration with Pedialyte without concern for uid quantity

B. Push fluids of 5 ml every 3 min at home until she is able to drink 100ml/kg in 4 hours + losses from vomiting Important clinical features in estimating the degree of dehydration include the capillary refill time, postural blood pressure, and heart rate changes; dryness of the lips and mucous membranes; lack of tears; lack of external jugular venous filling when supine; a sunken fontanelle in an infant; oliguria; and altered mental status.

67 yo patient in ICU develops peaked T waves with a heart rate of 52. The patient has muscle weakness in bilateral lower extremities. Labs drawn this morning showed Potassium level of 5.9. According to the bedside nurse, the patient has had decreased urine output of 20cc/hr for the past 2 hours. As the Nurse Practitioner, what will you prescribe as the first line treatment for hyperkalemia? A. K-Dur 40 mEq PO BID x 3 days B. Regular Insulin 10 units IV and D50 25g IV C. Aldactone 50mg IV and Bicarbonate 50 mEq IV D. Dialysis

B. Regular Insulin 10 units IV and D50 25g IV Insulin shifts potassium into cells within 15 minutes of administration, but also will decrease the blood glucose. Administering D50 IV, in conjunction with the insulin, will help stabilize the blood glucose while insulin is shifting potassium intracellularly.

Death is understood to be permanent starting at what age group ? A. Preadolescence B. School Age C. Toddler D. Preschool

B. School Age* Toddlers have no cognitive understanding of death. Preschoolers view death as something that is not permanent. Preadolescence understand at an adult level, but they can start to understand death is permanent at School Age.

A four year old child presents to the clinic for yearly well child visit in October. The child is current on vaccinations and there are no allergies or contraindications to vaccinations, which vaccinations should the child receive? a. DTaP, MMR, Varicella, Influenza b. DTaP, MMR, IPV, Varicella, Influenza c. DTaP, MMR, IPV, Varicella d. DTaP, MMR, IPV, Varicella, PCV 13, Influenza

B. She needs her 4 year old shots plus a flu shot. Remember 4 year old shots as "Very DIM at 4": Varicella DTap IPV MMR According to the CDC, a healthy four-year-old with no contraindications or allergies should have the Diphtheria, tetanus, and acellular pertussis (DTaP), Poliovirus Inactivated (IPV), Measles, mumps and rubella (MMR), Varicella and Influenza vaccine(CDC, 2018; Daley, O'Leary, & Nyquist, 2018 Question pool

A 27-year-old female comes in for a wellness visit. In reviewing her medication list, she reports taking sertraline (Zoloft) for mild depression, ethinyl estradiol and norethindrone, and ferrous fumarate (Lo Loestrin Fe), and some over-the-counter supplements. Which over-the-counter supplement do you need to assure that she is not taking, in order to guarantee effectiveness of her oral contraceptive and avoid serotonin syndrome? A. Garlic B. St. John's wort C. Echinacea D. Asian ginseng

B. St. John's wort* St. John's wort (Hypericum perforatum) is a botanical over-the-counter medicine that has shown to be more effective than placebo for mild to moderate depression and as effective as prescription antidepressants. It is very well tolerated; however, it interacts with several medications. It can cause serotonin syndrome if taken in conjunction with prescription antidepressants. It also induces the cytochrome P450 system (isozyme CYP3A4) which lowers the blood levels of certain medications, including protease inhibitors, digoxin, alprazolam, oral contraceptives, cyclosporine, and warfarin

A mother brings her 6-year old son to your clinic for an issue with his skin. She explains that he had a few ant bites on his leg last week, but now they are worse and spreading. When you examine the child's leg you find eight lesions with honey colored, crusted exudate. The appropriate treatment for this child is: a. Hydrocortizone cream 1% b. Mupirocin ointment c. Ketoconazole topical d. Miconazole topical

B. The correct answer is Mupirocin ointment. Lesions with dried, honey colored exudate indicate impetigo, usually cased by staph or strep. Topical Mupirocin (abx) is the recommended treatment for small areas of infection Question pool

In pediatric patients presenting with frequent migraine headaches, what should the NP recognize as being the best initial phase of treatment? A. Explain to the parent the best option is to administer an antileptic medication for prevention of future migraines. B. Treat early by placing child in a cool dark room. Explain to the parent and child the need to keep a headache diary to possibly distinguish the precipitating factor(s). C. Give abortive medications as frequently as needed. D. Use simple analgesics every 4 hours x 1 week.

B. Treat early by placing child in a cool dark room. Explain to the parent and child the need to keep a headache diary to possibly distinguish the precipitating factor(s). Management of headaches should emphasize the necessity for early and adequate treatment during a headache, in addition to self-management skills to reduce frequency and disability such as life-style modification and headache diaries. Simple analgesics should be limited to 2-3 times per week and migraine-specific medications to approximately 1-2 times per week. During a headache biobehavioral techniques include rest, relaxation, and cold/hot packs. Providing the child with a cool dark room in which to rest made provide added benefits

A 63-year-old patient is being seen in the clinic with complaints of a red, burning rash with grouped, vesicular lesions, along the right side of his back and flank. His pain is a 7/10. He reports a history of HIV. His current CD4 count is 180. Based upon the assessment, which treatment plan is the best course of action? A. Prednisone 60mg po daily tapered over 3 weeks. B. Valacyclovir 1g po TID until lesions are healed. C. Clindamycin 150mg po Q6h for 10 days D. Acyclovir 800mg po five times/day for 7 days.

B. Valacyclovir 1g po TID until lesions are healed. This patient is showing signs and symptoms of Herpes Zoster, which involves pain along a nerve that is burning in nature. The lesions are typically vesicular and can be grouped in up to 20 lesions. They are found unilaterally on the face or trunk of the body along a dermatome. Patients with a history of HIV are more likely to develop Herpes Zoster than those without HIV. The patient's CD4 count is 180, suggesting he is immunocompromised. Treatment for the immunocompromised host with Herpes Zoster are antivirals for about two weeks, which is the .amount of time it takes for the lesions to completely crust over and heal. Corticosteroid tapered therapy and treatment with antivirals for 7 days are appropriate treatment options for someone with Herpes Zoster that is immunocompetent. Clindamycin is an effective antibiotic for many skin infections, however antibiotics do not effectively treat Herpes Zoster.

Amy, aged 44, has a maternal aunt who died of breast cancer. You are counseling her on her individual risks for getting breast cancer. Risk factors for breast cancer include all of the following except: A. Family history of breast cancer B. Vigorous exercise C. Drinking 3-4 alcoholic drinks daily D. Having dense breast tissue

B. Vigorous exercise Breast cancer risks are increased with family history of breast cancer, drinking 2 or more alcoholic drinks every day, and breast density. Exercise is associated with a decreased risk of breast cancer.

As a FNP, it is important to know which category of sexually active individuals are most at risk for STI transmission due to their sexual behaviors? A. MSM B. WSW C. Heterosexual men and women D. Bisexual men and women

B. WSW WSW, women who have sex with women exclusively, are generally at higher risk for STI transmission. The CDC, reports the WSW have many diverse sexual practices, but have the lowest rates with use of barrier practice. This population tends to be at a higher risk in general due to their more divers sexual practices including: digital-vaginal, vaginal-vaginal, digital-anal, oral-vaginal, and oral anal contact. Statistically, studies showed specifically rates of chlamydial infection were higher among women who reported same-sex practices than those who reported heterosexual practices in the 14-24 age group

A 59 male presents with painful bladder x 4 days. Denies hematuria or fever. UA is negative, and CBC is within normal limits. Reports he gets these often and feels better after urination. Which diagnosis is most likely? A. acute epididymitis B. interstitial cystitis* C. acute cystitis D. bacterial prostatitis

B. interstitial cystitis these are all presenting symptoms of interstitial cystitis. With all of the other diagnoses, pt may have fever and positive urine culture.

An 18-month-old male presents to your office with his mother, who reports the infant has been experiencing recurring episodes of abdominal pain with screaming and drawing up of his knees for three days. She also reports that after these episodes, he vomits and has diarrhea. The last episode occurred last night around 9:00 PM. She reports this morning around 7:00 AM, he had a bloody bowel movement that contained mucus. Upon examination, you notice the abdomen is distended and tender, and you are able to palpate a sausage-shaped mass. Assuming all equipment is readily available, your next best action should be to: A. administer a barium enema B. obtain an abdominal ultrasound C. write a prescription for an anti-diarrheal medication D. attempt to reduce the sausage-shaped mass

B. obtain an abdominal ultrasound Abdominal ultrasound carries sensitivity for diagnosis of intussusception of 98%-100%. The likelihood of bowel compromise increases with the duration of symptoms. Reduction of the intussusception by barium enema should not be attempted if signs of strangulated bowel, perforation, or toxicity are present. Anti-diarrheal medications are ineffective in acute cases of diarrhea and can be dangerous in some circumstances

Which of the following statements is true regarding vitamins? A.) Fat-soluble vitamin deficiencies develop more quickly than water-soluble vitamin deficiencies. B.) Beginning shortly after birth, 400 IU of Vitamin D daily is recommended to all breastfed infants. C.) A vegan diet of a child that omits all animal protein sources does not require any vitamin supplementation as they can obtain this through other sources of food. D.) Rickets is caused by a deficiency in Vitamin E.

B.) Beginning shortly after birth, 400 IU of Vitamin D daily is recommended to all breastfed infants. Rationale: Fat-soluble vitamins are absorbed and stored in the body, while water-soluble vitamin excesses are excreted in the urine. This leads to water-soluble vitamin deficiencies developing more quickly. According to American Academy of Pediatrics, daily intake of 400 IU of Vitamin D is recommended for all infants. Infant formula is routinely already supplemented with this, while breast-fed infants will need the supplementation. A vegan diet without any animal sources of protein will require supplementation of Vitamin B ₁₂. Rickets is caused by a deficiency in Vitamin D.

Your 25 yo patient has just been diagnosed with bacterial vaginosis. She has no known allergies. All are recommended options for the treatment of bacterial vaginosis except: A.) Metronidazole, 500 mg orally twice a day for 7 days B.) Doxycycline, 100 mg orally twice a day for 7 days C.) Clindamycin, 300 mg orally twice a day for 7 days D.) Metronidazole, 0.75% gel, 5 g intravaginally once daily for 5 days

B.) Doxycycline, 100 mg orally twice a day for 7 days Doxycycline is not recommended as a treatment for BV. It can be used in the treatment of other issues such as pelvic inflammatory disease, chlamydia and urethritis.

1.) The nurse practitioner is formulating a treatment plan for sustained hypertension in an obese 13 year old male who has asthma. What medication would be the best first line agent? A.) Metoprolol B.) Lisinopril C.) Hydralazine D.) Minoxidil

B.) Lisinopril ACE inhibitors are the preferred treatment choice by pediatric nephrologists given the frequent renal etiology of hypertension in this population. Lisinopril is the only ACE inhibitor listed. Metoprolol is a beta blocker and is contraindicated in patients with reactive air disease. Hydralazine and Minoxidil are great treatment options, but typically require diuretics and/or beta blockers to combat associated sodium and fluid retention and reflex tachycardia.

Your 23 yo patient is looking to start birth control. She reports regular monthly periods that are painful, and heavy. She has never been on birth control. She has no history of pelvic inflammatory disease. Her pregnancy test is negative. As the NP you would recommend A.) The copper IUD B.) Mirena C.) NuvaRing D.) Skyla

B.) Mirena* The hormone-releasing IUD Mirena has been approved by the FDA to treat heavy menstrual bleeding

Why does treatment of vertigo include bedrest and an antihistamine?

Bedrest wtih vertigo is recommended for safety reasons. Antihistamines are vestibular suppressant drugs, and help alleviate the sx. The vestibular system is one of 3 systems that helps maintain spatial orientation and posture. The visual system gives clused to uprightness adn the somatosensory system gives information from the muscles, joints and skin about position. These system work together to compensate when one system is deficient.

What HTN medications are a good choice for patients with migraines or diabetics who do not have chronic lung disease?

Beta blockers NOTE: Betablockers are contraindicated in asthmatics or patients with chronic lung diseases such as asthma, chronic obstructive pulmonary disease (COPD), emphysema, or chronic bronchitis. Leik

Why are patients who have an anaphylactic reaction often prescribed a Medrol Dose Pack and a long-acting antihistamine after being discharged from the ED?

Biphasic anaphylaxis occurs in up to 23% of cases (symptoms recur within 8-10 hours after initial episode). Leik

Normal children's vision at: Birth 4 months 3-4 years 6 years

Birth - 20/100 4 months - 20/80 3 - 4 years = 20/50 6 years = 20/20 or 20/25

A 3-year-old child presents to the clinic with vomiting secondary to viral gastroenteritis. Which child should be referred to the hospital immediately? A. Dry mucous membranes, tachycardia, mild oliguria, normal blood pressure, decreased skin turgor B. Decrease in body weight, Dry mucous membranes, Capillary refill 2-3s C. Capillary refill >4s, tachycardia, oliguria, poor color, no tear production D. Pale color, capillary refll 3-4s, Normal Blood pressure, decreased tear production

C According to Hay, emergent intravenous therapy is indicated where there is evidence of compromised perfusion ( inadequate capillary refill, tachycardia, poor color, oliguria or hypotension) (Ford & Hannah, 2018). Question pool

Shelly is a 5-year-old child who was never vaccinated. Shelly's mom has done a lot of researching on the topic and has decided today that she was a fool and Shelly needs to be immunized. Which of the following combinations of vaccines are appropriate for Shelly, who has no allergies, no immunodeficiency, and no chronic illnesses? A: Hep B, Hib, Varicella, and PCV13 B: Tdap, Varicella, MMR C: DTaP, Varicella, MMR, and Hep B D: Hep B, Hep A, PCV13, and HPV

C Hib is not recommended for children 5 years and older. Tdap is not given until the patient is 7 years old. DTaP, Varicella, MMR and Hep B are all safe and recommended as catch up vaccines for patients aged 5 years. HPV is not recommended until patient is age 11 (may start at age 9). Question pool

A 62-year-old male is seen by the FNP this morning with complaints of sudden onset of epigastric abdominal pain, fever, nausea, and vomiting since last night. He denies coffee ground emesis, frank blood, or melena. He has not eaten this morning and his last meal yesterday consisted of bratwurst and sauerkraut with 2 beers at home with family, of whom none are showing similar symptoms. On physical examination, the presence of which sign supports the FNP's suspicion of acute cholecystitis? a. Cullen's sign b. McBurney's sign c. Murphy's sign d. Rovsing's sign

C Murphy's sign is pain in right upper quadrant of the abdomen elicited with inspiration when palpating the RUQ just below the costal margin at the mid-clavicular area and is nearly always present with acute cholecystitis. Cullen's sign appears as bruising near the umbilicus and may indicate peritoneal bleeding. McBurney's and Rovsing's signs are tests for acute appendicitis. Question pool

52 years old female present to the clinic with complaint of weight gain. She was diagnosed a year ago with chronic kidney disease and hypertension. She has gained 4 pounds over the last 3 months, despite exercising regularly. She has been complaining about her blood pressure medication and today BP 126/74, HR 72, T 98.6F, W=181 lbs. She denies any shortness of breath, fatigue, anxiety, or depression. Her last metabolic panel reveals a GFR of 29. Which diuretic would you recommend in addition to salt restriction? A Hydrochlorothiazide B Spironolactone C Furosemide D Triamterene

C Furosemide Loop diuretics (Furosemide, bumetanide, torsemide) are more effective in the treatment of a patient with CKD stage 3 when the GFR is less than 30ml/min. When GFR is greater than 30 ml/mi Thiazide diuretics are recommended.

A 39-year-old migrant worker presents to the clinic 2.5 days after a purified protein derivative (PPD) test. What minimum size of induration would be considered positive for this patient? A) 3 mm B) 5 mm C) 10 mm D) 15 mm

C) 10 mm The PPD is administered on the volar aspect of the lower arm and read 48 hours after the test is given. The PPD result must have induration and measure 10 mm or greater to be positive in a low-risk patient. Induration (firmness with palpation) must be present. If the site has erythema but no induration, result would be negative. Color is not important.

Mrs. J. L. is a 55-year-old female with a body mass index (BMI) of 24 and a history of asthma. She has hypertension that has been under control with hydrochlorothiazide 12.5 mg PO daily. Her total cholesterol is 230 g/dl. How many risk factors for coronary heart disease (CAD) does she have? A) 1 risk factor B) 2 risk factors C) 3 risk factors D) 4 risk factors

C) 3 risk factors Risk factors for coronary heart disease include hypertension, hypercholesterolemia, and women 55 years of age and older.

The following are patients who are at high risk for complications due to urinary tract infections. Who does not belong in this category? A) A 38-year-old diabetic patient with a HbA1C of 7.5% B) A woman with a history of rheumatoid arthritis who is currently being treated with a regimen of methotrexate and low-dose steroids C) A 21-year-old woman who is under treatment for 2 sexually transmitted infections D) Pregnant women

C) A 21-year-old woman who is under treatment for 2 sexually transmitted infections Risk factors for complications due to urinary tract infection include pregnancy, diabetes, and steroid therapy

All of the following are associated with emphysema except: A) A barrel-shaped chest B) Pursed-lip breathing C) A chest radiograph result with infiltrates and flattening of the costovertebralangle D) Dyspnea when at rest

C) A chest radiograph result with infiltrates and flattening of the costovertebralangle Emphysema is characterized by having a barrel-shaped chest, pursed lip breathing, and dyspnea when at rest. Infiltrates on an x-ray indicate bacterial infection, such as pneumonia.

Pulsus paradoxus is best described as: A) An increase in systolic blood pressure on inspiration B) A decrease in diastolic blood pressure on exhalation C) A decrease in systolic blood pressure on inspiration D) An increase in diastolic blood pressure on expiration

C) A decrease in systolic blood pressure on inspiration Pulsus paradoxus: With inspiration, systolic pressure drops due to the increased pressure (positive pressure). Some pulmonary risks of having increased pressure include asthma and emphysema.

All of the following are true regarding Multiple Sclerosis, except? A) Multiple Sclerosis is a neurologic disorder B) Is an autoimmune disease and the onset is mostly under 55 years of age. C) A definitive diagnoses can be made once laboratory results are completed. D) Several forms of the disease are recognized.

C) A definitive diagnoses can be made once laboratory results are completed. * A definitive diagnosis for Multiple Sclerosis can never be based solely on the laboratory findings. A MRI should demonstrate at least 1 lesion in at least 2 or more different regions of the central white matter

The following conditions are absolute contraindications for the use of oral contraceptives except: A) Hepatomas B) History of emboli that resolved with heparin therapy 15 years ago C) A family history of migraines with aura D) A history of gallbladder disease during pregnancy

C) A family history of migraines with aura All of the items are contraindications, but a family history does not substantiate a need to avoid oral contraceptives.

A college student has recently been informed that he has an HPV (human papilloma virus) infection on the shaft of his penis. Which of the following may reveal subclinical lesions on the penile skin? A) Perform a KOH (potassium hydroxide) exam B) Scrape off some of the affected skin and send it for a culture and sensitivity C) Apply acetic acid to the penile shaft and look for acetowhite changes D) Order a serum herpes virus titer

C) Apply acetic acid to the penile shaft and look for acetowhite changes Lesions of HPV infection will turn white with application of acetic acid.

Which of the following is a CDC-recommended treatment for a case of uncomplicated gonorrheal and chlamydial infection? A) Metronidazole (Flagyl) 250 mg PO TID x 7 days B) Valacyclovir (Valtrex) 500 mg PO BID x 10 days C) Azithromycin 1 g orally OR doxycycline 100 mg orally twice a day for 7 days D) 1 dose of oral fluconazole (Diflucan) 150 mg

C) Azithromycin 1 g orally OR doxycycline 100 mg orally twice a day for 7 days

A middle-aged hypertensive male presents to a public health clinic with complaints of an acute onset of fever, chills, and cough that is productive of rusty-colored sputum. The patient reports episodes of sharp pains on the left side of his back and chest whenever he is coughing. His temperature is 102.2 degrees Fahrenheit, the pulse is 100/min, and the BP is 130/80. The urinalysis does not show leukocytes, nitrites, or blood. This finding is most consistent with: A) Atypical pneumonia B) An upper urinary tract infection C) Bacterial pneumonia D) Acute pyelonephritis

C) Bacterial pneumonia Symptoms of bacterial pneumonia include high fever, chills, productive cough, and pain in back/chest area with deep inspiration and cough.

Which of the following classes of drugs is implicated with blunting the signs and symptoms of hypoglycemia in diabetics? A) Calcium channel blockers B) Diuretics C) Beta-blockers D) ARBs (angiotensin receptor blockers)

C) Beta-blockers

A new mother is breastfeeding her full-term 4-week-old infant. She wants to know whether she should give the infant vitamin supplements. The best advice is: A) Since she is breastfeeding, the infant does not need any vitamin supplements until he is at least 6 months of age B) Breast milk gives the infant all the vitamins he needs until 12 months of age C) Breastfed infants require iron and Vitamin D supplementation right away D) Breastfed infants require iron and Vitamin E supplementation at 3 months of age

C) Breastfed infants require iron and Vitamin D supplementation right away While infants do receive Vitamin D through human milk, the AAP recommends a daily supplement of 200 IU Vitamin D for all breastfed infants.

Which cranial nerve (CN) is being evaluated when patients are instructed to shrug their shoulders? A) CN IX B) CN X C) CN XI D) CN XII

C) CN XI Cranial nerves IX, X, XI, and XII are: glossopharyngeal, vagal, spinal accessory, and hypoglossal, respectively.

Balanitis is caused by: A) Staphyloccocus aureus B) Streptococcus pyogenes C) Candida albicans D) Trichomonads

C) Candida albicans

When the nurse practitioner is evaluating a patient for intermittent claudication, he/she would first: A) Order a venogram B) Order T.E.D. anti-embolism stockings C) Check the ankle and brachial blood pressure before and after exercise D) Check the pedal and posterior tibial pulses

C) Check the ankle and brachial blood pressure before and after exercise Initial evaluation for intermittent claudication would include checking the ankle and brachial blood pressure before and after exercise.

A 21-year-old woman with a history of mild intermittent asthma and allergic rhinitis complains of a cough that has been waking her up very early in the morning. She reports that she is wheezing more than usual. Her last office visit was 8 months ago. Which of the following is the best initial course of action? A) Initiate a prescription of a short-acting beta-2 agonist QID PRN. B) Refer the patient to an allergist for a scratch test. C) Discuss her symptoms and other factors associated with the asthmatic exacerbation. D) Perform a thorough physical examination and obtain blood work.

C) Discuss her symptoms and other factors associated with asthmatic exacerbation. Best Clues 1) The patient's asthma appears to be getting worse (asthmatic exacerbation). 2) There is a lack of information because of poor follow-up (none for 8 months). 3) There is a need to find out about precipitating factors, medication compliance, comorbid conditions, what is precipitating her asthma, and so on. Notes The correct order of actions to follow in this case scenario is the following: 1) Interview the patient to find out more about her symptoms, etc. (subjective). 2) Perform a thorough physical examination (objective). 3) Administer a nebulizer treatment. Check peak expiratory flow (PEF) before and after treatment to assess for effectiveness. 4) Initiate a prescription of a short-acting beta-2 agonist and steroid inhaler BID. Dose depends on severity of asthma (planning). 5) Refer the patient to an allergist for a scratch test if you suspect the patient has allergic asthma (evaluation). 6) Patients with asthmatic exacerbations whose PEF is less than 50% of predicted value after being given nebulized albuterol/saline treatments should not be discharged. Consider calling 911. 7) The differential diagnosis for an early morning cough includes postnasal drip, allergic rhinitis, sinusitis, GERD, and so forth. Leik

A new mother who is on her fourth day of breastfeeding complains to the nurse practitioner of very sore breasts. The nurse practitioner would: A) Recommend a decrease in the number of times she breastfeeds her infant per day B) Recommend that she stop breastfeeding and use infant formula for the next 48 hours C) Educate the mother that this is normal during the first week or 2 of breastfeeding and the soreness will eventually go away D) Recommend that she purchase plastic nipple pads for her nursing bra and use them daily

C) Educate the mother that this is normal during the first week or 2 of breastfeeding and the soreness will eventually go away Nursing during the first 2 weeks after delivery may cause tenderness and soreness of the nipples and usually resolves after this. The mother should continue to breastfeed as she has been advised, and she should make sure the infant is latching on appropriately.

Which of the following laboratory tests is a sensitive indicator of renal function in people of African descent? A) Serum blood urea nitrogen (BUN) B) Serum creatinine concentration C) Estimated glomerular filtration rate (GFR) D) Serum BUN-to-creatinine ratio

C) Estimated GFR Best Clues 1) Knowledge that the estimated GFR is a better test of renal function compared with the serum creatinine concentration. Notes 1) A GFR value of 60 or less is a sign of kidney damage (refer to a nephrologist). 2) The GFR is an "estimated" value (it is not measured directly) and is computed by using the serum creatinine value in the Cockcroft-Gault (or the MDRD Study) equations. 3) The serum creatinine is affected by age (less sensitive in elderly), gender (higher in males), ethnicity (higher with African background), and other factors. 4) The BUN is a waste product of the protein from foods that you have eaten. If you eat more protein before the test, it will increase (or decrease with low protein intake). 5) Dehydration will elevate the BUN value. Leik

An asthmatic male complains of a sudden onset of itching and coughing after taking two aspirin tablets for a headache in the waiting room. The patient's lips and the eyelids are becoming swollen. The patient complains of feeling hot. Bright red wheals are noted on his chest and arms and legs. Which of the following is the best initial intervention to follow? A) Call 911. B) Check the patient's blood pressure, pulse, and temperature. C) Give an injection of aqueous epinephrine 1:1000 (1 mg/mL) 0.5 mg IM (intramuscular) into the vastus lateralis muscle immediately. D) Initiate a prescription of a potent topical steroid and a Medrol Dose Pack.

C) Give an injection of aqueous epinephrine 1:1000 dilution (1 mg/mL) 0.5 mg IM into the vastus lateralis muscle immediately. Best Clues 1) The quick onset of symptoms such as angioedema after taking aspirin. 2) The classic signs and symptoms of anaphylaxis are described in this case. 3) Severe anaphylactic episodes occur almost immediately or within 1 hour after exposure. Leik

Possible side effects that may be seen in a patient who is being treated with hydrochlorothiazide for hypertension are: A) Dry cough and angioedema B) Swollen ankles and headache C) Hyperuricemia and hyperglycemia D) Fatigue and depression

C) Hyperuricemia and hyperglycemia (A) is caused by angiotensin-converting enzyme (ACE) inhibitors. Look for a sudden or new onset of a dry cough in a patient with hypertension (without signs of the common cold). Angioedema is a rare adverse effect and can be life-threatening. (B) is caused by calcium channel blockers (CCBs). Look for a hypertensive patient with swollen ankles (not associated with heart failure) and headache. (D) is caused by beta-blockers. Look for a patient with hypertension who complains of increased fatigue and depression (avoid if possible in depressed patients). Leik

Which of the following drugs is effective therapy for treating pain in patients who are having an acute exacerbation of gout? A) Acetaminophen (Tylenol) B) Systemic steroids C) Indomethacin (Indocin) D) Allopurinol (Zyloprim)

C) Indomethacin (Indocin) Common medications used for acute flare-ups of gout include two NSAIDs, such as indomethacin and naproxen sodium, BID prn. Colchicine may be added to the NSAIDs if relief is not obtained. Maintenance therapy consists of allopurinol and/or probenecid.

A new patient who is a 40-year-old female postal worker is being evaluated for complaints of a new onset of an erythematous rash on both her cheeks and on the bridge of the nose that is accompanied by fatigue. She reports a history of Hashimoto's thyroiditis and is currently being treated with Synthroid 1.25 mg daily. Which of the following conditions is most likely? A) Atopic dermatitis B) Thyroid disease C) Lupus erythematosus D) Rosacea

C) Lupus erythematosus Classic symptoms of lupus erythematosus are butterfly rash across both cheeks and bridge of nose and fatigue. Risk factors also include being female and 40 years old.

The red blood cells in pernicious anemia will show: A) Microcytic and hypochromic cells B) Microcytic and normochromic cells C) Macrocytic and normochromic cells D) Macrocytic and hypochromic cells

C) Macrocytic and normochromic cells The blood cells in pernicious anemia will display macrocytic, normochromic cells. RBCs in iron-deficiency anemia are microcytic and hypochromic.

What is the first-line class of antibiotics recommended by the American Thoracic Society (ATS) for patients younger than 60 years of age who are diagnosed with community-acquired pneumonia with no comorbidity? A) First-generation cephalosporins B) Second-generation cephalosporins C) Macrolides D) Beta-lactam antibiotics

C) Macrolides The American Thoracic Society recommends macrolides as the first line of therapy for community-acquired pneumonia without comorbidity.

Lead poisoning can cause which type of anemia? A) A mild macrocytic anemia B) Normocytic anemia C) Microcytic anemia D) A mild hemolytic anemia

C) Microcytic anemia Lead poisoning can cause microcytic anemia. Signs and symptoms of lead poisoning are abdominal pain, constipation, vomiting, and blue-black line on the gums. Lead causes anemia by mimicking healthful minerals such as calcium, iron, and zinc. It is absorbed by the bones, where it interferes with the production of RBCs. This absorption can also interfere with calcium absorption that is needed to keep the bones healthy.

A 27-year-old female patient presents with the complaint of a headache for the past 3 days. She reports that the pain is unilateral and describes it as dull and throbbing. She has nausea and photosensitivity. The pain is aggravated by bending over and physical activity. Her vitals are: BP 142/74, RR 17, Pulse 89, temp 99.0. Based on her presenting symptoms what is the most likely diagnosis? A) Essential Hypertension B) Cluster Headache C) Migraine Headache D) Tension-Type Headache

C) Migraine Headache*** Migraine diagnosis: pulsatile HA, lasts 4-72 hours, typically unilateral, With or without aura. Aggravated by physical activity, visual aura may proceed pain. Common associated symptoms include: N/V/photophobia, phonophobia.

Which of the following is a correct statement about colonic diverticula? A) Diverticula are more common in young adults. B) Diverticula formation happens over a period of months. C) Mild cases of diverticulitis can be managed outside the hospital. D) Dietary fiber supplementation is not recommended.

C) Mild cases of diverticulitis can be managed outside the hospital Notes 1) Diverticula are diagnosed by colonoscopy. Diverticula are asymptomatic, small, polyp-like pouches on the wall of the colon. 2) They are more common in Western society due to low intake of dietary fiber. 3) Diverticula and diverticulitis are rarely seen below the age of 50 years. 4) Mild cases of acute diverticulitis in stable patients are managed in the outpatient setting with antibiotics such as ciprofloxacin 500 mg PO BID plus metronidazole PO TID × 10 to 14 days. 5) Recommend fiber supplementation such as psyllium (Metamucil) and soluble fibers such as guar gum fiber, inulin, and apple pectin. 6) Diverticulitis infections can become life-threatening. Leik

A 12-year-old male's peak expiratory fl w results indicate 60 to 80% of the predicted range. How would you classify his asthma? A) Mild intermittent asthma B) Mild persistent asthma C) Moderate persistent asthma D) Severe asthma

C) Moderate persistent asthma Asthma classifications: Intermittent: normal FEV between exacerbations, FEV > 80%; mild persistent, FEV > 80%; moderate persistent, FEV 60-80%; severe persistent, FEV < 60%.

All of the following are true statements about diverticula except: A) Diverticula are located in the colon B) A low-fiber diet is associated with the condition C) Most diverticula in the colon are infected with gram-negative bacteria D) Supplementing with fiber such as psyllium (Metamucil) is recommended

C) Most diverticula in the colon are infected with gram-negative bacteria Diverticuli in the colon can be infected with both gram-negative and gram-positive bacteria.

Sarah a 30-week gestation patient has been diagnosed with a urinary tract infection. The safest treatment for her is: A) Sulfamethoxazole/trimethoprim 800mg/160mg BID x5 days B) Ciprofloxacin 250mg BID x3 days C) Nitrofurantoin 100mg BID x5 days D) Levofloxacin 250mg QD x3 days

C) Nitrofurantoin 100mg BID x5 days Nitrofurantoin 100mg BID, ampicillin 250mg QID, and cephalexin 250mg QID are acceptable medications to treat bacteriuria. Sulfonamides should be avoided during the third trimester because they may interfere with the binding of bilirubin and increase risk of neonatal hyperbilirubinemia and kernicterus. Fluoroquinolones are contraindicated because of potential teratogenic effects on fetal cartilage and bone

Which of the following foods are known to have high potassium content? A) Low-fat yogurt, soft cheeses, and collard greens. B) Aged cheese, red wine, and chocolate. C) Potatoes, apricots, and Brussels sprouts. D) Black beans, red meat, and citrus juice.

C) Potatoes, apricots, and Brussels sprouts. Best Clues 1) First, look at the answer option pairs for inconsistencies in the list of foods. 2) Rule out Option A because it is inconsistent and these foods do not contain high levels of potassium: low-fat yogurt and soft cheeses (calcium) with collard greens (vitamin K). 3) Rule out Option B because these foods have high tyramine content, not potassium. 4) Rule out Option D because it is inconsistent. Although citrus juices are high in potassium, both black beans and red meat are not (iron). 5) If Options A, B, and D are incorrect, then the only one left is Option C (potatoes, apricots, and Brussels sprouts. A large number of fruits and vegetables are rich in potassium and vitamins). Leik

A 25 -year-old woman's last menstrual period was 6 weeks ago. She is complaining of nausea with vomiting in the morning and fatigue. Her breasts feel bloated. The nurse practitioner suspects that she is pregnant. Her symptoms would be considered: A) Positive signs of pregnancy B) Probable signs of pregnancy C) Presumptive signs of pregnancy D) Possible signs of pregnancy

C) Presumptive signs of pregnancy Presumptive signs of pregnancy are symptoms experienced by the woman, such as amenorrhea, breast tenderness, nausea/vomiting, fatigue, and increased urinary frequency.

Which of the following is indicated for the prophylactic treatment of migraine headache? A) Ibuprofen (Motrin) B) Naproxen sodium (Anaprox) C) Propranolol (Inderal) D) Sumatriptan (Imitrex)

C) Propranolol (Inderal) + Medications used to prophylactically prevent migraine headaches include beta-blockers (propranolol) and tricyclic antidepressants (amitriptyline). +Motrin, Anaprox, and Imitrex are all medications used to treat migraine headache.

Which of the following is responsible for the symptoms of dysmenorrhea? A) Estrogen B) Human chorionic gonadotropin C) Prostaglandins D) Progesterone

C) Prostaglandins Prostaglandins are hormones the body produces prior to menses; they eventually cause the uterus to contract to shed the endometrial lining. Contractions cause pain. The greater the amount of prostaglandins that are released, the more pain one will experience. Contractions of the uterus cause vasoconstriction of blood supply to the uterus, which in turn will cause pain.

Which of the following tests would you order for an older diabetic male with the following CBC results? Hb 11 g/dL, Hct 38%, and an MCV 105 fl . His reticulocyte count is normal. A) Serum ferritin and a peripheral smear B) Hemoglobin electrophoresis C) Serum folate acid and B12 level D) Schilling test

C) Serum folate acid and B12 level Serum folate acid and B12 levels would be ordered to evaluate him for folic acid deficiency anemia. Many patients who are defi cient in folic acid are also defi cient in B12.

Pulsus paradoxus is more likely to be associated with: A) Sarcoidosis B) Acute bronchitis C) Status asthmaticus D) Bacterial pneumonia

C) Status asthmaticus Pulsus paradoxus is most likely to be seen with status asthmaticus. With inspiration, systolic pressure drops due to the increased pressure (positive pressure). Some pulmonary risks of having increased pressure include asthma and emphysema. Cardiac causes for pulsus paradoxus include tamponade, pericarditis, and cardiac effusion.

Carol, a 73-year-old patient, complains of episodic vertigo, slight confusion, and weakness that last nearly an hour each time. Movement does not worsen the vertigo. She "rests" and her symptoms subside, but she is puzzled because the weakness "jumps from side to side," sometimes on the right and sometimes on the left of her body. Her symptoms suggest: A) Benign paroxysmal positional vertigo B) Ménière's disease C) TIA (transient ischemic attack) D) CVA (cerebrovascular accident)

C) TIA (transient ischemic attack) Transient ischemic attack (TIA) is caused by vascular occlusion. Symptoms of a TIA usually last less than 1 hour; however, the symptoms may also be permanent. Common signs/symptoms include neurologic deficits, vertigo, confusion, weakness, hemiparesis, temporary monocular blindness, ataxia, and diplopia.

Which of the following methods is used to diagnose gonorrheal pharyngitis or proctitis? A) Serum chlamydia titer B) Gen-Probe C) Thayer-Martin culture D) Culture and sensitivity of the purulent discharge

C) Thayer-Martin culture

You suspect an enterobiasis infection in a 6-year-old girl. Which of the following tests would you recommend? A) Stool culture and sensitivity B) Stool for ova and parasites C) The scotch tape test D) A Hemoccult test

C) The scotch tape test Enterobiasis infection (pinworms) is caused by small worms that infect the intestines. Symptoms include itching around the anus, which is usually worse at night. The scotch tape test is done by applying the scotch tape on the anal area in the morning; the worms commonly come out at night and will stick to the tape, which is used for diagnosis.

A caregiver is concerned about the risk of falls for her elderly 90-year-old mother. The NP educates the caregiver by explaining the most common fracture resulting from falls include all of the following except? A) wrist B) vertebra C) elbow D) hip

C) elbow The most common fractures resulting from falls are of the wrist, hip, and vertebra. There is a high mortality rate approximately 20% in 1 year in elderly women with hip fractures.

Albert is a 40-year-old male who presents for care. He states he has let his weight go for the last few years because of one thing or the other but is committed to getting into better shape. Today Albert's BMI is 47. All of the following statements about obesity are true except: A: One third of all adults in the U. S. are classified as obese B: Metabolic syndrome includes waist >40 inches for a man C: Obesity is associated with lower all-cause mortality D: Prevention of being obese includes increasing physical activity and dietary modification.

C- Per Papadakis and McPhee, Obesity impacts 1/3 of the U.S. population. Metabolic syndrome consists of waist circumference of greater than or equal to 40(men) or 35(women) inches, triglyceride levels of 150mg/dL, HDL less than 40(men) or 50(women) mg/dL, BP of 130/85mmHg or greater, and fasting blood glucose of 100mg/dL or greater. Obesity is associated with a higher all-cause mortality. Persons whose BMI is 40 or greater have a 52% (men) or 62% (women) death rate from cancer than compared to men and women of normal weight (BMI under 25). To prevent being overweight or obese, increasing physical activity and decreasing caloric intake is the best way to prevent weight gain. Question pool

Typically, a platelet count of less than _________ is considered increased risk for spontaneous bleeding. A. 25,000-35,000 B. 40,000-50,000 C. 10,000-20,000 D. 5,000-10,000

C. 10,000-20,000* The risk of spontaneous bleeding (including petechial hemorrhage and bruising) does not typically increase appreciably until the platelet count falls below 10,000-20,000/mcL, although patients with dysfunctional platelets may bleed with higher platelet counts.

The Family Nurse Practitioner knows that the duration of antimicrobial therapy for uncomplicated cystitis in children should be? A. 3 days of Levofloxacin B. 3-4 days of Trimethoprim-sulfamethoxazole C. 7-10 days of Amoxicillin D. 3 -4 days of a first-generation cephalosporin

C. 7-10 days of Amoxicillin * Short course therapy of cystitis is not recommended in children, because differentiating upper and lower tract disease may be difficult and higher failure rates are reported in most studies of short-course therapy

Which patient would qualify for osteoporosis screening according to the USPSTF? A. A 32 year old Caucasian woman whose mother has osteoporosis B. A 63 year old African American woman who weighs 300 lbs C. A 67 year old Caucasian woman who weighs 105 lbs D. A 40 year old Asian woman who fell and broke her arm

C. A 67 year old Caucasian woman who weighs 105 lbs USPSTF recommends screening women who are 65 or older OR women younger whose fracture risk is equal to or greater than that of a 65-year-old white woman who has no additional risk factors. The Fracture Risk Assessment tool (FRAX) tool can be used to calculate 10-year risk of osteoporotic fracture.

2. During an educational class about sleep disorders in children, you teach the parents that (Choose the most correct answer): A. A child having a nightmare will not be able to describe the event the next day. B. Good sleep hygiene does not have any effect on insomnia. C. A child is often unresponsive and incoherent during a night terror. D. Stress and sleep deprivation have no correlation with sleep disorders.

C. A child is often unresponsive and incoherent during a night terror. Children who have night terrors have no memory of the event the next day and remain unresponsive and incoherent during the event. Children with nightmares can often recall the dream and frightening images the next day. Good sleep hygiene includes limited stimulating activity (i.e. television and video games)—this can help reduce night terrors, nightmares, and reduce insomnia. Stress, sleep deprivation, anxiety, and trauma can increase the frequency of night terrors, nightmares, and insomnia.

Your patient presents with facial hair, wearing cargo pants, and a baseball hat. The patient reports, "I identify as a female and prefer she/her pronouns." You see on the patient's chart that the biological sex states male. You recognize that the patient is ________. A. Cisgender B. Gender-nonconforming C. A transgender female D. A transgender male

C. A transgender female A transgender man is someone with a male gender identity and a female birth-assigned sex; a transgender woman is someone with a female gender identity and a male birth-assigned sex. A non-transgender person may be referred to as cisgender (latin root cis = near/next to) refers to people whose gender identity and birth sex are the same, ie, non-transgender. Non-binary, gender nonconforming, or genderqueer describes a person whose gender identity diifers from that assigned at birth but may be more complex, fluid, multifaceted, or otherwise less clearly defined than purely male or female. Non-binary people may prefer neutral pronouns such as "they," "them," or "their." Gender expression describes the outward manner in which an individual expresses or displays gender, including choices in clothing and hairstyle, speech, and mannerisms. Gender identity and gender expression may differ; for example, a woman (transgender or non-transgender) may have an androgynous appearance, or a man (transgender or non-transgender) may have a feminine form of self-expression

A 6-year-old male is brought to the clinic by his mother for flank pain, fever and dysuria. He was diagnosed and treated for strep throat 2 weeks ago. Urinalysis shows microhematuria with leukocytes. As the nurse practitioner, you suspect he has what diagnosis? A. Kidney stones B. Pediatric Nephritis C. Acute postinfectious-glomerulonephritis D. UTI

C. Acute postinfectious-glomerulonephritis Acute post infectious glomerulonephritis is most common in kids 4 - 12 years of age with a recent history of group A beta-hemolytic streptococcal infection. Clinical manifestations include: asymptomatic microhematuria to gross hematuria, edema, and hypertension. There is no specific treatment. Treatment is usually supportive and geared toward potential complications.

A 10-month-old girl is brought in to the clinic by her parents. The parents report the patient's symptoms over the last 24 hours to be: fever, irritability, poor food and milk intake, and she vomited three times. She has not had a wet diaper in the last eight hours. You confirm a UTI. What is your next action? A. Treat her with Amoxicillin. B. Send her for a renal ultrasound. C. Admit her to the hospital for treatment. D. Wait for the urinary culture results and treat with appropriate antimicrobial.

C. Admit her to the hospital for treatment. Rationale: Infants and children who show signs of dehydration or toxicity should be hospitalized for IV antimicrobials and hydration. Amoxicillin is a good choice for the treatment of uncomplicatedUTIs in children. The results of a urinary culture should be used to determine if the initial antimicrobial choice should be adjusted. A renal ultrasound is recommended for infants and children with UTIs as congenital abnormalities of the urinary tract may be the cause of their UTI. Question pool

A 62 year old female presents with complaints of new onset rectal pain and small amounts of bright red blood on her stool and toilet paper after each bowel movement following a recent bout with constipation. Physical examination of the anus reveals a midline, posterior ulcerated lesion. The family nurse practitioner knows these symptoms are most likely: A. Syphilis B. Crohn's Disease C. Anal Fissure D. Anal Carcinoma

C. Anal Fissure Anal Fissures are painful ulcerations stemming from trauma related to straining or constipation. They are most commonly posterior midline with minimal amounts of blood on the stool or toilet paper following bowel movements. Diagnosis confirmed by visual exam of anus. A fissure occurring off the midline would raise suspicion for syphilis, Crohn's disease, or anal carcinoma

During a newborn exam, you note that the infant's tongue movement is restricted and attached tightly to the mouth floor. This condition is: A. Oligodonitia B. Periodontitis C. Ankyloglossia D. Malocclussion

C. Ankyloglossia Hay identifies ankyloglossia as a tight, close attachment of the tongue to the mouth floor that may result in restricted movement of the tongue

2. A 10-year-old child presents to the clinic for abdominal pain with her father who has stepped out to use the bathroom. The NP notices bruises along the child's abdomen that are in different stages of healing and noticeable belt marks along her torso. The child tells the NP that she was hit by her father when he got home from the bar last night and that this sort of thing happens often. Which of the following is NOT the appropriate action to take in this situation. A. Consult with a multidisciplinary child protection team. B. Report to child protective services the findings you have collected. C. Ask the father in the waiting room to validate the child's story. D. Order an abdominal CT to rule out abdominal injury.

C. Ask the father in the waiting room to validate the child's story. * Always notify the appropriate officials, such as child protective services, when abuse is suspected. Never confront the suspected abuser as this could put the medical team in danger. Order a CT of the abdomen to rule out possible injury

When considering treatment for bronchiolitis, all of the following treatments are recommended except.: A. Oxygen, if hypoxemia is present B. Frequent suctioning C. Bronchodilators D. Anitpyretics

C. Bronchodilators* Bronchiolitis has the following symptoms: "1-2 days of fever, rhinorrhea, and cough, followed by wheezing, tachypnea, and respiratory distress" (Hay, Levin, Deterding, and Abzug, 2014, p.503). For the treatment of bronchiolitis, oxygen is acceptable if the patient is hypoxemic (Hay et al., 2014, p. 530). Supportive treatment such as frequent suctioning, and adequate hydration are recommended (Hay et al., 2014, p. 530). "Bronchodilators and corticosteroids have not been shown to change the severity or the length of the illness in bronchiolitis and therefore not recommended" (Hay et al., 2014, p. 530). Although Antipyretics and fever control are not explicitly stated, one can conclude that this is necessary for treatment.

You are treating a diabetic patient for mild hypertension. Which medication would you avoid prescribing? A. Amlodipine B. Benazepril C. Carvedilol D. Irbesartan

C. Carvedilol Beta-blockers can mask the signs of hypoglycemia and should be used cautiously in these patients. Beta-blockers can also induce or exacerbate bronchospasms, therefore, should be used cautiously in those with asthma.

A 36 year old African American female was recently diagnosed with peptic ulcer disease (PUD). The family nurse practitioner (FNP) has decided to treat the patient with the Standard Bismuth Quadruple Therapy. Which of the following medications is not part of the Standard Bismuth Quadruple Therapy when treating PUD? A. Tetracycline B. Proton Pump Inhibitor (PPI) C. Clarithromycin D. Metronidazole

C. Clarithromycin One of the treatment options for PUD is the Standard Bismuth Quadruple Therapy which consists of: PPI BID, Bismuth subsalicylate two tabs QID, Tetracycline QID, and Metronidazole TID. Clarithromycin is part of the Standard Nonbismuth Quadruple Therapy and the Standard Triple Therapy.

A patient comes into your clinic complaining of substernal chest pain and difficulty swallowing both foods and liquids. His spouse relates that this has been going on for years and she notices him frequently throwing his shoulders back while eating. What test will confirm your suspected diagnosis? A. Chest X-ray B. Electrocardiogram C. Esophageal manometry D. Flex Sigmoidoscopy

C. Esophageal manometry Achalasia is a disorder of motility with hallmark loss of peristalsis in the lower esophagus and dysfunctional relaxation of the lower esophageal sphincter. Signs and symptoms include gradual difficulty swallowing of solid foods and liquids. These symptoms can be present for years and can be associated with substernal chest discomfort after eating. Patients with achalasia tend to eat slower and compensate by lifting the neck or throwing the shoulders back to allow food to empty from the esophagus. Diagnosis of achalasia is confirmed with esophageal manometry.

What pain scale would be the most appropriate when assessing a non-verbal 2-year-old? A. Numeric Scale B. Wong-Baker Scale C. FLACC scale D. NIPS

C. FLACC scale The Numeric and Wong-Baker scale are self-report and the patient is non-verbal. NIPS is for neonates under the age of one. FLACC is for non-verbal children over the age of one which would be most appropriate for this patient.

Mr. Ford, a white 68-year-old man, presents to your clinic with the complaint of the inability to discriminate between voices when in a noisy environment like a restaurant. You recognize he is most likely suffering presbyacusis. Which statement would NOT be a possible cause of Mr. Ford's hearing loss? A. Noise trauma B. He may have inherited this from his parents C. Frequent ear infections as a child D. Over exposure to certain medications

C. Frequent ear infections as a child Presbyacusis is caused by various factors including prior noise trauma, drug exposure, or genetic predisposition. Most patients notice a loss of speech discrimination that is especially pronounced in noisy environments. About 25% of people between the ages of 65 and 75 years and almost 50% of those over 75 experience hearing difficulties.

Which sexually transmitted disease is not correctly matched to the corresponding diagnostic laboratory test? A. Syphilis (Treponema pallidum) - RPR B. Chlamydia (C trachomatis) - NAAT C. Gonorrhea (N gonorrhoeae) - VDRL D. Human Papillomavirus - Papanicolaou test

C. Gonorrhea (N gonorrhoeae) - VDRL* The correct answer is C - Gonorrhea (N gonorrhoeae) - VDRL. The correct diagnostic lab tests to confirm a gonorrheal infection is a culture or a NAAT that can either be a urine sample obtained from males or females, or a vaginal swab or cervical swab obtained from female patient

Lifestyle modifications used to manage hypertension include all of the following except: A. Weight Reduction B. Increase physical activity C. Increase NA+ intake D. Limit alcohol and quit smoking

C. Increase NA+ intake * Decreasing alcohol consumption and smoking, as well as increasing physical activity and weight reduction are all non-pharmacological ways to decrease blood pressure. Also, including the DASH diet and a decrease in sodium intake help achieve lowering blood pressure.

A 62-year-old white female presents to the walk-in clinic today with complaints of acute onset of nausea, vomiting, and diarrhea. While she has vomited twice and has had multiple episodes of diarrhea in the last 12 hrs after eating at a local restaurant, it is the continuous queasiness that she cannot tolerate. You obtain a thorough history including a list of her current daily medications: Atorvastatin 40 mg po q hs, Lisinopril/Hctz 10mg/12.5 mg po q am, ASA EC 81 mg po q hs, sotalol 80 mg po BID. When prescribing antiemetics for her today, which of the following would place her at greatest risk of torsades de pointes? A) Diphenhydramine B) Dexamethasone C) Ondansetron D) Prochlorperazine

C. Ondansetron (Zofran) While Serotonin 5-HT3 receptor antagonists such as Ondansetron are effective in reducing nausea and vomiting stimulated by the GI tract, they also have the potential to cause QT interval prolongation and thus, concurrent use with Sotalol should be avoided.

All of the following are signs & symptoms that are indicative of Acute Pancreatitis except ? A. Epigastric pain without guarding, rebound tenderness, or distention B. Elevated amylase and lipase C. Pain improves with walking D. Pain is worse while supine

C. Pain improves with walking Acute pancreatitis presents with epigastric pain worse with walking or lying supine. Pain is improved by sitting and leaning forward. Amylase and Lipase are usually 3x higher than normal and depending on severity of disease, can return to normal within 24 hours. Lipase usually remains elevated longer and is more specific to diagnosing Acute Pancreatitis.

Which of the following is important for the NP to remember when using technology for patient care? A. Patient initiated email implies consent to discussing confidential data via email. B. Instant messaging is a convenient, secure method for interacting with patients. C. Posting online recommendations for patient care could be seen as formal medical advice. D. Telemedicine is a great option because it is not limited by issues with liability, malpractice, and licensure.

C. Posting online recommendations for patient care could be seen as formal medical advice.* *Healthcare recommendations and patient educational materials that are posted online could be viewed as medical advice and should contain a statement explaining patients should always consult with their healthcare provider before initiating medical or lifestyle changes.

A 5-year-old patient that you have been following has had a prior history of retrograde flow of urine with history of hydronephrosis twice. Her mother asked what they can do to avoid any future problems. A. Referral to urology for surgery B. Lifetime prophylactic Bactrim is required C. Prevention of UTIs and Immediate treatment upon onset of symptoms D. Self-catheterization is necessary to empty bladder entirely

C. Prevention of UTIs and Immediate treatment upon onset of symptoms Appropriate management of reflux nephropathy includes prevention and prompt treatment of UTI (Hay, Levin, Deterding, & Abzug, 2016). Surgery may not be required as low-grade VUR resolves over time. While it may be necessary to prescribe prophylaxis antibiotics, lifelong is not required. Self-catheterization is not recommended and not a treatment for VUR.

Your 11w pregnancy patient presents for her Nucal Translucency ultrasound with complaint of nausea, vomiting, and fatigue. She has lost 6lbs since her intake appointment 4 weeks ago. What is the first-line choice anti-emetic for this patient? A. Ondansetron ODT B. Promethazine PR C. Pyridoxine PO D. Metoclopramide PO

C. Pyridoxine PO First-line treatment is Pyridoxine (B6) with or without doxylamine. If the patient is still unable to tolerate adequate PO intake, metoclopramide or promethazine may be warranted. Ondansetron is associated with increased risk of congenital anomalies.

The following conditions or risk factors are indication for early blood pressure monitoring in a child except: A. Hydrocephalus B. Tetralogy of Fallot C. Topical steroid treatment of eczema D. Family history of polycystic kidney disease

C. Topical steroid treatment of eczema Rationale: Indications for early blood pressure monitoring at well child exams should begin before age 3 years in instance of family history of renal disease, congenital heart defect, and elevated ICP, among others. *Systemic corticosteroids are considered a risk factor, but not topical agents*

What type of tremor would the Nurse Practitioner most likely see in a recently diagnosed patient with Parkinson's disease? A. Bilateral action tremor B. Unilateral action tremor C. Unilateral resting tremor D. Bilateral resting tremor

C. Unilateral resting tremor Although the tremor may be present in all limbs, the tremor is commonly confined to one limb or to the limbs on one side for months or years before it become more generalized. A variable resting tremor is observable

3 y/o WF arrives to clinic in acute respiratory distress. Patient has a loud barking cough with sternal retractions and stridor noted on physical exam. No wheezing nor drooling observed. Mother denies nausea/vomiting/fever and patient is afebrile at today's visit. This patient is exhibiting typical features of: A. Asthma Attack B. Epiglottitis C. Viral Croup D. Bronchiolitis

C. Viral Croup All of the options could cause the patient to present to clinic in respiratory distress. Option A - patients with asthma attack symptoms will present in respiratory distress, but they typically have wheezing noted. This patient had stridor noted upon auscultation, not wheezing. Option B - patients with Epiglottitis present in respiratory distress, but usually present with a high fever, drooling, and retractions as cardinal symptoms. This patient was afebrile and was not drooling. Option D - patients that present with Bronchiolitis present with wheezing and crackles on auscultation, vomiting, and coughing. Option C - Viral Croup is the best choice based on the loud barking cough, stridor, and sternal retractions. The absence of wheezing ruled out asthma and bronchiolitis, and the patient lacked cardinal signs of epiglottitis, so that choice was also eliminated.

Vegetarians who steer clear of meat, fish, and dairy should be advised of their risk for a which dietary vitamin deficiency? A.) A B.) C C.) B12 D.) E

C.) B12 With vitamin B12 being present in animal origin foods, a dietary vitamin B12 deficiency is rare but seen in vegans. More commonly in strict vegetarians who avoid all meat, fish and dairy.

Cephalosporins

CEPHALOSPORINS (Category B) 1st Generation: cephalexin (Keflex) 2nd Generation: cefprozil (Cefzil), cefuroxime (Ceftin), cefaclor (Ceclor). 3rd Generation: IM ceftriaxone (Rocephin), cefdinir (Omnicef), cefixime (Suprax) NOTE:Penicillin-allergic patients may be allergic to cephs. (cross-sensitivity). All antibiotics can cause Clostridium difficile overgrowth, pseudomembranous colitis,CDAD. Used for: strep throat, cellulitis, pneumonia, sinusitis, otitis media, secondary infection COPD, GC (Suprax, Rocephin)

Rate control for atrial fibrillation

Calcium-channel blockers, beta-blockers, or digoxin. Leik

How do warfarin and sulfa interact?

Can increae INR and bleeding risk Leik

Pityriasis Rosea - Cause, presentation and treatment

Cause unknown Presentation - Oval lesion with fine scales (possibly herald lesion) or christmas tree pattern. Self-limiting (4 - 8 weeks) and asymptomatic. no medications.

S4 heart sound

Caused by increased resistance due to a stiff left ventricle; usually indicates LVH; considered a normal finding in some elderly (slight thickening of left ventricle). ■ S4 occurs during late diastole (also called an "atrial gallop" or "atrial kick"). ■ Sounds like "Tennessee." ■ Best heard at the apex or apical area (mitral area) using the bell of the stethoscope. Leik

Best treatment for dog or cat bites?

Clean wound thoroughly Abx prophylaxis with ß-lactam abx + ß-lactamase inhibitor = Amoxicillin-clavulunate (Augmentin)

​Dupuytren's contracture

Condition in which one or more fingers become permanently bent in a flexed position. You may feel small hard nodules under the skin of the palm. It worsens over time until the fingers can no longer be straightened. Tx: steroid injections, PT/OT Leik

A 25-year-old healthy adult is diagnosed with atypical pneumonia by the NP. The patient reports a history of nausea, upset stomach, and vomiting with erythromycin. The patient is complaining of a sore throat. The vital signs are temperature of 99°F, pulse of 80/minute, and respiratory rate of 12 breaths/minute. What is the most appropriate treatment plan for this patient? A) Initiate a prescription of azithromycin (Z-Pack) PO × 5 days. B) Initiate a prescription of trimethoprim-sulfamethazole (Bactrim) 1 tab PO BID × 10 days. C) Order a chest x-ray with the anterior-posterior and lateral views. D) Order a sputum for culture and sensitivity.

Correct answer is Option A. (It usually does not cause GI side effects, has fewer drug interactions, and a broader spectrum of activity.) Best Clues + Because the patient has a definite diagnosis of atypical pneumonia, ordering additional tests will not change the treatment outcome (rule out Options C and D). + Bactrim is not effective against Mycoplasma or Chlamydia bacterial infections (rule out Option B), but it is an excellent drug for certain gram-negative infections. + For community-acquired pneumonia, ordering a sputum for C&S is not recommended (rule out Option D). Leik

A 16-year-old high school athlete is returning for follow-up for a severe sore throat. The test result reveals a positive throat culture for Group A beta hemolytic strep and a positive Monospot test (heterophile antibody test). What is the best initial clinical management of this patient? A) Initiate a prescription of amoxicillin 500 mg PO TID × 10 days. B) Initiate a prescription of penicillin VK 250 mg PO QID × 10 days. C) Order an Epstein-Barr virus (EBV) titer to determine whether the patient has an acute or a reactivated mononucleosis infection. D) Write a prescription for an abdominal ultrasound to determine the size of the patient's liver and spleen.

Correct answer is Option B. Best Clues ■ Positive test results (Monospot and strep). ■ Rule out: - Option A (avoid using amoxicillin due to high risk of a "drug rash" that is not due to an allergy). - Option C (not for initial management; strep infection must be treated first before ordering labs). - Option D (not for initial management; only for cases where the PE reveals an enlarged liver and/or spleen). Leik

Treatment for otitis externa

Cortisporin Otic drops. (OE commonly caused by Pseudomonas)

A 4-year-old patient presents to the clinic with his father because he has dry eyes, irritated eyelids, and his eyelashes have been crusted over every morning x 4 days. On exam, bilateral eyes are erythematous. How should the NP treat this? a. oral antibiotics b. observe since this is likely a virus c. refer d. suggest cleaning eyes with baby shampoo

D - Treatment for blepharitis includes lid scrubs with baby shampoo, warm compresses to the eyelids, and application of a topical antibiotic ointment.

You are educating your Chronic Renal Failure patients about the importance of controlling diabetes and hypertension and balancing minerals in their diet. Studies have shown that cardiovascular disease increases mortality in the early stages of CKD. For these reasons, you advise them to: A Increase the amount of phosphorus on the diet B Restrict the amount of vitamin D on the diet C Restrict the amount of calcium on the diet D Restrict the amount of phosphorus on the diet

D Restrict the amount of phosphorus on the diet Elevated levels of phosphorus are related with an increased risk of cardiovascular diseases and early mortality in CKD through ESRD.

A 16-year-old teenager with a history of ADHD is brought in to the ED by his mother. She does not want her son to be alone in the room. The NP doing the intake notes several burns on the teen's trunk. Some of the burns appear infected. The NP documents the burns as mostly round in shape and about 0.5 cm (centimeter) in size. Which of the following questions is most appropriate to ask the child's mother? A) "Your son's back looks terrible. What happened to him?" B) "Does your son have more friends outside of school?" C) "Did you burn his back with a cigarette?" D) "Can you please tell me what happened to your son?"

D) "Can you please tell me what happened to your son?" Best Clues 1) Option D is the only open-ended question in the group. 2) In addition, it is not a judgmental statement. Leik

Treatment for mild preeclampsia includes all of the following except: A) Bed rest except for bathroom privileges B) Close monitoring of weight and blood pressure C) Close follow-up of urinary protein, serum creatinine, and platelet count D) A prescription of methyldopa (Aldomet) to control blood pressure

D) A prescription of methyldopa (Aldomet) to control blood pressure Recommended care for women diagnosed with preeclampsia includes bed rest with bathroom privileges, weight and BP monitoring, and closely following urine protein and serum protein, creatinine, and platelet counts. Oral medications are not used as first-line treatment.

A 35-year-old sexually active male presents with a 1-week history of fever and pain over the left scrotum. It is accompanied by frequency and dysuria. The scrotum is edematous and tender to touch. He denies flank pain, nausea, and vomiting. He reports that the pain is lessened when he uses scrotal support briefs. His urinalysis shows 2+ blood and a large number of leukocytes. What is the most likely diagnosis? A) Acute urinary tract infection B) Acute pyelonephritis C) Acute orchitis D) Acute epididymitis

D) Acute epididymitis Acute epididymitis is the infection presented here. Scrotal edema and pain with palpation do not occur in UTI or pyelonephritis. Acute orchitis symptoms include testicular pain and edema, are usually associated with the mumps, but do not have frequency and dysuria.

Prophylaxis for Pneumocystis carinii pneumonia includes all of the following drugs except: A) Trimethoprim-sulfamethoxazole B) Dapsone C) Aerosolized pentamidine D) Aerosolized albuterol sulfate (Ventolin)

D) Aerosolized albuterol sulfate (Ventolin) Prophylaxis for Pneumocystis carinii pneumonia includes the use of Bactrim, dapsone, and aerosolized pentamidine. Aerosolized pentamidine is a antimicrobial treatment for prevention, along with Bactrim and dapsone, which are antibacterial medications.

Jim Wheeler is obese (BMI of 33), fatigued, and complaining of excessive thirst and hunger. You suspect type 2 diabetes mellitus. Initial testing to confirm diagnosis can include: A) Fasting plasma glucose level B) Glycosylated hemoglobin level (HbA1 c) C) Glucose tolerance testing D) All of the above

D) All of the above Type 2 diabetes mellitus screening tests include: fasting plasma glucose level (> 126 mg/dL), random plasma glucose level (> 200 mg/dL), and glucose tolerance testing (2 hr blood glucose level > 200 mg/dL) with 75 g glucose load. Normal HbA1c levels are < 6%.

A 57-year-old male walks into an urgent care center. The patient complains of an episode of chest pain in his upper sternum that is relieved after he stops the offending activity. He has had several episodes of the chest pain before. A fasting total lipid profile is ordered. The result reveals total cholesterol of 180 mg/dL, an LDL of 120 mg/dL, and a high-density lipoprotein (HDL) of 25 mg/dL. Which of the following is most likely? A) Acute esophagitis B) MI C) GERD D) Angina

D) Angina Best Clues 1) Classic presentation (chest pain that is precipitated by exertion and is relieved by rest) 2) History (several episodes of the same chest pain) 3) Positive risk factors (low HDL, elevated lipid levels, age, and gender) Notes All four answer options are some common conditions that can mimic angina (differential diagnoses). Rule out the pertinent negatives. 1) Pain is relieved by rest (angina). If pain not relieved by rest, then rule out angina. 2) Presence of risk factors for heart disease and chest pain (angina, MI). 3) Physical activity aggravates condition (angina, MI). 4) Lack of history of aggravating factors such as intake of certain meds such as NSAIDs, aspirin, bisphosphonates, or alcohol (rule out esophagitis). 5) Chest pain is not related to meals (rule out GERD). Leik

Which of the following is used to confirm a diagnosis of Hashimoto's thyroiditis? A) Serum TSH B) Free T4 test C) Antimicrosomal antibody test D) Any of the above

D) Any of the above In addition to conducting a physical examination and taking a thorough history and symptoms into account, 1 or more laboratory tests are used to diagnose Hashimoto's thyroiditis. The 3 most common diagnostic tests that detect this common thyroid disorder are: serum thyroid-stimulating hormone test (TSH), anti-thyroid antibodies tests, and the free T4 hormone test.

All of the following are not recommended for the outpatient treatment of mild preeclampsia except: A) Severe sodium restriction B) Restrict fluid intake to less than 1 liter per 24 hours C) Aldomet (methyldopa) 250 mg PO (orally) BID D) Bed rest on the left side with bathroom privileges

D) Bed rest on the left side with bathroom privileges Outpatient treatment for mild preeclampsia includes bed rest on the left lateral side with bathroom privileges.

Which type of hepatitis virus infection is more likely to result in chronic hepatitis and increased risk of developing hepatocellular carcinoma? A) Hepatitis A virus B) Hepatitis B virus C) Hepatitis C virus D) Both hepatitis B and hepatitis C

D) Both hepatitis B and hepatitis C Of the primary hepatitis viruses, only B and C are associated with hepatocellular cancer

Which of the following cranial nerves is evaluated when a wisp of cotton is lightly brushed against the corner of the eye? A) CN II B) CN III C) CN IV D) CN V

D) CN V Cranial nerves are assessed as follows: II (optic): distance vision, near vision . III, IV, VI (oculomotor, trochlear, abducens): EOMs, visual fi elds of gaze. V1 (trigeminal 3 branches: V19 ophthalmic), V2 (maxillary), V3 (mandibular): motor portion, clench jaws; sensory portion, corneal refl ex/facial sensation.

A 55-year-old female with a history of migraine headaches has recently been diagnosed with Stage II hypertension. Her EKG strips reveal second-degree heart block. The chest x-ray is normal. Which of the following drugs should this patient avoid? A) ACE inhibitors B) Angiotensin receptor blockers C) Diuretics D) Calcium channel blockers

D) Calcium channel blockers Common side effects of calcium channel blockers include headaches, edema of the lower extremities, and heart block or bradycardia. Contraindications for calcium channel blockers include second- or third-degree AV block, bradycardia, and congestive heart failure.

All of the following are infections that affect mostly the labia and vagina except: A) Bacterial vaginosis B) Candidiasis C) Trichomoniasis D) Chlamydia trachomatis

D) Chlamydia trachomatis Infections that commonly affect the labia and vagina include bacterial vaginosis, candidiasis, and trichomoniasis. Chlamydia trachomatis commonly affects the cervix, endometrial lining , fallopian tubes, and pelvic cavity.

All of the following drugs interfere with the metabolism of oral contraceptives except: A) Tetracycline B) Rifampin C) Phenytoin (Dilantin) D) Ciprofloxacin (Cipro)

D) Ciprofloxacin (Cipro)

Which of the following is considered a relative contraindication for combined oral contraceptive pills? A) Undiagnosed vaginal bleeding B) A hepatoma of the liver C) Suspected history of TIAs D) Depression

D) Depression Depression is a relative contraindication for combined OCPs due to the hormonal effects that can affect mood. Absolute contraindications include hepatoma of the liver, history of embolic episode, history of TIAs, and undiagnosed vaginal bleeding. OCP should not be considered in these instances due to the high risk factors and health risks.

A 21-year-old woman complains of left-sided pelvic pain accompanied by dyspareunia. During the gynecological exam, the nurse practitioner notices green cervical discharge. The patient mentions a new onset of a painful and swollen left knee and denies a history of trauma. This best describes: A) Septic arthritis B) Reiter's syndrome C) Chondromalacia of the patella D) Disseminated gonorrheal infection

D) Disseminated gonorrheal infection Symptoms of PID with painful, swollen joints of extremities indicate disseminated gonorrheal infection. Untreated disseminated gonorrhea can lead to septic arthritis. Symptoms may be mild from slight joint pain and no fever to severe joint pain with high fever. PID symptoms do not occur with septic arthritis, Reiter's syndrome, or chondromalacia of the patella.

An Rh-negative pregnant woman with negative rubella titers should be vaccinated at what time period in pregnancy? A) She can be vaccinated at any time in her pregnancy B) During the second trimester C) During the third trimester D) During the postpartum period

D) During the postpartum period Rubella should be administered to the woman during the postpartum period. Rubella is contraindicated during pregnancy.

A 14-year-old boy is brought in by his mother for a physical exam. Both are concerned about his breast enlargement. The teen denies breast tenderness. On physical exam, the NP palpates soft breast tissue that is not tender. No dominant mass is noted. The skin is smooth and there is no nipple discharge with massage. The teen has a BMI of 29. Which of the following statements is correct? A) The patient has physiologic gynecomastia and should return for a follow-up exam. B) Order an ultrasound of both breasts to further assess the patient's breast tissue development. C) Reassure the mother that the patient's breast development is within normal limits. D) Educate the mother that her son has pseudo-gynecomastia.

D) Educate the mother that her son has pseudo-gynecomastia. Best Clues 1) The boy is very overweight (BMI 29) and is almost obese. 2) The clinical breast exam does not show palpable breast tissue. Instead, the breast palpation reveals soft fatty tissue. 3) It is wrong to "reassure" a patient or a family member in the exam (poor therapeutic communication technique). Notes 1) Physiologic gynecomastia physical exam findings will show disc-like breast tissue that is mobile under each nipple/areola, the breast may be tender, and the breast can be asymmetrical (one breast larger than the other). 2) A BMI of 25 to 29.9 is considered overweight. Obesity is a BMI of 30 or higher. 3) Overweight to obese males are at highest risk for pseudo-gynecomastia. Leik

Medicare Part B will pay for all of the following services except: A) Outpatient physician visits that are medically necessary B) Durable medical equipment C) Outpatient laboratory and radiology tests D) Eyeglasses and routine dental care

D) Eyeglasses and routine dental care Medicare Part B covers: 1) outpatient physician visits, labs, x-rays; 2) durable medical equipment; 3) mammograms/colonoscopy after age 50 years annually; and 4) rehabilitation.

You would recommend the pneumococcal vaccine (Pneumovax) to patients with all of the following conditions except: A) Sickle cell anemia B) Splenectomy C) Patients infected with HIV D) G6PD deficiency anemia

D) G6PD deficiency anemia

A lipid profi le done on a newly diagnosed hypertensive patient shows a triglyceride level of 650 mg/dL, total cholesterol 240 mg/dL, LDL 145 mg/dL, and an HDL of 35 mg/dL. What is the best intervention for this patient? A) Educate the patient about lifestyle changes that will help lower cholesterol levels B) Initiate a prescription of pravastatin (Pravachol) C) Recommend that the patient exercise at least every other day and avoid eating fatty or fried foods D) Initiate a prescription of nicotinic acid (Niacin, Niaspan)

D) Initiate a prescription of nicotinic acid (Niacin, Niaspan) Niacin is recommended for treatment of high triglyceride and cholesterol levels. Exercise and healthy eating lifestyle is also recommended, but with the high level of triglycerides at 650 mg/dL and total cholesterol 240 mg/dL, niacin is recommended.

Koilonychia is associated with which of the following conditions? A) Lead poisoning B) Beta thalassemia trait C) B12 deficiency anemia D) Iron-deficiency anemia

D) Iron-deficiency anemia Koilonychia is an abnormal shape of the nail. The nailbed is thin, with irregular edges, and curves inward. Koilonychia is commonly associated with iron-deficiency anemia.

The following are acceptable methods of birth control for breastfeeding mothers except: A) Diaphragm with spermicidal gel B) Progesterone-only pills (Micronor) C) Condoms D) Low-dose oral contraceptives with at least 20 mcg of estradiol (Alesse, Lo-estrin)

D) Low-dose oral contraceptives with at least 20 mcg of estradiol (Alesse, Lo-estrin) Low-dose oral contraceptives that contain estradiol are contraindicated for breastfeeding mothers.

Which of the following fi ndings are seen in a patient with folate-deficiency anemia? A) Microcytic and hypochromic RBCs B) Microcytic and normochromic RBCs C) Normal size and color of the RBCs D) Macrocytic and normochromic RBCs

D) Macrocytic and normochromic RBCs

All of the following are clinical eye findings found in some patients with chronic uncontrolled hypertension. Which of the following findings is not associated with this disorder? A) AV nicking B) Copper wire arterioles C) Flame-shaped hemorrhages D) Microaneurysms

D) Microaneurysms "Keith Wagener Barker (KWB) Grades" for uncontrolled hypertension: + Grade 1: generalized arteriolar constriction seen as "silver wiring" and vascular tortuosities; + Grade 2: grade 1 plus irregularly located, tight constrictions known as "AV nicking" or "AV nipping"; + Grade 3: grade 2 plus cotton wool spots and flame-shaped hemorrhages; + Grade 4, grade 3 but with swelling of the optic disk (papilledema). Microaneurysms occur with diabetic retinopathy.

Potential complications of mitral valve prolapse (MVP) include all of the following except: A) Severe mitral regurgitation B) Endocarditis C) Increased risk of stroke and TIA D) Mitral stenosis

D) Mitral stenosis Complications of MVP include mitral regurgitation, endocarditis, and increased risk of stroke and TIAs.

Which of the following findings is associated with thyroid hypofunction? A) Graves disease B) Eye disorder C) Thyroid storm D) Myxedema

D) Myxedema Myxedema is a rare, and sometimes fatal, disease in which the thyroid is severely underactive and causes life-threatening symptoms. These symptoms include low blood pressure, decreased breathing, decreased body temperature, unresponsiveness, and even coma. Graves' disease, thyroid storm, and eye disorder are seen with an overactive thyroid disease known as hyperthyroidism.

A woman at 32 weeks gestation has a positive throat culture for strep pyogenes. She denies allergies but gets very nauseated with erythromycin. Which of the following is the best choice for this pregnant patient? A) Clarithromycin (Biaxin) B) Trimethoprim/sulfamethoxazole (Bactrim DS) C) Ofloxacin (Floxin) D) Penicillin (Pen VK)

D) Penicillin (Pen VK) Pen VK is safe to use for strep throat during pregnancy. Pen VK is a category B medication for pregnancy and lactation.

All of the following conditions are associated with an increased risk for normocytic anemia except: A) Rheumatoid arthritis B) Lupus C) Chronic autoimmune disorders D) Pregnancy

D) Pregnancy During pregnancy, women may experience microcytic, hypochromic anemia due to the dilutional effect of the increased blood volume during the pregnancy.

You are seeing a 55-year-old white male who recently moved from Texas. He presents with complaints of worsening tremors in both hands. He first noticed the subtle shaking ~ 5 yrs ago and they have gotten progressively worse. You notice that he does not exhibit tremors while at rest but seem to be pronounced with minimal activity. The rest of his physical exam is unremarkable. Based on the history and clinical findings, which of the following would be an appropriate choice of treatment? A) Levodopa B) Pramipexole C) Lyrica D) Propranolol

D) Propranolol For essential benign tremors, beta blockers such as Propranolol would be the correct 1st line treatment for this condition as they may reduce symptom severity by up to 50%. While long term treatment is often necessary, immediate release formulas may also be taken on as needed basis, and may be effective when taken prior to stress provoking activity.

June, 24 year old female, with severe anxiety and depression showed up at the clinic without an appointment. You hear her begging the front office staff to see a doctor, a nurse, or anyone. She is breathing excessively, crying, and then you hear her yell, "I'M NUMB WITH TINGLING ALL OVER! I'M GOING TO PASS OUT!" You suspect respiratory alkalosis. How should you treat her condition? A) Call the front office staff from your cell phone and have her leave. There are no available appointments today and she's been seen 3 times already and it's only Tuesday. B) Empty out your lunch from your brown paper sack and tell her to breathe into it. C) Place a non-rebreather mask on June's face and turn on the portable oxygen tank up as high as it will go. D) Reassure the patient, attempt to calm her by talking to her.

D) Reassure the patient, attempt to calm her by talking to her. In acute cases (hyperventilation) there is light headedness, anxiety, perioral numbness, and paresthesias. Arterial blood pH is elevated and Pco2 is low. Serum bicarbonate is decreased in chronic respiratory alkalosis. Treatment is directed towards the underlying cause. In acute hyperventilation syndrome from anxiety, the traditional treatment of breathing into a bag should be discouraged because it does not correct Pco2. Reassurance may be sufficient for the anxious patient, but sedation may be necessary if the process persists.

An 18-year-old waitress is diagnosed with pelvic inflammatory disease (PID). The cervical Gen-Probe result is positive for Neisseria gonorrhoeae and negative for Chlamydia trachomatis. All of the following statements are true regarding the management of this patient except: A) This patient should be treated for chlamydia even though the Gen-Probe for chlamydia is negative B) Ceftriaxone 250 mg IM and doxycycline 100 mg PO BID x 14 days are appropriate treatment for this patient C) Advise the patient to return to the clinic for a repeat pelvic exam in 48 hours D) Repeat the Gen-Probe test for Chlamydia Trachomatis to ensure that the previous test was not a false-negative result

D) Repeat the Gen-Probe test for Chlamydia trachomatis to ensure that the previous test was not a false-negative result

A male nursing assistant who works in a nursing home is complaining of multiple pruritic rashes that have been disturbing his sleep at night for the past few weeks. He reports that several members of his family are starting to complain of pruritic rashes. On physical examination, the NP notices multiple small papules, some vesicles, and maculopapular excoriated rashes on the sides and the webs of the fingers, on the waist, and on the penis. Which of the following is the most likely diagnosis? A) Scarlatina B) Impetigo C) Erythema migrans D) Scabies

D) Scabies Best Clues 1) The history (pruritic rashes disturb sleep at night, several family members with same symptoms, and works in a higher-risk area such as nursing homes) 2) Classic location of the rashes (finger webs, waist, penis) Notes 1) Assume that a patient has scabies if excoriated pruritic rashes are located in the finger webs and the penis until proven otherwise. Higher-risk groups are health care givers or any person working with large populations such as schools, nursing homes, group homes, or prisons. 2) The usual recommendation is that all family members and close contacts be treated at the same time as the patient (spread by skin-to-skin contact). Wash used clothes/sheets in hot water, then dry or iron in high heat. Leik

Ken has type 2 diabetes mellitus and a "sensitive stomach." Which medication is least likely to cause him gastrointestinal distress? A) Naproxen sodium (Anaprox) B) Aspirin (Bayer's aspirin) C) Erythromycin (E-mycin) D) Sucralfate (Carafate)

D) Sucralfate (Carafate) Naproxen sodium, aspirin, and erythromycin all have gastrointestinal side effects. Sucralfate is used to protect the stomach lining by building a protective layer over the stomach lining; it allows healing to occur.

The Romberg test is done to check for problems with balance. Which area of the brain is responsible for balance? A) Frontal lobe B) Temporal lobe C) The midbrain D) The cerebellum

D) The cerebellum

Which of the following is a true statement regarding the effect of aspirin on platelet function? A) The effect on platelets is reversible B) The effect on platelets is reversible and lasts only 1 week C) It has a minimal effect on platelet function D) The effect on platelet function is irreversible and lasts 15 to 20 days

D) The effect on platelet function is irreversible and lasts 15 to 20 days

A 16-year-old male presents to a community clinic accompanied by his grandmother, who reports that the patient fell off his bike this morning. The patient now complains of a headache with mild nausea. The patient's grandmother reports that he did not wear a helmet. The health history is uneventful. Which of the following statements is indicative of an emergent condition? A) The patient complains of multiple painful abrasions that are bleeding on his arms and legs. B) The patient complains of a headache that is relieved by acetaminophen (Tylenol). C) The patient makes eye contact occasionally and answers with brief statements. D) The patient is having difficulty with following normal conversation and answering questions.

D) The patient is having difficulty with following normal conversation and answering questions. Best Clues 1) History of recent trauma that is followed by a headache with nausea. 2) The patient did not wear a bicycle helmet. NOTES: 1) Any recent changes in LOC, even one as subtle as difficulty with normal conversation, should ring a bell in your head 2) Notice the words "normal conversation." Do not overread the question and ask yourself what they mean by "normal conversation." Take it at face value. 3) Changes in LOC on the test are usually subtle changes. Signs to watch for: has difficulty answering questions, has slurred speech, seems confused, does not understand instructions/conversation, is sleepy/lethargic, and so forth. 4) Even though the patient is bleeding, note that he has "abrasions," which are superficial. 5) The behavior under Option C is considered "normal" for an adolescent male (or female). Leik

The following statements are true about Wilms tumor except: A) The most frequent clinical sign is a palpable abdominal mass B) It is a congenital tumor of the kidney C) Microscopic or gross hematuria is sometimes present D) The tumor commonly crosses the midline of the abdomen when it is discovered

D) The tumor commonly crosses the midline of the abdomen when it is discovered

A fracture on the navicular area of the wrist is usually caused by falling forward and landing on the hands. The affected wrist is hyperextended to break the fall. The nurse practitioner is aware that all of the following statements are true EXCEPT: A) It has higher rate of nonunion compared with the other bones in the wrist when it is fractured B) The fracture frequently does not show up on an x-ray film when it is taken immediately after the injury C) The x-ray film will show the fracture if the fi lm is repeated in 2 weeks D) These fractures always require surgical intervention to stabilize the joint

D) These fractures always require surgical intervention to stabilize the joint A fracture of the navicular area of the wrist occurs when one tries to break a fall with the hands, thus hyperextending the wrist. Immediately following the injury, the fracture may not show on x-ray; however, fracture is usually visible on x-ray if the x-ray is repeated in 2 weeks. The need for surgery is dependent on the extent of the fracture.

Which of the following medications is used for prophylaxis for migraines? A) Cafergot B) Rizatriptan C) Fioricet D) Topiramate

D) Topiramate Abortive treatment for migraines: ergotamines, triptans, prochlorperazine. Prophylaxis treatment for migraines: topiramate, valproic acid, botulinum toxin A, amitriptyline, candesartan.

A 55-year-old white male comes to clinic complaining of a right sided 7/10 "throbbing" intermittent headache occurring at night that lasts 20 minutes for the past 3 weeks. Denies photophobia, vision changes, nausea or history of migraines. The headache is relieved with 100% 12L O2 via face mask. The patient asks for a medication for prophylaxis, so the FNP orders/prescribes what? A) Cafergot 1mg PO daily B) Zolmitriptan 5mg PO daily C) Botulinum toxin Type A injection into head & neck muscles at visit D) Topirimate 100mg PO daily

D) Topirimate 100mg PO daily Cluster headaches occur in middle-aged men who normally do not have a h/o migraines. Pt's have unilateral headaches that typically occur at night and bouts can last from 4-8 weeks. Cluster headaches are relieved with high dose oxygen. Medication used for prophylactic treatment of cluster headaches includes lithium, verapamil and topirimate. Oral triptans are usually not effective, but intranasal or injectable triptans can be used

Jack is a 67 year old male. He has a history of chronic kidney disease & hypertension. Current medications are Lisinopril and newly prescribed Spironolactone after a recent ER visit. The patient reports feeling weak all over and especially in his legs. What electrolyte imbalance do you expect to find when you receive results from his lab work? A) hypokalemia B) hypernatremia C) hypocalemia D) hyperkalemia

D) hyperkalemia Inhibitors of renin-angiotensin-aldosteron axis (ie ACE, ARB, and spironolatone) and potassium sparing diuretics (eplerenone, triamterene) should be used cautiously and laboratory monitoring should be performed within one week.

A 10 year old boy presents to for a well child exam. His BMI today is in the 86th percentile. Previous BMI has been under 75th percentile for the last 3 years. Based on this BMI, how should the nurse practitioner proceed? A. This issue does not need to be addressed. Do not discuss weight so that the child does not develop poor self-esteem. Give the mother a pamphlet on decreasing sugar intake and play outside more than video games. B. Due to severe obesity, a referral to a physician so that they can be prescribed weight loss medications or surgery is indicated. C. The mother should be given strict diet and exercise regimen to enforce. This may differ from the diet from the rest of the family. Cut out all sweets and snacks from the child's regular diet and have set meal times. D. Ask the child his favorite foods and outside activities. Discuss healthy alternatives or things to add to meals. Encourage a team sport or exercise activity of the child's preference. Limit snacks and desserts for special occasions and not entirely restricted. Offer healthy eating resources but encourage whole family wellness and exercise.

D. The child in this scenario is overweight (> 85th percentile), thus it warrants addressing it with the parent and child. Children mostly model their parent's habits, so encouragement of family wellness over child-only interventions is paramount. Discourage parents letting a child "graze" and have open access to the kitchen. The first step of intervention is an emphasis on lifestyle modification through motivational interview, helping to add to compliance. Extreme diets and strict elimination of all treats decreased compliance and can cause body shame. Only in the case of severe obesity (over the 99th percentile) is referral for weight loss medications or surgeries needed

A parent brings her 6-year-old daughter to the clinic with concerns that she has been sexually abused. She states that she was gone for the weekend and when she returned, her daughter was complaining of pain in her genital area. The mother was alarmed with what she saw and brought her daughter immediately to the clinic. The child denies inappropriate contact. She admits to taking a bath in her mother's bathtub and slipping when trying to get out. Physical exam reveals no physical signs of trauma. Genital exam reveals unilateral ecchymosis to the left labia majora extending to the mons pubis. There are no signs of penetrating injuries. Based on the history and physical exam findings, the most likely diagnosis is: A. Labial adhesions B. Inguinal lymphadenopathy C. Sexual abuse D. Straddle injury

D. * Straddle injury straddle injuries can be misdiagnosed as sexual abuse. Having knowledge of those conditions is important so misdiagnosis can be avoided.

The nurse practitioner knows that the patient with OCD taking clomipramine, who has reached a dose of 50 mg/day should have their plasma level drawn after A. 1-2 days B. 3 months C. 1 year D. 2-3 weeks

D. 2-3 weeks Plasma levels of clomipramine and its metabolite should be checked in 2-3 weeks after a dose of 50 mg/d has been reached, with plasma levels kept below 500 ng/ml to avoid toxicity

When starting a patient on oral Warfarin (Coumadin) therapy for anticoagulation, the NP knows it takes on average _____ days for coagulation factors to stabilize? A. 10 B. 2 C. 7 D. 5

D. 5 An average of 5 days is required to achieve a steady-state reduction in the activity of vitamin K-dependent coagulation factors.

Blood pressure screening should occur in all the following pediatric patients except: A. A healthy 3-year-old during his annual well child check. B. A 12-month-old who spent her rst month of life in the NICU. C. An 18-month-old with renal parenchymal disease. D. A 2-year-old with a BMI in the 80th percentile.

D. A 2-year-old with a BMI in the 80th percentile. Blood pressure screening should begin at age 3 for healthy children at their well-child annual exam. However, blood pressure screening should occur earlier in the following circumstances: neonatal complication requiring NICU stay, repaired or non-repaired CHD (congenital heart disease), recurrent UTIs, renal disease or malformations, organ and bone marrow transplants, elevated ICP, and pharmacological treatment that raises BP

Which of the following individuals would benefit from a one-time ultrasound screening for abdominal aortic aneurysm (AAA)? A. A 50-year-old female who has a nightly vodka tonic. B. A 60-year-old female who has HTN and HLD. C. An 80-year-old male smoker with a recent CVA. D. A 66-year-old male with a 30-pack-year history of smoking.

D. A 66-year-old male with a 30-pack-year history of smoking. Females do not seem to benet from AAA screening. However, males, aged 65 to 75, do benefit from a one-time AAA ultrasound screening. Screening reduces AAA-related deaths by almost half in this male demographic. It is especially important to screen male smokers over 65 as smoking increases AAA risk. Abdominal aortic aneurysm screening is a cost-effective means to drastically reduce disease morbidity and mortality

All the following findings require hospitalization in a 38-year-old female patient with an eating disorder except... A. Oral temperature of 96.4 degrees Fahrenheit. B. A resting HR of 38 beats per minute C. A potassium of 2.8 D. A QTc of 400 milliseconds

D. A QTc of 400 milliseconds Inpatient treatment for eating disorders is aimed at stopping weight loss and stabilizing hemodynamics. The body is in a catabolic state and electrolyte imbalances are common. A potassium of 2.8 is a dangerous finding. It can result in tachyarrhythmias and lead to death. Autonomic dysfunction such as HR less than 45 beats per minute and hypothermia are findings that require inpatient services. A QTc of 400 milliseconds is a normal finding. Prolonged QTc is a criterion for inpatient treatment of eating disorders. To answer this question, you must know that a concerning QT intervals is greater than 450 milliseconds.

55y/o AAF presented to clinic for a routine follow up. She denies any complaints today and her medical hx includes CAD/DM/Hyperlipidemia. Based on this patient's medical history, which class of medications would you prescribe to reduce her risk for cardiovascular event? A. Diuretic B. Beta Blocker C. Benzodiazepine D. ACE Inhibitor

D. ACE Inhibitor Option A - Diuretic may be needed for this patient, but ACEI and BB are both better med selections for DM, Hyperlipidemia, and CAD dx. Option B - Beta Blocker is 2nd best choice for this patient and is used as therapy/adjunct therapy for treatment of CAD. Option C - Benzodiazepine medications are not indicated for CAD and DM dx. Option D - ACE Inhibitor is the best medication choice for this patient due to evidence-based trials that prove that ACEI reduce mortality rates with cardiovascular events. Trials also demonstrated reduction in strokes with DM/CAD patients on ACEI.

A 58 year-old-male presents with nasal congestion, clear nasal drainage, and mild cough for the last 3 days. The patient denies fever. Upon exam, the NP finds red, swollen turbinates. What is the best diagnosis for this patient? A. Acute Bacterial Rhinosinusitis B. Fungal Sinusitis C. Nasal Vestibulitis D. Acute Viral Rhinosinusitis

D. Acute Viral Rhinosinusitis Clinical findings of acute viral rhinosinusitis include: nasal congestion, clear rhinorrhea, erythematous and edematous mucosa, hyposmia, and may also include cough, malaise, throat discomfort, and headache. Acute viral rhinosinusitis is self-limiting and usually lasts less than 10 days. Purulent discharge would indicate bacterial rhinosinusitis.

1. According to several surveys, what is the most frequently abused substance by adolescents in the United States? A. Cocaine B. Over-the-counter cough and cold medications C. Marijuana D. Alcohol

D. Alcohol * The Monitoring and Future survey and others show that alcohol is the most frequently abused substance in the United States, with alcohol experimentation usually beginning in or before middle school

A four-year-old male presents to the clinic with chief complaint of eye drainage. Upon exam observe injected sclera, tearing, stringy discharge, and persistent rubbing of the eyes by the patient These findings best describe: A. Bacterial conjunctivitis B. Blepharitis C. Nasolacrimal obstruction D. Allergic conjunctivitis

D. Allergic conjunctivitis allergic conjunctivitis is associated with a history of eye redness, itching, and watering or tearing of the eye. The discharge associated with allergic conjunctivitis is stringy with a possible association of intense tearing of the eye(s).

A 68-year-old male with CKD has been diagnosed with a new DVT. His creatinine clearance is 25 ml/min. Which direct-acting oral anticoagulant (DOAC) would be the safest option: A. Dabigatran B. Rivaroxaban C. Edoxaban D. Apixaban

D. Apixaban While all choices are viable options, only 25% of apixaban is renally excreted. The other options would need a dosage adjustment for CrCl < 30 ml/min.

Which of the following syndromes correlates with a spinal cord injury clinical pattern of bladder and bowel dysfunction, sensory changes in the perineum, and flaccid weakness in the lower extremities? A. Anterior cord syndrome B. Central cord syndrome C. Conus medullaris syndrome D. Cauda equina syndrome

D. Cauda equina syndrome* Cauda equina syndrome caused by spinal cord injury results in bowel and bladder dysfunction, sensory abnormalities in the perineum, and flaccid bilateral weakness in the legs. The only other spinal cord injury syndrome in children that results in bowel and bladder dysfunction is conus medullaris syndrome; the bowel and bladder dysfunction is significant and is accompanied by minimal motor impairment. Anterior cord syndrome results in loss of pain and temperature sensation and motor deficits. Central cord syndrome results in greater weakness in the arms than in the legs. Brown-Sequard injury is another clinical pattern of spinal cord injury in children which results in loss of temperature and pain on the contralateral side, motor paralysis, and loss of vibration and proprioception on the ipsilateral side.

Which of the following statements is true regarding diagnostic testing? A. Diagnostic testing is necessary for accurate diagnosis. B. High accuracy and low precision are ideal for diagnostic tests. C. Diagnostic tests with high specificity results in many false positives. D. Diagnostic tests with high sensitivity result in few false negatives

D. Diagnostic tests with high sensitivity result in few false negatives * * The ability of a diagnostic test to detect disease is referred to as sensitivity. The greater the sensitivity, the greater the ability to detect disease resulting in a higher value of positive tests in those with the disease, and fewer negative results in those with the disease (false negatives).

Several comorbidities can present with a diagnosis of ADHD, all of the following can be comorbidities of ADHD except: A. Depression B. Anxiety C. Mood Disorders D. Hyperthyroidism

D. Hyperthyroidism Hyperthyroidism is a medical diagnosis that can present with similar problems as ADHD. Anxiety, depression, mood disorders, and conduct disorders can be common comorbidities of ADHD.

As a practicing NP, you are performing an assessment on a 6-day old infant and are suspicious of patent ductus arteriosus. What is an early presentation of this disease? A. Increased peripheral pulses B. Widened pulse pressure C. Hyperdynamic Precordium D. Hypotension

D. Hypotension Signs of patent ductus arteriosus presents during days 3-7. The symptom includes a hyperdynamic precordium, increased peripheral pulses, and a widened pulse pressure. Early presentations sometimes include hypotension without a murmur.

A 4-month-old child presents to the clinic for a well child check. The mother states that she is worried her baby is not getting enough iron. She has a niece that was recently diagnosed with iron deficiency anemia at age 3. As the provider you can reassure the parent by stating all the following as effective measures to guard against iron deficiency anemia EXCEPT: A. Avoiding low-iron formulas during infancy B. Introducing iron fortified cereals and ground meats by age 6 months C. Universal screening for anemia at 12 months of age D. Limiting cow's milk to 32 oz/d in children 1-5 years

D. Limiting cow's milk to 32 oz/d in children 1-5 years Primary prevention for iron deficiency anemia should be achieved through dietary means, including feeding ground up meats and iron fortified cereals at 6 months, avoiding low-iron formula during infancy, and limiting cow's milk to *24 oz/d in children 1-5 years*

A nurse practitioner works at a local health department in suburban Alabama. They should advise a mother AGAINST breastfeeding or pumping and feeding their own breast milk in all of the following situations except: A. HIV infection of the mother B. A mother receiving lithium therapy C. A mother using methamphetamines D. Mastitis in the mother's right breast

D. Mastitis in the mother's right breast A mother suffering with mastitis needs antibiotic treatment, but it is not a contraindication to breastfeeding. Use of a breast pump may be used during healing to decrease pain from a newborn's latch Rationales a. Breastfeeding is contraindicated for HIV infected mothers in developed countries, including those on antiretroviral medications, as the virus can be excreted in the breast milk. In the case of a woman living in a developing country where access to clean drinking water is limited by which to mix formula, the risk of fatal diarrheal illness in a newborn outweighs the risk of HIV transmission. This is the only situation in which breastfeeding is encouraged with HIV infection. b. and c. Lithium and illegal drugs such as methamphetamines can be excreted in breast milk, thus causes contraindication to breastfeeding.

2. A 56 y/o male presents to clinic for complaints of fatigue, decreased libido and difficulty maintaining erections. Based on low serum testosterone levels, he is diagnosed with Male Hypogonadism. The Nurse Practitioner plans to prescribe testosterone therapy. The NP understands prior to initiating therapy, the following testing should be performed? A. FSH B. INR C. LH D. PSA

D. PSA Testosterone should not be prescribed to someone with active breast or prostate cancer. PSA screening has not shown to decrease mortality related to prostate cancer, however, clinicians regularly monitor PSA in someone receiving testosterone therapy/

Mrs. T 68 years old female presents to your clinic with a CC of "fever". Your physical assessment includes the following findings: inspiratory crackles, and diminished breath sounds at bases. You order a chest X-ray. What finding would be diagnostic of CAP? A. Multiple areas of cavitation within an area of consolidation B. Pulmonary opacities, most often apical C. Thick-walled solitary cavity surrounded by consolidation D. Parenchymal opacity

D. Parenchymal opacity "Clinical evaluation is less than 50% sensitive compared to chest imaging for the diagnosis of CAP" (Papadakis & Mcphee, 2018, p. 274). "A pulmonary opacity on chest radiography or CT scan is required to establish a diagnosis of CAP" (Papadakis & Mcphee, 2018, p. 275 ). "Radiographic findings range from patchy airspace opacities to lobar consolidation with air bronchograms to diffuse alveolar or interstitial opacities" (Papadakis & Mcphee, 2018, p. 275)."Additional findings can include pleural effusions and cavitation" (Papadakis & Mcphee, 2018, p. 275). "A lung abscess presents as a thick-walled solitary cavity surrounded by consolidation" (Papadakis & Mcphee, 2018, p. 281). Necrotizing pneumonia presents as "multiple areas of cavitation within an area of consolidation" (Papadakis & Mcphee, 2018, p. 281). Pulmonary opacities , most often apical is indicative of pulmonary tuberculosis (Papadakis & Mcphee, 2018, p. 282).

A G1P0 diabetic female with confirmed ultrasound pregnancy, comes in for a well check visit. She has mild urinary urgency, hesitancy, and burning during voiding. A random u/a is collected, and is positive for nitrates and pyuria. The next best step is to... A. Prescribe Ciprofloxacin 250mg BID for 3 days B. Prescribe Bactrim160/800 mg BID for 5 days. C. Encourage fluids, but prescribe no antibiotics, she is pregnant. D. Prescribe Nitrofurantoin 100mg BID for 5 days.

D. Prescribe Nitrofurantoin 100mg BID for 5 days. Macrobid, ampicillin, and cephalexin are preferred agents for UTI in pregnancy A. Fluoroquinolones are not recommended during pregnancy due to potential teratogenicity B. Sulfonamides are contraindicated due to increased risk for neonatal hyperbilirubinemia and kernicterus. C. Pyuria and positive nitrates are confirmatory for UTI. UTI's are common in pregnancy, even more frequent in diabetics, due to increased urinary stasis, and require treatment.

A 25 yr old male presents to your clinic and states he is worried he may have contracted HIV. He had anal sex with another man yesterday but they did not use a condom. What is the best course of care for your patient: A. Draw blood work, and tell him that if his partner's HIV status is unknown he does not need to start medication until his blood work shows HIV infection. B. Inform him that the window of oral treatment for HIV prophylaxis is 24 hours and so he cannot start oral medications now C. Administer 2 doses of Gardasil vaccination 2 weeks apart D. Prescribe antiretroviral medications for 28 days

D. Prescribe antiretroviral medications for 28 days Clinicians who care for MSM should also be aware of postexposure prophylaxis (PEP), which consists of antiretrovirals started within 72 hours of a discrete exposure to HIV and taken for 28 days, and either provide PEP themselves or be able to rapidly link patients to PEP care. PEP is recommended after condomless sexual exposure to a person known to be infected with HIV. Due to the high prevalence of HIV infection among MSM, PEP is likely also warranted after condomless sexual exposure to an MSM of unknown HIV status. PEP is not recommended if more than 72 hours have elapsed since the exposure. Some MSM who seek PEP may have ongoing high-risk sexual behavior and may thus benefit from PrEP after completion of PEP.

You are counseling a 24-year-old healthy female on recommendations for cancer risk reduction. Which of the following strategies would not be recommended? A. Be physically active and maintain a healthy weight B. Eat mostly fruits, vegetables, and whole grains while avoiding trans fats C. Avoid tobacco and excessive sun exposure D. Radiation from radiographic studies does not increase the lifetime cancer risk

D. Radiation from radiographic studies does not increase the lifetime cancer risk * All the options are modifiable risk factors for cancer prevention. Radiation exposure can increase the lifetime risk of developing cancer as well.

As a Family Nurse Practitioner, a patient who identifies with the LGBT community arrives as a new patient. It is important to understand what must be assessed to fully care for them? A. Sexual orientation and gender identity B. Gender identity, age, and sexual orientation C. Biological sex, age D. Sexual orientation, biological sex, gender identity, and sexual behavior

D. Sexual orientation, biological sex, gender identity, and sexual behavior As an FNP it is important to understand the differences between gender identity, sexual orientation, and sexual practice. Due to a lack of understanding great disparities have been created in the LGBT community in care. Papadakis and Mcphee, explain gender identity, what physical gender a person identifies, may differ from their sexual orientation, meaning which gender they are attracted to. Statistically many transgender women, male sex gender at birth who identify as women, are actually attracted sexually to women. They also report, 70% of women who identify as lesbian are exclusively involved sexually with other women, but 30% are involved sexually with men and women

A 35-year-old man presents to the office with complaints of hoarseness, burning chest pain after meals, and a chronic cough that worsens at nighttime. His symptoms have occurred daily over the past several months and are worsened by large meals and fatty foods. He denies any current medication use. Which of the following should not be included in this patient's treatment plan? A. Omeprazole 20 mg once daily for 4-8 weeks B. Elevate the head of the bed C. Elimination of peppermint D. Take omeprazole with the largest meal of the day

D. Take omeprazole with the largest meal of the day Proton pump inhibitors (PPI) should be taken 30 minutes prior to meals. Patients with symptomatic GERD should begin treatment with a PPI for 4-8 weeks

Mother brings newborn baby girl to clinic for checkup. She is breast-feeding only. Counseling should include all except: A. Newborns eat approximately 8-10 times a day with longer intervals during the night. B. Stool may be loose after each feeding until about 3-4 months old. C. Ensure proper latching on by compressing the nipple between thumb and index fingers, then quickly insert as much as possible of the nipple and areola. D. The baby should gain 1-3 pounds every week.

D. The baby should gain 1-3 pounds every week. Newborns will lose weight for the first few weeks then gain back to birth weight usually week 3 or 4.

A six-month old infant is brought to your clinic for a rash. The child was recently on antibiotics for an ear infection. On examination, you find sharply defined erythematous patches with pustule satellite lesions on the buttock. You explain to the parent that: A. This is an allergic reaction from the antibiotic B. This is a bacterial infection that needs an antibiotic C. Benadryl will clear up the rash D. The child has diaper dermatitis and topical Nystatin will improve the condition

D. The child has diaper dermatitis and topical Nystatin will improve the condition The treatment for diaper dermatitis is a topical imidazole cream. Question pool

What is the single most prognostic factor when evaluating a patient with a malignant melanoma? A. Color B. Shape C. Diameter D. Thickness

D. Thickness. Tumor thickness is the most important prognostic factor. The deeper/thickness in millimeters (mm) is associated with ten-year survival rates. Less than 1mm, 95%. 1-2mm 80%. 2-4mm, 55%. Greater than 4mm 30%. With lymph node involvement, the 5-year survival rate is 62%; with distant metastases, it is 15%.

What is the first-line therapy for children with sleep apnea with no other complications? A. weight loss B. CPAP C. supplemental oxygen D. adenotonsillectomy

D. adenotonsillectomy Adenotonsillectomy is the first-line therapy for children. Weight loss, CPAP, and supplemental oxygen are additional interventions that may still be needed after an adenotonsillectomy.

Which of the following interventions may be helpful in treating an infant with gastroesophageal reflux disease (GERD)? A. use of prokinetic agents, such as metoclopramide B. offering smaller feedings at less frequent intervals C. lying infant flat while feeding D. histamine-2-receptor antagonists, such as famotidine

D. histamine-2-receptor antagonists, such as famotidine Acid suppression may be used to treat suspected esophageal or extraesophageal complications of acid reflux in infants. Therapeutic options include histamine-2-receptor antagonists. There is not sufficient evidence to support the routine use of prokinetic agents for treatment of pediatric GERD. Regurgitation volume may be reduced by offering smaller feedings at more frequent intervals. Reflux resolves spontaneously in 85% of affected infants by 12 months of age, coincident with assumption of erect posture

A 50-year-old male with a history of tobacco use is in the office today with a complaint of severe and cramping pain in his calf muscles. He further states that the pain is relieved by rest and occurs again when walking. The symptoms he is describing is indicative of A.) Varicose veins B.) Lymphedema C.) Chronic venous insufficiency D.) Peripheral arterial disease

D.) Peripheral arterial disease * Peripheral arterial disease is more prevalent in 30% o patients who are 50 years of age, have either diabetes mellitus, a history of tobacco use, or 70 years of age without those risk factors. When peripheral artery disease occurs, usually lower extremities are not receiving enough blood flow to keep up with body demand. Blood flow reduction causes symptoms such as leg pain when walking. The symptoms are commonly in the calf muscles described as severe and cramping pain. In addition, rest relieves these symptoms and is but is brought back on when walking. + Dull, aching heaviness, or a feeling of leg fatigue from periods of standing is commonly associated with varicose veins. + Lymphedema is painless persistent edema of one or both lower extremities. + Progressive pitting edema of the leg with usual symptoms of itching, a dull discomfort made worse by periods of standing, and pain if an ulceration is present is commonly associated with chronic venous insufficiency.

Which of the following statements is true regarding the Rotavirus vaccine? A.) The Rotavirus series can be initiated at 6-months of age, but no later. B.) The infant who vomits up a dose immediately after it was initiated should receive another dose. C.) The child with a history of intussusception can still receive the vaccine as long as it has been surgically corrected. D.) The Rotavirus series can be started no earlier than 6-weeks of age.

D.) The Rotavirus series can be started no earlier than 6-weeks of age. Rationale: The Rotavirus vaccine must be started between the ages of 6 weeks - 14 weeks and 6 days old. The child 15 weeks or older is too old to start the series, due to the lack of safety data of administering the dose to older infants. History of intussusception is a contraindication to receive the vaccine. Doses do not need to be re-administered due to vomiting it up.

The nurse practitioner is evaluating a 10 day old newborn. The mother states the infant has developed an abdominal mass and hematuria in the last 24 hours. Renal vein thrombosis is suspected. Which diagnostic test would be most appropriate to confirm this diagnosis? A.) CT scan of abdomen and pelvis w/contrast B.) KUB x-ray C.) Cystourethrogram D.) Ultrasonography and Doppler flow studies

D.) Ultrasonography and Doppler flow studies While no single laboratory test is diagnostic of renal vein thrombosis, hematuria is usually present and is generally characterized by the sudden development of an abdominal mass. The diagnosis of renal vein thrombosis is made by ultrasonography and doppler flow studies. CT scan may be helpful, but subjecting the infant to a large amount of radiation at this point is not recommended. KUB is more useful in diagnosis of renal stones. Cystourethrogram is used to diagnose vesicoureteral reflux.

What is the gold standard for diagnosis of acute uncomplicated cystitis? A: Urinalysis B: Renal ultrasound C: Laboratory studies - BUN, creatinine D: urine culture

D: urine culture - The gold standard remains culture of a urine specimen.

Which of the following is NOT a risk factor for erectile dysfunction? a. Smoking b. Diabetes c. BPH d. Hypertension

Diabetes, hypertension, and smoking are all risk factors for ED. BPH treatment regimen (alpha blockers) may cause ED; but BPH in and of itself is not a risk factor. McPhee p 975

MS. ART (Use for All Diastolic Murmurs)

Diastolic murmurs are always indicative of heart disease (unlike systolic murmurs). MS (Mitral Stenosis) A low-pitched diastolic rumbling murmur: ■ Heard best at the apex of the heart or the apical area ■ Also called an "opening snap" (use bell of the stethoscope) AR (Aortic Regurgitation) A high-pitched diastolic murmur: ■ Best heard at the second ICS at the right side of the sternum ■ High-pitched blowing murmur (use diaphragm of the stethoscope)

What is a good abx for penicillinase-producing staph skin infections (e.g. mastitis and impetigo).

Dicloxicillin Leik

Key indicators for considering a dx of COPD

Dyspnea that is: + Progressive + Worse with exercise + Persistent Chronic cough (poss intermittent and non-productive) Chronic sputum production Hx of exposure to risk factors + Tobacco smoke + Smoke from home cooking/heating fuels + Occupational dusts and chemicals Family hx of COPD

Cause and treatment of lyme disease?

Early Lyme disease (erythema migrans rash stage): ■ Ixodes (deer) tick bite; spirochete called Borrelia burgdorferi. ■ Treat with doxycycline × 21 days. ■ Majority of the cases are in the Mid-Atlantic and New England states (i.e., CT, MA, NY, NJ, PA). Leik

1st generation cephalosporins: Examples, uses

Examples: Cefazolin, cephalexin Used against: Gram-pos cocci, Proteus mirabilis, E. coli, Klebsiella pneumoniae. Destroyed by beta lactamase. Specific uses: Cefazolin used prior to surgery to prevent S. aureus wound infections. Good for skin infections not caused by MRSA. Impetigo. Narrowest antimicrobial spectrum

3rd generation cephalosporins: Examples, uses

Examples: Cefdinir, ceftriaxone Used against: MUCH more active against gram-negative bacteria than gen 1 or 2 Specific uses: + 3rd gen are drug of choice for meningitis caused by gram negative bacteria. + Ceftriaxone (rocephin) is drug of choice for gonorrhea, h. influenzae, proteus, calmonella, klebsiella. + AOM in children, acute sinusitis, otitis media + Pyelonephritis, CAP NOTES: Reaches decent concentrations in the CSF. Use sparingly r/t BROAD spectrum.

2nd generation cephalosporins: Examples, uses

Examples: Cefuroxime, cefoxitin Used against: Gram-negative bacteria Specific uses: + Cefuroxime has been used for pneumonia caused by H. influenzae, Klebsiella, pneumococci and staphylococci. Also good for otitis, sinusitis and resp tract infections. +Cefoxitin has been used for abdominal and pelvic infections. NOTES: Inccreased resistance to beta lactamase. Does NOT work against Pseudomonas.

Fluoroquinolones

FLOUROQUINOLONES (Category D) Older forms: ciprofloxacin (Cipro), ofloxacin (Floxin). For gram-negatives, pseudomonas, some atypical bacteria. Best antipseudomonal antibiotic is ciprofloxacin. Newer forms: levofloxacin (Levaquin), gemifloxacin (Factive), gatifloxacin (Tequin). NOTE: Careful, this drug class has a lot of drug interactions and causes prolongation of QT interval. Warning: Achilles & other types of tendon rupture. Interacts with theophylline, phenytoin, anticoagulants. Avoid using in pregnancy, lactation, & children <18 years (affects cartilage development) . If on Coumadin (Cat. X), monitor INR. Used for: UTIs, pyelonephritis, pneumonia, sinusitis, otitis media (newer forms), anthrax, bone infections, joint infections, GI, prostatitis, epididymitis

The most common triggers for anaphylaxis in children?

FOODS Leik

Dermatological condition with maculopapular rash in a lace-like pattern?

Fifth's disease Leik

Treatment plan for AOM or acute sinusitis

First-Line ■ Amoxicillin is the gold standard for any age group. - First line: Amoxicillin 500 mg to 875 mg BID to TID × 10 to 14 days. ■ OR consider starting with second-line antibiotic (Augmentin, Ceftin, Cefzil) if severe disease (severe ear pain, fever). ■ Most patients will respond within 48 to 72 hours. If no improvement noted (TM same with symptoms), then switch to second-line drug (Augmentin, Ceftin, Cefzil). Second-Line Second-line antibiotic treatment criteria: History of antibiotic use in the past 3 months, no response to amoxicillin, or severe case of AOM (high fever, severe ear pain). Time duration is from 10 to 14 days. ■ Amoxicillin/clavulanate (Augmentin) PO BID. ■ Or third-generation cephalosporins. - Cefdinir (Omnicef) PO BID. - Cefpodoxime (Vantin) PO BID. - Ceftriaxone (Rocephin) 3 g IM × 1 dose. ■ OR Cefuroxime (Ceftin) PO BID—second-generation cephalosporin. Penicillin-Allergic Patients ■ Azithromycin (Z-Pack) × 5 days or clarithromycin (Biaxin) PO BID. ■ Trimethoprim sulfamethazole (Bactrim DS) PO BID. ■ Levofloxacin (Levaquin) or moxifloxacin (Avelox) if 18 years or older. Increases risk for tendinitis/Achilles tendon rupture.

Symptoms and treatment of polymyalgic rheumatica

First-line treatment for PMR includes long-term steroids. Long-term, low-dosed steroids are commonly used to control symptoms (pain, stiffness on shoulders, and hip girdle). PMR patients are also at higher risk for temporal arteritis. LEIK

Common forms of plague transmission

Flea bite, contact with or ingestion of contaminated tissue. Also aerosolization of plague has been discussed as a possible mode of transmission Forum

Hep B Surface antibody (anti-HBs)

Generally interpreted as indicating recovery and immunity from Hep B. Can also develop in persons successfully vaccinated. CDC

Hepatitis serology -- A, C and E

Hep A + HAV Igm = Active Hep A infection + HAV IgG = Protective! (Recovery from Hep A or Vaccination) Hep E (Danger for PG!) + HEV Igm = Active Hep A infection + HEV IgG = Protective! (Recovery from Hep E) Hep C + HCV IgG = NOT protective (or dx of acute vs chronic) + HCV RNA test with PCR tests for levels of HCV RNA in blood.

Monitoring of patients on atypical antipsychotics (e.g. Risperdal <risperdone>, Zyprexa <olanzapine>, Seroquel <quietapine>)

High risk of weight gain, metabolic syndrome, and T2DM. + Monitor weight every 3 months. <<Black Box Warning: higher mortality in elderly patients.>> +Monitor: TSH, lipids, weight/body mass index (BMI) Leik

Hordeolum vs Chalazion

Hordeolum (Stye) - acute, PAINFUL localized swelling of the eyelid (usu Staph Aureus). - Warm compress. I&D if no change in 48 hours. Chalazion - chronic, STERILE, PAINLESS inflamm from blocked Meiobomian Gland. - Rx - warm compress; but may need I & Curettage. Biopsy if recurrent. Hint: Hordeolum = oh HOLY crap (painful, acute) Chalazion = chronic

Indications for calcium channel blockers

Hypertension Raynaud's (first line!) Leik

A patient has an INR of 8.0. Physical examination is negative for petechiae, bleeding gums, bruising, or dark stools. What is the best treatment plan for this patient?

INR between 5.0 and 9.0 (without bleeding): + Hold the warfarin for 1 to 2 doses. Recheck INR every 2 to 3 days until it is stable (INR between 2.0 and 3.0). +Another option is to hold the warfarin and add a small dose of oral vitamin K. Limit and/or avoid high vitamin K foods (green leafy vegetables, broccoli, brussels sprouts, cabbage). After the INR becomes stable, recheck it monthly. Leik

Presentation and treatment of ITP?

ITP severity ranges from mild to severe (platelet count less than 30,000/μL). Platelets are broken down by the spleen, causing thrombocytopenia. Look for easy bruising, petechiae, purpura, epistaxis, and gingival bleeding (combined with low platelet count). Initial treatment for ITP is glucocorticoids (i.e., prednisone) based on platelet response. Leik

​Pseudotumor cerebri

Idiopathic intracranial hypertension: high ICP, papilledema, headache (84-92%), transient visual symptoms, diplopia, vision loss, pulsatile tinnitus, retrobulbar pain. Imaging MRI, lumbar puncture. Patient: Obese woman (childbearing age) who complains of headaches with papilledema found on fundoscopic exam.

Which religion states that after death, the body should not be touched for 3 - 8 hours?

In the Culture Powerpoint it states that in Buddhism the body is not to be touched for 3 to 8 hours after the person stops breathing because the spirit is still there.

First line treatment for gastroparesis

Initial management of gastroparesis consists of dietary modification, optimization of glycemic control and hydration, and in patients with continued symptoms, pharmacologic therapy with prokinetic and antiemetics. Dietary modification — Dietary modification is considered first-line therapy in patients with mild gastroparesis. Patients with gastroparesis should be advised by a dietician to consume small, frequent meals four to five times a day that are low in fat and contain only soluble fiber. Optimize glycemic control — Diabetes mellitus is a common cause of delayed gastric emptying [11]. In patients with diabetes, incretin-based therapies such as pramlintide (amylin analog) and GLP1 analogues or agonists (eg, liraglutide, exenatide) should be avoided as they can delay gastric emptying. In contrast, dipeptidyl peptidase IV inhibitors (eg, vildagliptin, sitagliptin) do not affect gastric emptying. Prokinetics — Metoclopramide — Metoclopramide is first-line therapy for gastroparesis for no longer than 12 weeks unless patients have a therapeutic benefit that outweighs the risks. The side effects associated with metoclopramide include central side effects of anxiety, restlessness, and depression, hyperprolactinemia, and QT interval prolongation. We initiate treatment with a low-dose liquid formulation (eg, 5 mg, 15 minutes before meals and at bedtime), titrating up to identify the lowest effective dose. Macrolide antibiotics Erythromycin — Erythromycin, a motilin agonist, induces high-amplitude gastric propulsive contractions that increase gastric emptying [33,34]. Erythromycin also stimulates fundic contractility, or at least inhibits the accommodation response of the proximal stomach after food ingestion [35]. Patients who fail to respond to a trial of metoclopramide should be treated with oral erythromycin (liquid formulation, 40 to 250 mg three times daily before meals) (algorithm 1) [2]. Oral erythromycin should be administered for no longer than four weeks at a time, as the effect of erythromycin decreases due to tachyphylaxis. Azithromycin — Azithromycin has a longer half-life, and fewer gastrointestinal adverse effects and drug interactions as compared with erythromycin. However, azithromycin is a weak inhibitor of CYP3A4 and like erythromycin may prolong cardiac repolarization and the QT interval, increasing the risk of cardiac arrhythmias and torsade de pointes.

Which meds are the respiratory quinolones?

Levofloxacin and moxifloxacin are good against strep pneumonia.

Which quinolones are effective against gram postive AND gram negative bacteria?

Levofloxacin, moxifloxacin and gatifloxacin.

Recommended screening for lipid disorders?

Lipid Disorders ■ Total lipid profile after a 9-hour (minimum) fast. Males ■ Age 35 years or older: Screen for lipid disorder. ■ Age 20 to 35 years: Screen if at increased risk of heart disease. Females ■ Age 45 years or older: Screen for lipid disorder. ■ Age 20 to 45 years: Screen if at increased risk of heart disease (i.e., hypertensive, strong family history of MI, or stroke). Leik

Diagnostic lab test for primary adrenal insufficiency

Low serum cortisol High ACTH

Macrolides

MACROLIDES (most are Cat. B and two are Cat. C) 1. erythromycin estolate (Ilosone) 2. clarithromycin Category B macrolides: Erythromycin (Ery-Tab, E-mycin), azithromycin (Zithromax, Z-Pack), clarithromycin (Biaxin) Macrolides may: Increased risk of arrythmias, prolong the QT interval, nausea/GI upset, diarrhea, etc. NOTE: This drug class has more drug-drug interactions, compared with the other classes of antibiotics. Warfarin (Coumadin - increase risk bleeding), theophylline, carbamezapine (Tegretol), benzodiazepines (alprazolam, midazolam - slows down clearance so last longer), cisapride (risk extrasystoles, prolongs QT interval), HMG-CoA reductase inhibitors (lovastatin, simvastatin -increased risk rhabdomyolysis ,quinidine (risk arrythymias)

Foods with high tyramine content can cause dangerous food-drug interactions with WHAT medications?

MAOI inhibitors (Marplan, Nardil, and Parnate) Leik

MR. ASH (Use for All Systolic Murmurs)

MR (Mitral Regurgitation) A pansystolic (or holosystolic) murmur: ■ Heard best at the apex of the heart or the apical area ■ Radiates to axilla ■ Loud blowing and high-pitched murmur (use the diaphragm of the stethoscope) AS (Aortic Stenosis) A midsystolic ejection murmur: ■ Best heard at the second ICS at the right side of the sternum ■ Radiates to the neck ■ A harsh and noisy murmur (use diaphragm of stethoscope) Patients with aortic stenosis should avoid physical overexertion, as there is increased risk of sudden death.

The most common triggers for anaphylaxis in adults?

Medications and insect stings Leik

Foods which are high in potassium

Most fruits (especially apricot, banana, orange, prune juice), some vegetables.

When should a mother receive Rhogam?

Mother = Rh Neg Cord = Coombs Neg Newborn = Rh Pos Mom must receive Rhogam after delivery to prevent isoimmunization.

B. Ziprasidone antipsychotic medications can prolong the QT interval and lead to ventricular arrhythmias.

Mother brings her 16-year-old daughter to the clinic for complaints of new-onset palpitations. Family history includes paternal grandfather deceased in early 50's due to MI. A thorough medication history obtained. An ECG was obtained in clinic. Which of her medications listed below is most likely to cause the ECG abnormality? A. Ranitidine B. Ziprasidone C. Cetirizine D. Albuterol

Dry needling: Who can do it and what is it for?

Must be performed by licensed PT (who used to only require a masters degree, but now must have a doctorate). Indications include myofascial release. Forum

Best medications for lowering triglycerides?

Niacin and fibrates

Who gets screened for AAA?

One-time screening (only men aged 65-75 years) who are cigarette smokers. Screening test is ultrasound of the abdomen Leik

Penicillins

PENICILLINS (Category B) 1st Generation: po penicillin V (Veetids), IM benzathine penicillin (Bicillin L-A). 2nd Generation: dicloxacillin (Dynapen) NOTE: Penicillin-allergic, can use macrolides, cephalosporins, newer quinolones (age >18 years), Preferred for strep throat. Also cellulitis, AOM, sinusitis

Dermatological condition with maculopapular rashes that are oval-shaped with a herald patch?

Pityriasis rosea Leik

Which form of plague is most effective as a bioweapon?

Pneumonic plague Forum

What are the anterior drawer maneuver and lachman maneuver used to test?

Positive if anterior cruciate ligament (ACL) of the knee is damaged. The knee may also be unstable. Anterior Drawer - The patient's knee is flexed to 90 degrees, and the hip is flexed to 45 degrees. In this position, the anterior cruciate ligament is almost parallel with the tibial plateau. The patient's foot is held on the table by the examiner's body with the examiner sitting on the patient's forefoot and the foot in neutral rotation. The examiner's hands are placed around the tibia to ensure that the hamstring muscles are relaxed. The tibia is then drawn forward on the femur. The normal amount of movement that should be present is approximately 6mm. Lachmann -- The therapist holds the patients knee between full extension and 30 degrees of flexion. The patient's femur is stabilized with one of the examiner's hands (the "outside hand) while the proximal aspect of the tibia is moved forward with the other ("inside") hand. You should feel a "clunking" sensation.

What is Finkelstein's test?

Positive in De Quervains tenosynovitis. To do this test, you bend your thumb down across the palm of your hand, and then cover your thumb with your fingers. Next, bend your wrist toward your little finger.

IgM antibody to hepatitis B core antigen (IgM anti-HBc)

Positivity indicates recent infection with hepatitis B virus (<6 mos). Its presence indicates acute infection.

Presentation and treatment of impetigo rash?

Presentation: Initially appear as papules that develop into bullae that rupture easily, becoming superficial, bright red "weeping" rashes with honeycolored exudate that becomes crusted as it dries. The rashes are very pruritic and are located on areas that are easily traumatized, such as the face, arms, or legs. Insect bites, acne lesions, and varicella lesions can also become secondarily infected, resulting in impetigo. Treatment ■ Cephalexin (Keflex) QID, dicloxacillin QID × 10 days. ■ PCN allergic: Azithromycin 250 mg × 5 days (macrolides), clindamycin × 10 days. ■ If very few lesions with no bulla, may use topical 2% mupirocin ointment (Bacitracin) × 10 days. ■ Frequent hand-washing, shower/bathe daily to remove crusts. Leik

Tinea Vesicolor diagnosis and treatment

Presentation: Hypopigmented round macules on chest, shoulders and/or back that appear after sun exposure. Not pruritic. KOH prep: spaghetti and meatballs look" Treat with topical azole (selenium sulfide) shampoo or cream BID x 2 weeks.

Presentation of varicella

Prodrome of fever, pharyngitis, and malaise that is followed within 24 hours by the eruption of pruritic vesicular lesions in different stages of development over a period of 4 days. The rashes start on the head/face and quickly spread to the trunk and extremities. It takes 1 to 2 weeks for the crusts to fall off and the skin to heal. Contagious from 1 to 2 days before the onset of the rash and until all of the lesions have crusted over (chickenpox and shingles).

Hep B Surface Antigen (HBsAg)

Protein on surface of Hep B, can be detected in high levels *during acute OR chronic Hep B infection*. The presence of HBsAG indicates that the *person is infectious*. The body normally responds with antibodies to this as immune response. HBsAG is what is used in the HBV vaccine. CDC

The most common bacterial pathogen in otitis externa?

Pseudomonas Leik

Causes of secondary HTN

Renal: + Renal artery stenosis (epigastric or flank bruit) + Polycystic kidneys + CKD (↑ Cr and ↓ GFR) Endocrine + Hyperthyroidism (↑ HR, ↓ weight, tremor, anxiety) + Hyperaldosteronism (↓ K+, ↑ Na+) + Pheochromocytoma (anxiety, sweating, severe HA, arrhythmias) Other: + Obstructive sleep apnea + Coarctation of the aorta (arm BP ↑ than legs) Leik

Pediatric maintenance fluid requirements

STEPS TO CALCULATE USUAL MAINTENANCE FLUIDS: 1. Convert child's wt in lbs to kg 2. Calculate 100 ml of fluid/kg/24 hr for the 1st 10 kg of wt 3. Calculate 50 ml of fluid/kg/24 hr for the 2nd 10 kg of wt 4. Calculate 10 to 25 ml of fluid/kg/24 hr for each kg over 20 of wt 5. Add the products of steps 2,3, and 4 to determine the ml of fluid needed/24 hrs

Sulfonamides

SULFONAMIDES (Cat. C 1st & 3rd trimester) Trimethoprim-sulfamethazole (Bactrim, Septra), sulfadiazine, sulfisoxazole. Nitrofurantoin also has sulfa and should be avoided in the last month of pregnancy (hyperbilirubinemia). Avoid sulfas with G6PD anemia (hemolysis/jaundice). NOTE: Avoid combining sulfas with warfarin (increases INR). Avoid sulfas in pregnant women in the 1st trimester (folate antagonist) and 3rd trimester (hyperbilirubinemia). G6PD anemia (hemolysis). Rash (Stevens-Johnson syndrome). Monitor INR if on warfarin (bleeds). Used for: UTIs, pyelonephritis, toxoplasmosis, AOM, etc.

Presentation of scarlatina (scarlet fever) rash?

Sandpaper-like texture and is accompanied by a sore throat, strawberry tongue, and skin desquamination (peeling) of the palms and soles. It is not pruritic. Leik

HIV infection testing

Screening test - ELISA Confirmatory test (if ELISA is +) - Western blot

Hemoglobin electrophoresis is the gold standard test for WHAT conditions?

Sickle cell anemia G6PD anemia Alpha or beta thalassemia Leik

S/S Renal (adrenocortical) Insufficiency

Signs and symptoms may include: + Fatigue/weakness. + Weight loss and anorexia. + Darkening of your skin (hyperpigmentation) + Low blood pressure, even fainting. + Hyponatremia + Low blood sugar (hypoglycemia) + Abdominal pain. + Hyperkalemia Primary cause -- Addison's dz Secondary cause -- Often due to withdrawal of a glucocorticoid medication

Alpha thalassemia is more common among WHAT ethnic group?

Southeast Asians (such as Filipinos) LEIK

Gold standard for diagnosis of COPD

Spirometry The presence of a postbronchodilator FEV1/FVC < 0.70 confirms the presence of persistent airflow limitation and thus of COPD

A 12-year old, obese male presents with left medial knee pain, which he explains is sometimes felt in the thigh and a limp that began approximately 2 months ago. He is able to bear weight on the left leg, and denies any recent trauma or falls. Examination reveals limited internal rotation of the left hip. The nurse practitioner suspects which of the following: Medial meniscal tear Medial femoral condyle osteochondritis Stable slipped capital femoral epiphysis Medial collateral ligament sprain

Stable slipped capital femoral epiphysis * Slipped capital femoral epiphysis (SCFE) is seen most often in adolescent, obese males. In stable SCFE, children are able to bear weight on the affected limb and present with pain and a limp that persists over an extended period of time. Pain can often refer to the thigh or medial side of the knee of the affected extremity. Physical exam displays an unchanged limited internal rotation of the hip. Acute SCFE usually occurs after direct trauma to the hip or a after a fall. SCFE is classified as stable or unstable. With unstable SCFE children are unable to bear weight on the affected limb and with stable SCFE the child is able to bear weight

Tanner Stages Boys

Stage I: Prepubertal pattern. Stage II: Testes with scrotum starts to enlarge (scrotal skin starts to get darker/more ruggae). Stage III: Penis grows longer (length) and testes/scrotum continues to become larger. Stage IV: Penis become wider and continues growing in length (testes are larger with darker scrotal skin and more ruggae). Stage V: Adult pattern.

Safety issues with bisphosphenates (e.g. Fosamax <alendronate>, Actonel <risedronate>)

Symptoms: + Jaw pain (jaw necrosis). + Chest pain, difficulty swallowing, burning mid-back (perforation). Dosage info: + Take alone upon awakening with 8 oz glass *water* (NOT juice) before breakfast. + Do not lie down for 30 minutes afterward. + Do not mix with other drugs). + Take first thing in the morning before breakfast Leik

Tetanus vaccine recommendations

Tetanus Vaccine ■ Give every 10 years for lifetime. ■ Boosters: For "dirty"/contaminated wounds, give a booster if the last dose was more than 5 years prior. ■ Age 7 years and older: Use only tetanus and diphtheria (Td) and tetanus, diphtheria, and acellular pertussis (Tdap) forms of the vaccine. Leik

Tetracyclines

Tetracyclines (Category D) Tetracycline, doxycycline, minocycline (Minocin) Stains actively growing tooth enamel. Used for: chlamydia STDs, atypical pneumonia, UTIs, otitis media, PCP in AIDs, Lyme disease

USPSTF recommendations for breast cancer screening?

The USPSTF recommends biennial screening mammography for women aged 50 to 74 years. Leik

Which of the following topical medications can be used to treat post-herpetic neuralgia: 1. Capsaicin 2. Hydrocortisone 0.5% 3. Clotrimazole 4. Diphenhydramine

The answer is capsaicin option 1. FYI: Po oral acyclovir capsules/pills cost about $18 - $20 or higher. The pills or capsules are cheaper than the cream (topical form). The price for topical acyclovir ointment ranges from $80 to $840 dollars.

The CDC recommends which group should be tested at least once with the Hepatitis C antibody test: 1. People born from 1945 through 1965 2. People born from 1966 through 1975 3. People born from 1976 through 1985 4. People born from 1986 through 1995

The answer is option 1. People born from 1945 to 1965 belong to the "Baby Boomers" group and the CDC recommends testing them at least once (without prior ascertainment of HCV factors). https://www.cdc.gov/hepatitis/hcv/guidelinesc.htm

​What is the preferred medication for an adult patient with stable heart failure: 1. Furosemide 2. ACE inhibitors 3. Calcium channel blockers 4. Beta-blockers

The answer is option 2. They are asking about stable heart failure (fluid overload is controlled). Leik

A 65-year-old male presents with a history of a chronic cough that is productive of large amounts of off-white to light-yellow colored sputum. The patient reports a history of cigarette smoking. The chest x-ray reveals hyperinflation with flattened diaphragms and several small bullae. Which of the following drug classes is the initial treatment of choice for this condition? A) Short-acting B2 agonists. B) Anticholinergics. C) Pneumococcal polysaccharide vaccine (Pneumovax). D) Oxygen by nasal cannula.

The correct answer is Option B. The initial treatment of choice for chronic bronchitis/COPD is ipratropium bromide or Atrovent (an anticholinergic). Leik

A patient who is treated with amoxicillin returns in 48 hours without improvement (complains of ear pain, bulging TMP). What is the next step?

The next step is to discontinue the amoxicillin and start the patient on a second-line antibiotic (for otitis media) such as amoxicillin-clavulanate (Augmentin) BID × 10 to 14 days. Leik

Potent inhibitors of the cytochrome P450 system.

These drugs are respnsible for a large number of drug -drug interactions: ■ Macrolides (erythromycin, clarithromycin, pediazole) ■ Antifungals (ketoconazole, fluconazole) ■ Cimetidine (Tagament) ■ Citaprolam (Celexa)

What HTN medications are preferred for patients with osteopenia or osteoporosis?

Thiazide diuretics, because thiazide decreases calcium excretion by the kidneys and it stimulates osteoclast activity that helps with bone formation. Leik

Patients with sulfa allergies should avoid what type of HTN medication?

Thiazides. Potassium-sparing diuretics such as triamterene and amiloride (Midamor) are the alternative options for these patients. Leik

Balanitis is caused by which of the following organisms: a) staphylococcus aureas b) stroptococcus pyogenes c) candida albicans d) tinea fungi

Tip. The correct answer is c. Balanitis is more likely to be seen in uncirmcumcised males especially diabetic males and those who are immunocompromised (ie: HIV/AIDs).

A 15 year old high school athlete complains of a painful area below both knees. He tells you they feel bone-like and are tender to palpation. He denies any hot joints, fever, rash, or difficulties with weight-bearing. Which of the following conditions is most likely: a) osteomyelitis b) internal tibial torsion c) Osgood-Schlatter Disease d) Calve-Perthe's Disease

Tip: A good clue is the patient's age ( a teen) and the presentation. The correct answer is c.

A 13 year old female patient who is allergic to erythromycin and sulfa drugs has a positive strep culture. Which of the following pharmacologic agents would you recommend: a) clarithromycin (Biaxin) 500mg po BID x 10 days b) penicillin 250mg po QID x 10 days c) levofloxacin (Levaquin) 500mg po daily x 7days d) trimothoprim-sulfamethazole (Bactrim DS) one tablet BID x 10 days

Tip: Remember, flouroquinolones (or quinolones) are contraindicated in patients younger than 18 years of age due to TENDON RUPTURE. The correct answer is b.

Which of the following drugs is contraindicated in pregnant women: a) amoxicillin (Amoxil) b) cephalexin (Keflex) c) isotretinoin (Accutane) d) acetaminophen (Tylenol)

Tip: The answer is c. Accutane is a category X drug and any reproductive-aged female on this drug should always be on two forms of reliable birth control. Accutane causes massive birth defects of the CNS, face, eyes, heart, glandular system, etc.

Which of the following drugs can exacerbate the symptoms of GERD: 1. Calcium channel blockers 2. Diuretics 3. Proton pump inhibitors 4. Antacids

Tip: The answer is option 1. CCBs can worsen GERD symptoms. Many other drugs can also worsen GERD symptoms such as nitrates, biphosphanates, NSAIDs, anticholinergics, iron supplements, etc.

All of the following can be used to treat generalized anxiety disorder in an older adult except: 1. SSRI 2. Buspirone (Buspar) 3. Cognitive behavior therapy (CBT) 4. Quietipine (Seroquel)

Tip: The answer is option 4. Seroquel is used to treat schizophrenia, psychosis, bipolar disorder

Sue D., a 30 year old bank teller, tells the nurse practitioner during a routine physical that she forgot to take her birth control pills for 2 consecutive days (or 2 days in a row). The NP would recommend which of the following: a) Take two pills now, then two pills the next day and use condoms as back-up until the next pill cycle. b) Throw away the current package and start a new pill cycle. c) Use condoms and throw away the two forgotten pills, then start a new pill cycle d) Take two pills today, then one pill daily until the next pill cycle

Tip: The correct answer is a

Which cranial nerve is the nurse practitioner assessing with the Rinne and the Weber test: a) CN 7 b) CN 8 c) CN 9 d) CN 3, 4, and 6

Tip: The correct answer is b. CN 8 is the acoustic nerve. Other methods to test hearing is the whisper test and the Weber.

A 35 year old woman has a history of mitral valve prolapse (MVP) and is scheduled for a root canal the next day. She tells the NP that she needs an antibiotic before undergoing the procedure. Which of the following is the best option for the NP to follow: a) Write a script for amoxicillin 2 g before and 1.5 g after the procedure. b) Write a script for amoxicillin 2 g one hour before the procedure c) According to the American Heart Association's latest guidelines, there is no need for prophylaxis before dental procedures if the patient does not have a prosthetic valve or does not have a history of infective endocarditis. d) Tell the patient that prophylaxis before certain procedures is still recommended.for patients with her diagnosis

Tip: The correct answer is c.

Treatment plan for strep throat

Treatment Plan ■ Throat culture and sensitivity (C&S) or Rapid Strep testing. - First line: Penicillin QID × 10 days. - Ibuprofen (Advil) or acetaminophen (Tylenol) for throat pain and fever. - Symptomatic treatment: Salt water gargles, throat lozenges. Drink more fluids. - Repeat culture if high risk: History of mitral valve prolapse (MVP) or heart valve surgery. ■ PCN allergy. - Z-Pack (azithromycin) × 5 days. - Levaquin (levofloxacin) × 10 days (contraindicated if age less than 18).

50 year old non-smoker without comorbidities presents to the clinic and is dx with CAP. His VS are normal except for temp of 101.6. Sputum specimen is cllected and sent for C&S. What action should be taken by the NP? a) Wiat for culture before starting abx b) Start clarithromycin 500 mg BID for 5 days c) Start PCN V 500 mg QID for 7 - 10 days d) Start cipro 500 mg BID for 10 to 14 days

Treatment of nonsmokers < 60 yrs with no comorbidities: + Erythromycin or other macrolide (OR doxy) Treatment of pts with comorbidities OR use of abx in last 3 months: + Resp quinolone PLUS beta lactam Duration of treatment is minimum of 5 days and pt should be afebrile and stable for 48 hours before treatment is discontinued.

periorbital cellulitis treatment:

Usually Related to Sinusits: - Bacterial pathogens = S. pneumoniae, NTHI, S. aureus, GABHS - 1st Line Therapy = Amoxicillin/Clavulanic acid - Duration = 7-10 days Secondary to trauma: - Bacterial pathogens = S. aureus, GABHS - 1st Line Therapy = Cephalexin - Duration = 10-21 days EMERGENCY - CONSULT/REFER

Dermatological condition with maculopapular rrashes with papules, vesicles, and crusts?

Varicella Leik

What is the confirmatory test for HIV?

Western blot (high SPECIFICITY, meaning it is good at ruling out a person who does NOT have the disease.) Leik

USPSTF recommendations for cervical cancer screening?

Women aged 21 to 29 + Screening for cervical cancer every 3 years with cervical cytology alone Women aged 30 to 65 years should be screened + Every 3 years with cervical cytology alone, OR + Every 5 years with high-risk human papillomavirus (hrHPV) testing alone, OR + Every 5 years with hrHPV testing in combination with cytology (cotesting). Leik

A 58yr old obese, Caucasian male patient presents with complaints of chronic heart burn. The FNP would order an EGD to rule out: a. Barrett Esophagus b. Esophageal Varices c. Celiac Disease d. Diverticulitis

a Barrett Esophagus is a change in the epithelium of the esophagus in which squamous cells are replaced by metaplastic columnar cells. The most serious complication of this disease is esophageal adenocarcinoma. This should be screened for endoscopically in men with two or more risk factors. The additional risk factors include white race, truncal obesity, >50 years old, current or prior history of smoking, or a family history of esophageal cancer or Barrett's Esophagus. Treatment should include long-term use of a PPI daily or twice daily. PPI use in patients with Barrett Esophagus have shown to provide symptom relief and reduce the risk of developing cancer

Which of the following is the most common malignancy found in childhood? a) Acute Lymphoblastic Leukemia (ALL) b) Acute Myloid Leukemia (AML) c) Hodgkin Lymphoma d) Neuroblastoma

a) Acute Lymphoblastic Leukemia (ALL) (ALL) is the leading cause of malignancy in children accounting for 25% of any cancer diagnoses in children. (ALM) occurs in 25% of children with anemia alone. Hodgkin Lymphoma is more prevalent in adults. Neuroblastomas account for only 7-10% of malignancies in children

1. A mother brought her 2-year-old daughter complaining of "Right elbow will not bend". The Nurse Practitioner suspects that the child has a nursemaid's elbow if: a) Child's right elbow appears fully pronated and painful b) Child's right elbow with obvious deformity, edema, and hematoma c) Child's right elbow with no pain or any point of tenderness d) Radiologic findings on the right elbow are abnormal

a) Child's right elbow appears fully pronated and painful Subluxation of the radial head is also known as nursemaid's elbow. In nursemaid's elbow, the child appears with the elbow fully pronated and painful. The typical complaint is the child's inability to bend the elbow. There is a noted point of tenderness over the radial head. Radiology findings are normal. Obvious elbow deformity, edema, and hematoma are indications of fracture or serious injury.

Your 19-month-old patient is in the clinic for his 18-month-old health supervision visit. The parent is concerned that he has stopped responding to social cues. You know the following is false regarding autism: a) Intervention for developmental delays is not available for children younger than 3 years old b) The MCHAT is a tool used to screen for autism c) Autism screening should be administered at the 18-month health supervision visit d) Autism screening should be administered at the 24-month health supervision visit

a) Intervention for developmental delays is not available for children younger than 3 years old This is FALSE. Children with delays, regardless of the cause, make better developmental progress if they receive appropriate developmental therapies than if they do not. Many infants and toddlers younger than 3 years with delays are eligible to receive a range of therapies and other services, often provided in the home, at no cost to families. Children aged 3 years and older with delays are eligible for developmental services through the local school system. Rationale for not choosing other answers: All are TRUE. b) c) d) In addition to general developmental screening, autism-specic screens (such as the Modified Checklist for Autism in Toddlers [MCHAT]) should be administered at the 18- and 24-month health supervision visits.

A 45-year-old healthy male comes in to your clinic complaining of severe, sharp headaches, worse behind his right eye, occurring nightly for the past 2 weeks. The pain comes on suddenly in the late evening or in the middle of the night, often awakening him from his sleep. The headaches last approximately 45 minutes to 1 hour and then they remit. The headaches are often accompanied by a runny nose and severe restlessness. He rates the pain as a 10/10, and states he has had no relief with ibuprofen or acetaminophen. He reports similar symptoms two months ago but the headaches went away after a few weeks. The nurse practitioner knows that the best treatment for this condition in the outpatient setting includes: a) PO Verapamil daily plus intranasal sumatriptan PRN at onset of headache. b) PO Propranolol BID plus PO Indomethacin PRN at onset of headache. c) Electrical stimulation of the right occipital nerve with implantable device. d) PO Lithium Carbonate daily plus PO Percocet at onset of headache.

a) PO Verapamil daily plus intranasal sumatriptan PRN at onset of headache. Rationales: a) *Verapamil is listed as first-line treatment for cluster headache prophylaxis. Intranasal sumatriptan is listed as first-line treatment for abortive therapy during an acute attack. PO medications are generally ineffective during an acute attack. b) Propranolol is first-line treatment for *prophylaxis in migraine headache*, not cluster headache. Cluster headaches are generally unresponsive to indomethacin. c) Electrical stimulation has not been proven to be effective in the treatment of cluster headache. d) Lithium Carbonate is listed as a first-line treatment option for prophylaxis of cluster headache. However, opioids are not first-line abortive therapy for cluster headache.

Which statement is FALSE regarding smoking cessation: a) Persons who quit smoking after the age of 65 do not see an increase in their life expectancy b) Smokers whose providers advise them to quit are more likely to quit smoking c) Weight gain occurs in most patients after smoking cessation d) Nicotine replacement therapy doubles the chance of successful quitting

a) Persons who quit smoking after the age of 65 do not see an increase in their life expectancy On average, women smokers who quit smoking by age 35 add about 3 years to their life expectancy, and men add more than 2 years to theirs. Smoking cessation can increase life expectancy even for those who stop after the age of 65. Rationale for not choosing other answers: b) Although tobacco use constitutes the most serious common medical problem, it is undertreated. Persons whose clinicians advise them to quit are 1.6 times as likely to attempt quitting. Over 70% of smokers see a physician each year, but only 20% of them receive any medical quitting advice or assistance. c) Weight gain occurs in most patients (80%) following smoking cessation. Average weight gain is 2 kg, but for some (10-15%), major weight gain—over 13 kg—may occur. Planning for the possibility of weight gain, and means of mitigating it, may help with maintenance of cessation. d) Nicotine replacement therapy doubles the chance of successful quitting. The nicotine patch, gum, and lozenges are available over the counter and nicotine nasal spray and inhalers by prescription.

4 year old female complains of leg pain at night which resolves by morning. This has lasted for the past 4 months. The NP should tell the patient's mother that: a) These are growing pains that last from 1 to 2 years. This is a common complaint in this age group. b) this complaint is typical of osgood schlatter disease. it will resolve spontaneously in 2 to 4 weeks. c) this pain is typical with juvenile arthritis. blood work wil confirm the dx. d) this pain is typical of children who need more attention spend more time with her at bedtime.

a) These are growing pains that last from 1 to 2 years. This is a common complaint in this age group. NOTES: Osgood-Schlatter is an overuse injury common in young athletes. Usually affects 1 knee but can be bilateral.

1) What would be the best assessment tool in evaluating substance abuse in teenagers? a) general psychosocial assessment b) CAGE questionnaire c) urine drug screen d) ask parents about substance abuse

a) general psychosocial assessment* the best way to screen for substance abuse among teenagers is a general psychosocial assessment. He discourages routine urine drug screening. CAGE questionnaire is useful to prompt discussion of a patient's thoughts on their substance use.

WHich of the following deficiencies is associated with ophthalmoplegia, hyporeflexia, ataxia and sensory loss? a) thiamin, b12 b) vitamin c, b-complex vitamins c) Riboflavin d) Vitamin e

a) thiamin, b12 Neuro deficits are often associated with the buildup of homocystine due to failure of b-12 dependent conversion enzymes.

A mother brings her 6-month-old baby for a health supervision visit. She is concerned her baby may have growth deficiency because she is only gaining less than 20 grams/day. Your response is based on the acceptable weight gain by age of: a. 3-6 months 15-20 g/d b. 3-6 months 20-30 g/d c. 3-6 months 6-11 g/d d. 3-6 months 3-7 g/d

a. 3-6 months 15-20 g/d Acceptable weight gain varies by age. The diagnosis of growth deficiency is reasonable if a child younger than 6 months has not grown for 2 consecutive months or a child older than 6 months has not grown for 3 consecutive months

According to the recommendations for preventive pediatric care from Bright Futures and the American Academy of Pediatrics (AAP), the following interventions should be part of the 24-months health supervision visit: a. BMI, head circumference, length/height and weight, autism screening, developmental surveillance. b. Weight for length, head circumference, hearing screening, developmental screening. c. BMI, head circumference, autism screening, developmental screening, lead screening. d. Length/height and weight, BMI, developmental screening, oral health.

a. BMI, head circumference, length/height and weight, autism screening, developmental surveillance. Rationale: These health supervision guidelines constitute a consensus by the AAP and the Bright Futures program to improve continuity of care and avoid gaps of care

2. A mother presents with her 2-year-old son to a rural urgent care center after what she thinks may have been her son having a seizure. She explains to the nurse that shortly after her son awoke from his nap he became unresponsive and began shaking violently for 30 seconds. The boy is now alert and interactive with a temperature of 40.1°C. Upon taking the medical history, the boy's mother states he had recently been complaining of left ear pain, and examination of the left ear demonstrates a bulging tympanic membrane. How would you best explain her son's condition to the mother? a. Explain that the patient likely had a febrile seizure related to a left ear infection and that these episodes are usually harmless and do not require any further testing. Reinforce the use of antipyretics while treating him for his ear infection. b. Tell the mother that the patient likely has epilepsy and will require further evaluation by a pediatric neurologist. c. State that the patient may have had a febrile seizure but will require an electroencephalogram and lumbar puncture study to be sure that no long-term damage was caused. d. Inform the mother that the patient will need to begin taking carbamazepine (20mg/kg) daily and will likely remain on this medication until adolescence.

a. Explain that the patient likely had a febrile seizure related to a left ear infection and that these episodes are usually harmless and do not require any further testing. Reinforce the use of antipyretics while treating him for his ear infection. Approximately 2%-3% of children will experience a febrile seizure between the ages of 3 months and 6 years of age with fevers greater that 38.8°C. These episodes are typically harmless, occur early in a febrile illness and last less than 5 minutes. Diagnostic studies are of little value if a non-CNS source of fever can be readily identified and the patient's mental status is reassuring. A diagnosis of epilepsy requires two unprovoked episodes at least 24 hours apart. Antiepileptic drugs are not typically indicated for simple febrile seizure prophylaxis, if prophylaxis is indicated phenobarbital or valproic acid are the treatments of choice.

The hormone responsible for producing a positive pregnancy test is: a. Human chorionic gonadotropin b. Estradiol c. Human growth hormone d. Progesterone

a. Human chorionic gonadotropin

63-year-old Hispanic female being seen at the urgent care clinic with complaints of right upper gastric pain, nausea with positive Murphy's sign. HIDA scan indicates thickening of gallbladder wall and was admitted to hospital. What post-inpatient hospital symptom is most alarming? a. Hydrops and jaundice b. Requested continued narcotic treatment outpatient c. Constipation since discharge d. Continued temps of 100.6

a. Hydrops and jaundice Complications of acute cholecystitis: "Hydrops of the gallbladder results when acute cholecystitis subsides but cystic duct obstruction persists. A stone in the neck of the gallbladder may compress the common hepatic duct and cause jaundice"

A patient presents to your clinic complaining of red plaques with silver scaling of the knees and elbows that get mildly itchy. He also tells you he has noticed "dents" in his fingernails recently. What would be the FNPs best diagnosis? a. Psoriasis b. Seborrheic Dermatitis c. Tinea Corporis d. Atopic Dermatitis

a. Psoriasis Criteria for psoriasis includes silvery scales on a bright red plaque. The areas involved are usually extensor surfaces of the body. Nail changes associated with psoriasis include pitting and onycholysis. +Seborrheic dermatitis usually involves the scalp and Centro cranial face. + Tinea Corporis is a fungal infection that appears as a ring shape scaly border with central clearing. + + Atopic dermatitis usually involves the flexor surfaces, and the patient usually has a history or family history of asthma and allergic rhinitis.

An obese 32-year-old male presents with complaint of a rash. Physical exam shows silvery scales on red well demarcated plaques located on the extensor surfaces of bilateral knees and elbows. Upon further examination, onycholysis is noted to the nails. What is the diagnosis? a. Psoriasis b. Atopic Dermatitis c. Eczema d. Seborrheic Dermatitis

a. Psoriasis * "The combination of red plaques with silvery scales on elbows and knees, with scaliness in the scalp or nail findings, is diagnostic. Psoriasis lesions are well demarcated and affect extensor surfaces-in contrast to atopic dermatitis, with poorly demarcated plaques in flexural distribution. In body folds, scraping and culture for Candida and examination of scalp and nails with distinguish "inverse psoriasis" from intertrigo and candidiasis."

A 14-year-old male comes to the clinic with complaint of enlarged breasts and is embarrassed to take his shirt off in the locker room at school. After a thorough history and assessment, the NP notes he has a healthy BMI, no family history of enlarged breasts, is currently in Tanner Stage 4 of development, and has enlargement of subareolar breast tissue bilaterally. What should the NP tell the patient? a. Reassure him this can be normal during this stage and should regress in 1-3 years b. This usually persists throughout adult life c. Referral to an endocrinologist is necessary at this time d. Hormone levels should be drawn at this time

a. Reassure him this can be normal during this stage and should regress in 1-3 years * Gynecomastia, or benign breast enlargement, typically occurs in young adolescent males 6 months after the onset of secondary sex characteristics (Hay, Levin, Deterding, & Abzug, 2016, p. 124), including pubic hair and lengthening and widening of the penis. A sexual maturity rating of 4 indicates this patient has an increase in width of the penis as well as full pubic hair in limited areas (Hay et al., 2016, p. 110). Gynecomastia can peak when SMR has reached stages 3 and 4 (Hay et al., 2016, p. 124). At this time, the patient should be reassured this can be a normal finding during this stage of development and it should subside in the next 1-3 years (Hay et al., 2016, p. 124). It is uncommon for gynecomastia to continue after the age of 17 (Hay et al., 2016, p. 124). At this time, lab work and referral are unnecessary due to an otherwise normal exam. The question did not indicate presence of hardened breast tissue, one-sided breast enlargement, skin changes, irregular testicles, or other pathological etiologies, which could warrant further workup and possible referral

2. A 79-year-old male claims feelings of anhedonia for several months. Which item is the most important in the practitioners first assessment of the patient? a. Suicidal ideations b. Withdrawal from normal activities c. Vague somatic complaints d. Insomnia

a. Suicidal ideations Suicide is the complication that is most serious. Patients who are male and over 50 y/o are of greatest risk of being successful with suicide. All the other signs and symptoms are concerning to a nurse practitioner, but not as serious as thought of suicide

2. A 12-year-old female presents to the office with the following symptoms: macular rash, photosensitivity, fatigue, weight loss, painless ulcers to nasal mucosa, and joint pain in her fingers. Which of the following is the most likely diagnosis for this patient? a. Systemic Lupus Erythematosus b. Juvenile Idiopathic Arthritis c. Vasculitis d. Dermatomyositis

a. Systemic Lupus Erythematosus* Systemic Lupus Erythematosus (SLE) is a multisystem inflammatory disease, which produces autoantibodies. SLE is a disease of the joints, skin, blood, kidneys, and CNS. Signs and symptoms will depend on the systems involved. Pediatric onset is more common in girls 9-15 years old (Soep, 2016, p. 885). Four out of 11 criteria must be met to make a diagnosis of SLE. The criteria include macular rash, discoid rash, mucous membrane ulcers, arthritis, photosensitivity, serositis, neurologic and renal, and blood count abnormalities, positive antinuclear antibodies (ANA), and autoantibodies. Other signs and symptoms seen with SLE are weight loss, fever, fatigue, myositis, vasculitis, and depression (Soep, 2016, p. 885-886). SLE treatment is aimed at treating the affected organ systems. The treatment of choice, which has shown to lower mortality rates, is prednisone. The goal is to use the minimum amount required to control the disease and minimize toxicity. Patients should be monitored clinically and labs evaluated to tailor the dose of prednisone. Antimalarials are used to treat skin manifestations, fatigue, and arthritis. NSAIDS are used to manage arthritic pain (Soep, 2016, p. 886). SLE prognosis has improved dramatically since the 1950's. Today's survival rate is 90%. Prognosis is affected by renal system involvement and complications from infection. SLE can flare at any time (Soep, 2016, p. 887).

A 65 year old patient presents with a history of recurrent right upper quadrant pain associated with intermittent nausea and vomiting. Laboratory tests reveal isolated elevations of serum alkaline phosphatase and normal amylase levels. Physical examination results are within normal limits. The tentative diagnosis is: a. biliary obstruction b. peptic ulcer c. chronic pancreatitis d. hepatic dysfunction

a. biliary obstruction AANP

A 78 year old patient presents with complaints of left-sided "rib pain" during the past few days. The patient also complains of headache, a feverish feeling, and general malaise. Physical examination reveals an area of papular eruptions with a few vesicles on the left side of the chest. The most likely cause of the patient's symptoms is: a. herpes zoster b. eczema c. intertrigo d. actinic keratosis

a. herpes zoster

John, 56 yr old male, presents to your oce with "heartburn" 4-5 times per week He reports a burning sensation occurring about 1 hr after eating and does get relief with Tums occasionally. He mentions the heartburn is worse when he lays down at night requiring him to sleep in his recliner. He reports sometimes it is painful to swallow. Upon review of his records, the provider notes he has gained 5 lbs since his last visit. Which of the following "alarm" symptoms warrant further investigation of John's heartburn? a. odynophagia b. relief with Tums c. weight gain d. increased reflux when laying down

a. odynophagia There are 4 alarm signs of GERD that warrant further workup due to the risk of esophagitis or other complications. The alarm signs are iron deficiency anemia, dysphagia, weight loss, and odynophagia (painful swallowing). If a pt presents with these symptoms, referral to gastrointestinal doctor may be necessary for further testing such as an upper endoscopy or pH testing to rule out other disease processes.

Research has shown a link of hypothyroidism due to cardiac medication. Which medication is known to cause significant hypothyroidism in approximately 15-20% of patients, due to its high iodine content? a) Lisinopril b) Amiodarone c) Amitriptyline d) Sumatriptan

b) Amiodarone

Providers should screen, counsel, and educate adolescents regarding sexually transmitted diseases a) Within six months of becoming sexually active b) At annual visits and every acute care visit c) When they're being seen for STI treatment d) If their parents approve of the conversation

b) At annual visits and every acute care visit * Sexual health education and counseling should take place at every acute care visit because not all adolescents receive regular preventive care (Hay, Levin, Deterding, and Abzug, 2014). This education should be open and non-judgmental, allowing freedom of expression

The provider instructs the patient that he (the patient) should "Just follow what is ordered" without taking into account what the patient's wishes are. This is an example of...? a) cultural conflict b) cultural imposition c) cultural incongruence

b) Cultural imposition. Cultural imposition is when you expect the patient to do what you say to do, in this case, your professional knowledge trumps what they know and ignores it. Medicine has been patriarchal for a long time. Cultural conflict is what is happening across the world. When we can't get along because we do not understand each other. We do not respect each other's views...we fight with words and bodies. Dr. Leininger said this would happen back in 1952.The point is...if we do not provide culturally congruent care (professional and patient ways), we get cultural conflict. If we push our values on the patient (imposition), then we get cultural conflict. Cultural incongruence is when we provide care that is not fitting for the patient's culture. Forum

A 33-year-old woman presents to the clinic in her first trimester of pregnancy. She asks the NP how much weight she should be gaining during this pregnancy. The NP knows that the following statement is true: a) For women with a normal BMI, weight gain should be 40 lbs or greater during pregnancy. b) For women with a normal BMI, weight gain should be between 25-35 lbs during pregnancy. c) For overweight women, weight gain should be 25-35 lbs during pregnancy. d) For obese women, weight gain should be under 9 lbs during pregnancy.

b) For women with a normal BMI, weight gain should be between 25-35 lbs during pregnancy. Nutrition in pregnancy is extremely important to discuss with women in the early stages of pregnancy. Excessive weight gain can lead to pregnancy related issues, increased birth weight, and increased postpartum weight. Insufficient weight gain can lead to low birth weight infants and adverse outcomes. Discussing good nutrition and appropriate weight gain is important for positive outcomes of both mother and baby. + Normal Weight Pregnancy - 25-35 lbs + Overweight Women Pregnancy- 15-25 lbs + Obese Women Pregnancy (BMI 30 or greater)- limit to 11-20 lbs

The nurse practitioner is performing a well-child exam of a 6-month-old girl. Her mother is planning to raise her on a vegan diet and introduce solid foods soon. The nurse practitioner advises the mother that she should give the infant: a) A daily multi-mineral supplement rich in copper, calcium, and magnesium. b) Fortified cereals or formulas rich in iron, zinc, and B12, or a daily supplement with these nutrients. c) Pureed fruits and vegetables rich in riboflavin, pyridoxine, and folic acid. d) 6 ounces of soymilk daily to ensure she gets enough protein.

b) Fortified cereals or formulas rich in iron, zinc, and B12, or a daily supplement with these nutrients Rationales: a) Copper, magnesium, and calcium can all be found in vegetables, whole grains, and nuts, and a daily supplement is not necessary. b) Children consuming vegan diets are at risk for iron and zinc deficiency as these nutrients are found in limited supply in fruits and vegetables. B12 is only found in animal-based foods or fortified cereals/formulas, it is not found in plant-based foods. Therefore, the infant should consume foods fortified with iron, zinc, and B12, or a daily supplement with theses nutrients. c) Riboflavin, pyridoxine, and folic acid can all be found in vegetables, whole grains, and nuts, and a daily supplement is not necessary. d) There is no recommendation for soymilk supplementation in infants consuming a vegan diet.

2. A 46 year old female, presents to the clinic with complaints of muscle and abdominal cramps, carpopedal spasms, tingling in her lips and hands, and tetany. During your assessment, you confirm a positive Chvostek sign. You know that a positive Chvostek sign is indicative of which abnormal findings? a) Hypothyroidism, Hypercalcemia b) Hypocalcemia, Hypoparathyroidism c) Hypercalcemia, Hyponatremia d) Hypocalcemia, Hyperparathyroidism

b) Hypocalcemia, Hypoparathyroidism Chvostek sign, which is facial contraction upon tapping the facial nerve in front of the ear, is positive in both hypocalcemia and hypoparathyroidism.

All of the following drugs below provide only transient benefits for hyperkalemia in children EXCEPT, _____________ which reduces potassium levels through proper renal excretion: a) Sodium Bicarbonate b) Sodium Polystyrene Sulfonate (Kayexalate) c) Insulin with glucose d) Albuterol

b) Sodium Polystyrene Sulfonate (Kayexalate) Sodium Bicarbonate, Albuterol, and Insulin with Glucose all provide transient shifts of potassium from the extracellular fluid to the intracellular fluid compartments. Sodium Polystyrene Sulfonate (Kayexalate) uses ion exchange resins to exchange potassium for sodium through renal excretion. This will decrease the total body potassium

A 18-year-old college student presented to the clinic complaining of right foot pain after a long week of practice and cross-country competition. He was diagnosed with plantar fasciitis and advised with the following: NSAIDs, local massage, wear good fitting shoes, arch support. As a Nurse Practitioner, what additional advice would you give the patient? a) Apply a shin pad to the affected foot when running b) Stretching of the gastrocnemius-soleus-Achilles complex c) No change on usual running activities d) Apply cast on the affected foot

b) Stretching of the gastrocnemius-soleus-Achilles complex * Plantar fasciitis is a typical problem among adolescents or athletes who accrue long hours of running, not wearing good fitting shoes, and have tight Achilles tendons. Treatments include: Local massage, stretching of the gastrocnemius-soleus-Achilles complex, NSAIDS, arch supports, and local steroid injection. Shin pad and casting on the affected foot are not indicated. Runners may need to decrease the amount of running until pain is eliminated.

In which of the following scenarios would breastfeeding be contraindicated? a) Baby born at 38 wees to a mother who is positive for hep c virus antibody b) baby born at 36 weeks who tests positive for galactose 1-phosphate uridyltransferase deficiency on the newborn screen c) baby born at 35 weeks with hyperbilirubinemia who is admitted to hospital for phototherapy d) Baby born at 41 weeks to mother who is seropositive for CMV

b) baby born at 36 weeks who tests positive for galactose 1-phosphate uridyltransferase deficiency on the newborn screen

2. The NP performs a wellness exam on a healthy 4-year-old and includes a visual acuity screening exam. What is the minimal acceptable acuity for this patient? a. 20/30 b. 20/40 c. 20/50 d. 20/60

b. 20/40* An acceptable acuity for patients 3-5 years of age is 20/40 at minimum (Hay, Levin, Deterding, & Abzug, 2016, p. 233). Pediatric patients six and older should hold a minimal acuity of 20/30 (Hay et al., 2016, p. 233). Patients who do not meet this minimal acuity or have differing scores between both eyes of two lines or greater, should be referred to an ophthalmologist

Screening for Tuberculosis with the Mantoux test can be done in children as young as: a. 6 months of age b. 3 months of age c. 12 months of age d. 18 months of age.

b. 3 months of age The Mantoux test is the only recommended screening test and can be done as early as 3 months of age

Which of the following physical exam findings should alert the healthcare provider of potential physical abuse? a. A 2-year-old with bruising on both of his elbows b. An 8-month-old with unexplained bruising on his abdomen c. A 12-month-old with limited subcutaneous fat in the extremities d. A 2-year-old with labial erythema

b. An 8-month-old with unexplained bruising on his abdomen Unexplained bruising in an infant who is not yet mobile is considered an exam finding consistent with physical abuse. Bruising over bony prominences such as the elbows or shins in toddlers and older children is usually the result of accidental injuries. Limited subcutaneous fat in the buttocks, cheeks, and extremities is found in children with failure to thrive and suggests physical neglect. Most sexually abused children display no physical findings. Labial erythema is a nonspecific finding that does not correlate with physical, sexual, or emotional abuse.

A 15-year-old female patient with a history of Turner syndrome and hypertension presents to your clinic for a new patient appointment. As a nurse practitioner, you recognize the patient is at risk of which of the following? a. Atrial fibrillation b. Aortic dissection c. CHF d. Endocarditis

b. Aortic dissection Patient's with Turner syndrome are at risk for aortic dissection particularly in the presence of cardiovascular abnormalities such as hypertension, bicuspid aortic valve, and coarctation of the aorta which put them at a greater risk of having an aortic dissection

The most common medications that are prescribed for headaches in the pediatric population include all of the following except a. Topiramate b. Divalproex c. Amitriptyline d. Propranolol

b. Divalproex The most commonly prescribed medications for pediatric headaches include topiramate, propranolol, amitriptyline, and cyproheptadine. Topiramates should be started slowly in and at low doses in order to avoid cognitive side effects. Amitriptyline should be given at nighttime due to the common side effect of sedation. Divalproex is not proven to be effective and can cause side effects such as weight gain, tremor, hair loss, and can be teratogenic so should be used with caution in female adolescents.

A father brings his 5-year-old daughter to the office inquiring about her tetanus vaccinations. According to the patient's records, she received her 4th DTaP vaccine at the age of 4. The NP should: a. Order the 5th and final dose of DTaP b. Inform the father the child does not need an additional tetanus vaccine. c. Order a single dose of Tdap d. Give the child a booster dose of Td

b. Inform the father the child does not need an additional tetanus vaccine. Children should receive the 5th dose of DTaP at 4-6 years of age. However, a fifth dose of DTaP is not needed if the fourth dose was given after the child's 4th birthday. Children between the ages of 7-10 who are not fully immunized against pertussis should receive a single dose of Tdap. Adolescents and adults should receive a single dose of Tdap followed by a booster dose of Td every 10 years.

Randy, a 29-year-old male, is a known asthmatic that presents to your clinic. He reports waking up with coughing once a week and shortness of breath when jogging on occasion. He uses his rescue inhaler 2 times per week. You know that his asthma can be diagnosed as the following based on his symptoms? a. Intermittent b. Mild, persistent c. Moderate, persistent d. Severe, persistent

b. Mild, persistent According to the classification of asthma severity, if a pt is having problems on more than 2 days/week but not daily, waking up 3-4 times per month at night, having to use rescue inhaler greater than 2 days/week but not daily, and never >1 time/day, the pt is considered to have mild, persistent asthma needing Step 2 therapy.

A seven year old who presents with two lesions on the extremities is diagnosed with impetigo. Which topical treatment is most appropriate? a. Clotrimazole (Lotrimin) cream b. Mupirocin (Bactroban) cream or lotion c. Hexachlorophene emulsion (pHisoHex) d. Acyclovir (Zovirax) ointment

b. Mupirocin (Bactroban) cream or lotion

A 12-year-old male and his mother present to the clinic with the complaint of "being tired all the time." His mother is also asking about the dark, velvety areas in his neck creases and what causes it. The NP notices in his chart that the patient's BMI >95th percentile and begins to consider Nonalcoholic Fatty Liver Disease (NAFLD) in her differential diagnoses. What other findings would help confirm this diagnosis? a. Elevated bilirubin and alkaline phosphatase, tenderness in RUQ with sudden inspiratory arrest on palpation, postprandial pain episodes after ingesting fatty meal b. Normal-Mild elevation of ALT/AST, hyperglycemia, and infiltrates noted on liver ultrasound c. Moderate elevation of ALT/AST, recurrent fevers, abdominal distention, positive ANA d. Extreme elevation of ALT/AST, nausea/vomiting, and tender hepatomegaly on palpation

b. Normal-Mild elevation of ALT/AST, hyperglycemia, and infiltrates noted on liver ultrasound According to the 23rd edition of Current Diagnosis & Treatment of Pediatrics, NAFLD may have normal to mildly elevated AST/ALT (1.5x the normal), mildly elevated alkaline phosphatase but normal bilirubin, hyperglycemia and hyperlipidemia, fatty infiltrations of liver on US/MRI/CT along with obesity, c/o fatigue, hepatomegaly (nontender), sleep apnea, and insulin resistance (acanthosis nigricans/buffalo hump). The other options are associated with other GI disorders: option A- cholelithiasis, option C- Autoimmune hepatitis, option D- acute liver failure.

An 18-month female presents with fever, irritability, and no other symptoms or complaints. The mother admits to strong smelling urine when changing her diaper this morning. The FNP understands that unexplained fever should prompt urinalysis and culture. Collecting a urine sample for culture includes: a. Cleaning the patient's perineum, sitting the patient on the toilet until she voids into a toilet specimen container, and then pouring the urine into a specimen cup. b. Obtaining a urine sample via urethral catheterization. A bagged urine sample is only useful if the results are negative. c. Treating the patient with an antibiotic prior to collecting urine sample. d. Is not needed, and the patient should be treated with a broad-spectrum antibiotic.

b. Obtaining a urine sample via urethral catheterization. A bagged urine sample is only useful if the results are negative. A properly collected urine specimen for culture is the gold standard for diagnosing urinary tract infections. Although less invasive, the bagged urine sample may not be reliable in diagnosing urinary tract infection because of the contamination risk during the collection process. The FNP would still need to obtain a urine sample via urethral catherization. If using urine collected from a bag, appropriate antibiotic selection can be delayed while waiting for the culture and sensitivity results.

2. A 50-year-old female presents to your clinic with a headache. She has a history of migraines but hasn't had one in years. She explains that this headache does not feel like the migraines she used to get in the past. She states the pain is worse when she is lying down and wakes her up at night. She is also complaining of night sweats. She has tried NSAIDs, Acetaminophen, Aspirin and Sumatriptan (Imitrex) with no relief. Denies alcohol, caffeine, drug, or tobacco use. What should the FNP do next? a. Prescribe Ergotamine and caffeine tablets b. Refer for brain imaging c. Treat with antidepressant (amitriptyline or venlafaxine) d. Have the patient keep a headache diary and follow up in 2 weeks

b. Refer for brain imaging * The key feature in prompting brain imaging is a new or worsening headache in middle or later life. Patient stated that the headache was not her typical migraine thus ruling out considerations for migraine treatment or prophylaxis like ergotamine or antidepressants. Obtaining a headache diary could give more data or information but as a provider we must see red flags and not delay care. She identified that the headache comes at night, while lying down. Intracranial mass lesions often cause worsening pain when lying down, awaken patients at night, or peak in the morning after overnight recumbency.

2 A 32-year-old female that is 26 weeks pregnant comes to the clinic for her 50g oral glucose tolerance test. Her glucose at 1 hour is 160mg/dL. The provider's next step is: a. None, this is a normal value. b. Schedule the patient for a 3-hour glucose tolerance test c. Start the patient on insulin d. Repeat the 50g oral glucose tolerance test

b. Schedule the patient for a 3-hour glucose tolerance test Diagnosis of gestational diabetes requires testing of blood glucose levels after a 100g oral glucose tolerance test. The patient should fast overnight, for at least 8 hours and not exceed 14 hours. Venous glucose is measured at fasting, 1 hour, 2 hour and 3 hours. The diagnosis is made when two or more of the values are met or exceed: fasting, 95mg/dL; 1 hour, 180mg/dL; 2 hours, 155mg/dL; 3 hours, 140mg/dL.

You are seeing a 33 year old woman for her annual well woman exam. You perform a PAP test to screen for cervical cancer. This as an example of: a. Primary prevention b. Secondary prevention c. Tertiary prevention d. None of the above

b. Secondary prevention* Primary prevention is preventing a disease before it happens by modifying and identifying risk factors of that disease. Secondary prevention is early identification of the disease. Tertiary prevention is treating complications of an existing disease or limiting its progression.

2.) A 2 month old has just been diagnosed with clubfoot. Your teaching for the parent will include all of the following except. a. Clubfoot may be treated non-surgically. b. Serial casting is performed weekly for 1-2 weeks c. Infants that undergo the Ponseti technique can ambulate independently only 2 months later than infants with no deformity. d. The French method includes a combination of taping and splinting.

b. Serial casting is performed weekly for 1-2 weeks.* B is incorrect because serial casting is performed weekly for 6-8 weeks. All other answers are correct.

1. A 10-year-old male was recently diagnosed with an uninhibited neurogenic bladder after sustaining an upper motor neuron injury resulting in a hyper-reflexive voiding pattern. Which of the following is the treatment of choice the NP can teach the patient and family for an uninhibited neurogenic bladder? a. Intermittent catherization b. Timed voiding c. Credé maneuver d. Valsalva maneuver

b. Timed voiding* An upper motor neuron injury can cause failure of the detrusor to contract, which results in a hyper-reflexive voiding pattern. This type of neurogenic bladder is an uninhibited neurogenic bladder Neurogenic bladder treatment is dependent on the type of bladder dysfunction. Timed voiding is the simplest technique and is the most effective method for an uninhibited neurogenic bladder. Timed voiding is when children are given verbal cues to void before bladder capacity is reached. (Wilson & Clayton, 2016, p. 879). Patient's with a flaccid bladder should be taught the Credé and Valsalva maneuvers to assist in draining their autonomous bladder. These maneuvers should never be taught in patients with a reflex neurogenic bladder due to the risk of increasing intravesicular pressure, which can cause vesicoureteral reflux (Wilson & Clayton, 2016, 879). Intermittent catherization, medications, or vesicostomy may be needed in children with a high-pressure bladder causing reflux (Wilson & Clayton, 2016, p. 879-880).

An 18 year old gravid female at 37 weeks gestation presents to the clinic with complaints of headache, ankle swelling, and "seeing spots". She has been followed for elevated BP that is diet controlled and occasional proteinuria. Her BP today is 180/100 with platelet count 98, 3+ proteinuria, AST 32 and ALT 38. The NP educates the patient on the optimal course of action. a. Complete a 24 hour urine for protein/creatinine and return to the office in 1 week for results and a BP check. b. We need to deliver the fetus because this is the only cure for severe pre-eclampsia. c. Follow up with cardiology this week for hypertension. d. Begin treatment with Labetalol 100 mg PO BID.

b. We need to deliver the fetus because this is the only cure for severe pre-eclampsia. Severe pre-eclampsia is defined as BP >160/110, thrombocytopenia, elevated liver enzymes, headache, and visual changes. Awaiting results of 24 hour urine, effectiveness of Labetalol and cardiology consult may hinder optimal outcome of mother and fetus. The only curative treatment is delivery of fetus.

The nurse practitioner is aware that which of the following is the most accurate way to assess weight when determining BMI and assessing degree of malnutrition? a. obtaining the weight within the first hour of waking in the morning b. gown-only weight after urination c. the average of three weight measurements in any given week d. obtaining the weight at least one hour after last meal

b. gown-only weight after urination The most accurate way to assess weight is a gown-only weight after urination. Patients may hide weights in their pockets, drink excessive fluid, or wear bulky clothing to alter weight measurements.

A 25 year-old White male with a history of asthma presents to the clinic for shortness of breath (SOB). He reports using his rescue inhaler at least once daily for the past two weeks, needing to rest several times when exercising, and experiencing nighttime awakenings twice this week due to SOB. If his only current medication is albuterol PRN, what medication(s) would be most appropriate to add for the patient's asthma control? a. medium-dose ICS + LABA b. low-dose ICS only c. LABA only d. low-dose ICS + LABA

b. low-dose ICS only The patient's HPI suggests that his asthma is not well controlled (SABA >2 days/week, some limitation of normal activity, and 1-3 nighttime awakenings/week) and that the next "step" of asthma management is needed to control symptoms. The preferred medication for Step 2 of asthma management would be a low-dose ICS only. Alternatives would include cromolyn, LTRA, nedocromil, or theophylline.

Conductive hearing loss involves the: a. inner ear b. middle ear c. 5th cranial nerve d. 8th cranial nerve

b. middle ear

Mrs. Jones is a 66-year-old Caucasian female who is being discharged from the hospital after suffering from a myocardial infarction. Which statin should she be discharged on and by how much should it decrease her LDL cholesterol? a. simvastatin 40 mg; 50% b. rosuvastatin 20 mg qhs; 50% c. pravastatin 40 mg qhs; 30% d. atorvastatin 40 mg qhs; 30%

b. rosuvastatin 20 mg qhs; 50% This patient who has known ASCVD needs a high-intensity statin. Only two statins provide high-intensity lowering ability, rosuvastatin and atorvastatin, however they will lower the LDL by approximately 50%, not 30%.

Most common type of skin cancer

basal cell carcinoma

The most common type of skin cancer is.....?

basal cell carcinoma Leik

What tests are recommended for the patient on Isoniazid as tx for TB?

baseline LFT level and follow-up testing Leik

Gold standard exam for temporal arteritis

biopsy of the temporal artery. refer pt to opthalmologist for mgt LEIK

Which of the following statements about medications in a 45-year old patient with diabetes is correct? a) Start her on a statin b) Ace inhibitors are no longer venoprotective after albuminuria is present c) Daily ASA tx should be recommended for the seondary prevention of CVD

c) Daily ASA tx should be recommended for the seondary prevention of CVD Daily ASA therapy for the secondary prevention of CVD is recommended for pts with DM. the use of ASA for primary preventionm especialy in low risk pts, is not routinely reciommened. ACEI are especially beneficial after albuminuria is present. Even if BP is normal, an ACEI may be used for renoprotection. The initiation of statin therpay is dependent on CV risk or the presence of CVD. ACEI may not be required for all pts with DM.

The FNP knows which vaccination is ALWAYS recommended during pregnancy: a) Varicella vaccine at any point in the first trimester. b) Hepatitis A Vaccine from 20-36 weeks of pregnancy. c) Tdap booster between 27-36 weeks of pregnancy. d) MMR vaccine between 12-15 weeks.

c) Tdap booster between 27-36 weeks of pregnancy. Live virus vaccines such as measles, mumps, rubella, yellow fever, smallpox and varicella are contraindicated during pregnancy. Hepatitis A vaccination may be given during pregnancy as an inactivated form, however, is only given to women who are at risk for this during pregnancy. Tdap booster is ALWAYS recommended for all women between 27-36 weeks during each pregnancy she has. The Tdap booster works by giving the mother antibodies to pass to the fetus in order to maximize antibodies against pertussis once baby is born.

2) Which of the following are characteristics of bulimia nervosa? a) below average BMI b) sick or unwell appearance c) normal or above average BMI d) dieting or restricting food intake

c) normal or above average BMI* Sigel (2016) describes the typical BN patient as normal or above average weight with no physical appearance of illness. This information is found on page 167. Dieting and restricting food intake are more likely to be characteristics of anorexia nervosa.

2. The diagnosis of preeclampsia can be made when which of the following are observed: a. Blood pressure of 150/90 at first pre-natal visit at 6 weeks gestation. b. A random protein level of 30 grams on urinalysis. c. A blood pressure of 140/90 or greater and proteinuria of 300 mg in 24 hours occurring after 20 weeks gestation or before 6 weeks postpartum. d. Complaint of a severe headache and generalized swelling.

c. A blood pressure of 140/90 or greater and proteinuria of 300 mg in 24 hours occurring after 20 weeks gestation or before 6 weeks postpartum. Any elevated BP prior to 20 weeks gestation is most likely due to an undiagnosed primary hypertension. Random protein level is not diagnostic. Preeclampsia is diagnosed when a patient has an elevated BP of 140/90 or higher and 300 mg or more of protein in urine within 24 hours. Edema is no longer required for preeclampsia diagnosis. Headache and edema may be present in preeclampsia but are not diagnostic. They can be present without preeclampsia and would require further evaluation.

Which of the following children should receive the Rotavirus vaccine at their well child appointment today? a. A 7 month old child who has not started the Rotavirus series b. A 4 month old child whose last Rotavirus vaccine was 3 weeks ago c. An 8 week old coming in for first series of vaccinations d. A 4 month old child who has not started the Rotavirus series

c. An 8 week old coming in for first series of vaccinations The Rotavirus vaccine is an oral vaccine that comes in 2 forms- either RV5 (a 3 series vaccination) or RV1 (a 2 series vaccination). This vaccine should not be started after age 15 weeks, and must be completed by 8 months of age. A child must be a minimum of 6 weeks of age for first dose and a maximum of 14 weeks 6 days for first dose. There must be a minimum of 4 weeks between the doses.

Johnny, a 9-year-old male who is diagnosed with moderate depression presents to the clinic today accompanied by his Mother for a follow-up appointment. He has been participating in cognitive behavioral therapy (CBT) for the past 6 months, but he reports persistent feelings of sadness and his Mother is concerned with his lack of interest in playing soccer, which he usually enjoys. Based on his age and condition, you decide to add on treatment with which of the following FDA approved medications? a. Escitalopram b. Sertraline c. Fluoxetine d. Citalopram

c. Fluoxetine : When depressive symptoms are mild to moderate in nature, CBT alone often improves the symptoms. If moderate symptoms persist or in cases of severe symptoms, antidepressants may be indicated. Healthcare providers should discuss treatment options including medications which are FDA approved for the child's age and condition. *Fluoxetine is the only medication FDA approved for children younger than 12 years old.* Fluoxetine can be utilized for children beginning at age 8. Citalopram and Sertraline are not FDA approved for depression. Sertraline is indicated for OCD, autism and aggression, or mania and schizophrenia. Escitalopram, although indicated for depression, is only FDA approved for children 12 and older.

When introducing solid foods to a normal 6-month-old infant, the NP should advise the parents to: a. Introduce multiple, new foods per day to determine what the infant likes the best b. avoid giving meats, as they lack nutritional value in infants c. Introduce a single-ingredient food, one at a time, every 3-4 days d. not introduce solid foods until 12 months of age to prevent atopic disease

c. Introduce a single-ingredient food, one at a time, every 3-4 days The AAP and WHO recommend introducing solid food in normal infants starting about 6 months of age. Single-ingredient food should be introduced one at a time, with 3-4 days in between trying new foods. This will allow the parents to detect if the child is having an allergic reaction to the food.

Initial management of slipped capital femoral epiphysis includes which of the following: a. Referring the patient to physical therapy b. Instructing the patient to apply ice to the hip 2-3 times daily c. Making the patient non-weight bearing with crutches, and referring to an orthopedic surgeon as soon as possible d. Ordering a lateral xray of the hip

c. Making the patient non-weight bearing with crutches, and referring to an orthopedic surgeon as soon as possible * Initially, the patient should be made non-weight bearing, placed on crutches and referred immediately to an orthopedic surgeon

A 28-year-old patient with a history of PCOS and obesity presents to your clinic for a lab review. It is noted that the patient's Hgb A1C is 5.9 % and her fasting glucose was 119. Based on this information, what medication would be beneficial to aiding in both the glucose abnormality and menstrual abnormality accompanying her PCOS? a. Finasteride b. Spironolactone c. Metformin d. Medroxyprogesterone acetate

c. Metformin Metformin is the drug of choice for patients with PCOS and insulin resistance. Metformin can be used to treat the glucose and menstrual abnormalities

Which statement by the NP demonstrates understanding of cervical cancer screening and prevention for the lesbian patient? a. HPV vaccine is unnecessary in this population. b. PAP screening is not needed for the lesbian patient. c. PAP screening for the lesbian population should match that of the heterosexual recommendations starting at age 21 years. * d. HPV screening with PAP should begin at age 21 years.

c. PAP screening for the lesbian population should match that of the heterosexual recommendations starting at age 21 years. PAPs start at age 21 years for all women regardless of sexual orientation. HPV vaccination is recommended for the lesbian population. Regarding screening, PAP and HPV screening recommendations are the same for lesbian as is for heterosexual with PAP starting at age 21 and HPV age 30.

2. A 16 year old female presents into the clinic for an annual exam. Pt reports being on olanzapine for the last three years for the treatment of anorexia nervosa. FNP knows to periodically screen which of the following labs in this patient: a. Hgb and Hct b. PT/INR c. Plasma glucose d. BUN

c. Plasma glucose Second generation antipsychotics can cause new-onset diabetes. Blood glucose levels should be checked before starting the therapy, 12 months after and then annually

Mr. Adams is a 34-year-old male whom you are seeing today after complaints of chest pain with his last episode being yesterday. Upon questioning him, you note he rarely comes to the practitioner's office and he is hypertensive, with a blood pressure of 164/88. He is a poor historian, but he reports he take any medication. He also reports he doesn't really know if his family has any medical history. He is currently chest pain free. He admits that he uses cocaine about once a week, the last time being 2 days ago. Given Mr. Adams presentation and symptoms, his most likely diagnosis is: a. Unstable Angina b. GERD c. Prinzmental angina d. Pericarditis

c. Prinzmental angina Cocaine can induce coronary vasoconstriction and thereby produce chest pain. Otherwise the coronaries of these patients are generally normal. This type pf pain from intermittent acutely induced vasoconstriction is referred to as prinzmetal or variant angina.

You are examining a 9-year-old female in your clinic for a well child check. During the breast examination, you note the child has separate mounds for the areola and nipple. You document in your note that the child is in which stage of SMR? a. SMR 2 b. SMR 3 c. SMR 4 d. SMR 5

c. SMR 4 For female breast development, a typical sequence is followed. SMR 1 indicates pt is prepubertal. SMR 2 results in small, raised buds. During SMR 3, the breast and areolar tissue enlarge and elevate as a union. The nipple and areola form 2 separate mounds during SMR 4. The areola is uniform with the breast in SMR 5, which is considered adult maturity.

A 10 year male presents with a sore throat and fever that began approximately 24 hours ago. While in the clinic, he tests positive for streptococcal pharyngitis and you prescribe a 10 day course of amoxicillin. When asked when he may return to soccer practice, which of the following would be included in this patient's education? a. You may return to soccer practice this evening if you feel up to it b. You may return to soccer practice after you complete the 10 day course of amoxicillin c. You may return to soccer practice 24 hours after starting the amoxicillin and if you no longer have a fever d. You may return to soccer practice once your throat stops hurting

c. You may return to soccer practice 24 hours after starting the amoxicillin and if you no longer have a fever * With the diagnosis of streptococcal pharyngitis, sports participation may resume once antibiotic treatment has been provided for 24 hours and fever is absent

A 16 year-old female patient presents with an edematous ankle. Your examination reveals a pinpoint wound at the lateral aspect of the ankle and X-rays show a distal fibular fracture. In addition to managing the fracture, which intervention is most appropriate? a. administer tetanus prophylaxis and submit wound scraping for culture b. irrigate the wound and apply topical antibiotic c. administer tetanus prophylaxis and prescribe oral antibiotics d. apply topical antibiotic and cover the wound with a sterile dressing

c. administer tetanus prophylaxis and prescribe oral antibiotics

A 24 year-old patient complains of intermittent heartburn, which has become worse since he started his new job. The discomfort is worse after eating and at night, and is relieved by antacids. Your most likely diagnosis is: a. diffuse esophageal spasm b. infectious esophagitis c. gastroesophageal reflux disease d. carcinoma of the esophagus

c. gastroesophageal reflux disease AANP

A Diabetic female patient comes in complaining of feeling cold and fatigued often. You order a CMP which shows a high serum Phosphate and low serum Calcium. Which of the following is UNLIKELY to cause the abnormal values: a. Hypoparathyroidism b. Vit D3 Deficiency c. Hypertension d. Chronic Kidney Disease

c. hypertension Most common cause of hypocalcemia is advanced CKD in which decreased production of active D3 and hyperphosphatemia play a role. True hypocalcemia (decreased ionized calcium) implies insuicient action of PTH or active vitamin D.

2. A 62 year old female with a known history of diverticulosis presents with a 2 day history of temperature max 100.1 orally, left lower quadrant tenderness 4/10, and constipation. Outpatient treatment of this patient for suspected diverticulitis should include all the following except: a. Clear liquid diet b. Metronidazole (Flagyl) 500mg PO TID for 7 days c. Pantoprazole (Protonix) 40mg PO Daily for 4 weeks d. Ciprofloxacin (Cipro) 500mg PO BID for 7 days

c. pantoprazole (protonix) Most patients with uncomplicated diverticular disease can be managed as an out patient on 7-10 days of antibiotics if warranted and a clear liquid diet until symptoms improve. Antibiotic treatment includes Augmentin BID or Flagyl TID, plus Cipro or Bactrim BID. A patient who cannot tolerated oral liquids, or has increasing pain or fever, requires hospitalization.

A 70 year-old female with urinary stress incontinence should be instructed to: a. perform abdominal strengthening exercises twice a day b. perform pelvic floor muscle (Kegel) exercises 100 times per day c. perform pelvic floor (Kegel) exercises 35-40 times per day d. void frequently

c. perform pelvic floor (Kegel) exercises 35-40 times per day

A 37 year-old male diagnosed with hypertension has been treated with a low sodium diet and hydrochlorothiazide (HCTZ) 50 mg qd for the past two months. He denies Family history of cardiovascular disease. At today's followup visit his BP=150/90 and T=I00 F. Physical examination reveals no bruits, clear chest, no atrial gallop, edema and tenderness of the left ankle, and an intact neurological system. Which laboratory values will provide the most useful follow-up information? a. serum sodium and potassium b. total serum cholesterol and serum glucose c. serum uric acid and complete blood count d. blood urea nitrogen and creatinine

c. serum uric acid and complete blood count AANP

In order to determine the presence of postural hypotension, blood pressure should be taken in which of the following positions? a. sitting to standing b. supine to sitting c. supine to standing d. standing to supine

c. supine to standing

What is the preferred tx for cutaneous anthrax?

ciprofloxacin 500 mg orally BID for 60 days or for 8 weeks. If the patient is allergic to ciprofloxacin, use doxycycline 100 mg BID Leik

The NP suspects Otitis Externa when presentation and assessment reveals which of the following? a. Ear pain and pressure b. Decreased hearing c. Fever d. Pain when touching ear auricle with history of swimming

d With external otitis there is often recent exposure to water with excessive moisture to the canal allowing growth of bacteria. The physical exam will reveal erythema of the ear canal with purulent exudate and pain upon manipulation of the auricle. Question pool

Which of the following is NOT a risk factor assessed in the FRAX fracture risk assessment tool? a) Age b) Parental history of hip fracture c) Smoking status d) Daily weight-bearing exercise

d) Daily weight-bearing exercise The World Health Organization utilizes the fracture risk assessment tool (FRAX) to determine a woman's 10-year risk of hip or osteoporotic fracture (Papadakis and McPhee, 2018). The assessment includes risk factors that include age, history of fracture, parental history of hip fracture, corticosteroid use, alcohol intake, smoking, BMI, and secondary osteoporosis. The assessment does not assess exercise habits

An infant presents to clinic with reports of vomiting, diarrhea and poor oral intake. The following would be considered a severe sign of dehydration: a) dry mucous membranes b) capillary refill less than 2-3 seconds c) decreased urinary output d) lack of tears

d) lack of tears Severe clinical manifestation of dehydration are 11-15% decrease in body weight, markedly decreased skin turgor and color, mottled or gray mucous membranes, tachycardia, capillary refill greater than 4 seconds, low blood pressure, anuria, and absent tears

2. Bone mineral density screening is recommended by the USPSTF at age: a. 50 b. 55 c. 60 d. 65

d. 65* The USPSTF recommends screening women aged 65 and older and only screening younger women whose fracture risk is equal to or greater than that of a 65-year-old white woman who has no additional risk factors

A 67 year old woman presents to the primary care office following right-sided cataract removal complaints of vision problems. She states that she woke this morning with decreased vision in her right eye which has been getting progressively worse throughout the day. She denies pain and states that it feels as though a curtain is closing over the affected eye. On exam the nurse practitioner notes a gray cloud appearance across the inferior aspect of the right retina. Based on this information which of the following conditions is suspected? a. Acute angle-closure glaucoma requiring reduction of intraocular pressure b. Corneal ulcer requiring ophthalmic ciprofloxacin 0.3% c. Normal healing process following cataract removal requiring no intervention d. Acute retinal detachment requiring emergent evaluation by an ophthalmologist

d. Acute retinal detachment requiring emergent evaluation by an ophthalmologist Retinal detachment is an ophthalmic emergency which typically presents as a rapid, painless loss of vision often describes as a curtain spreading across the visual field. Cataract extraction is a typical predisposing cause of retinal detachment in individuals 50 years and older. Patients with intact central vision require emergent referral to an ophthalmologist as this indicates that the macula has not yet detached and may have a better prognosis with earlier intervention. Patients with acute angle-closure glaucoma often complain of severe pain, have injected conjunctiva and cloudy appearance of the involved cornea. Corneal ulcers typically present with pain, photophobia and tearing without retinal involvement.

To reduce the threat of hemolytic disease of the newborn, an Rh-negative mother carrying an Rh-positive fetus is recommended to have which of the following? a. Rh immune globulin administered within 72 hours after birth. b. Rh immune globulin administered at the 28th week gestation. c. Rh immune globulin within 72 hours of a spontaneous abortion or ectopic pregnancy. d. All of the above.

d. All of the above When an Rh negative mother is exposed to fetal red blood cells of an Rh-positive baby during delivery or any other antepartum event associated with bleeding, the mother produces Rh antibodies that attack the Rh-positive fetal red blood cells in subsequent children. Administration of Rh immune globulin (RhoGAM) eliminates the Rh-positive red blood cells before the mother can produce Rh antibodies. Immune globulin clearance by the mother is slow providing continued protection for 12 weeks, which is why it is administered around the 28th week gestation. Protection is up until a full-term delivery. A second dose is administered within 72 hours of delivery or abortion for added protection.

1. A 14 year old male is presenting into the clinic in a hyperalert state with visible tremors and increased restlessness. Assessment reveals increased blood pressure, discoordination and dilated pupils, FNP suspects an overdose of which of the following substances: a. THC b. MDMA (ecstasy) c. Opiates d. Amphetamines

d. Amphetamines THC signs of intoxication include euphoria, mood changes, psychosis, and impaired thinking. MDMA (ecstasy) s/s of intoxication include increased self-esteem, illusions, difficulty with concentration, headache, flushing, increased energy. Opiate intoxication includes the s/s of sedation, constricted pupils, decreased blood pressure, and impaired thinking. Amphetamine intoxication includes hyperalert state, ataxia, tremors, hyperactivity, increased blood pressure, fever/flushing

A 6-year-old female presents to your clinic with nausea and diarrhea for 2 days. Mother reports that she has had 3 loose stools per day but no vomiting or fever. The patient weights 22 kg. The FNP notes that the patient's skin turgor is normal, but the mucous membranes are dry. After determining the patient has an uncomplicated case of gastroenteritis and mild dehydration, the FNP educates the mother to: a. Offer small, frequent sips of water and/or Pedialyte to equal 50 mL/kg over the next 4 hours and 1500 mL + 40 mL per day. b. Avoid giving soda, juice, and teas. c. Administer an antibiotic to shorten the length of illness. d. Both A and B.

d. Both A and B. Offering small sips of about 15 to 20 mL about every 15 mins helps lessen risk of vomiting liquid. Daily fluid replacement is based on patient's weight. For patient weighing 22kg, 1500mL + 20ml/kg for each kg greater than 20kg. For this patient, 1500 ml + 20ml/kg X 2kg = 1540ml/day. Rationale for not choosing the other answers: The sugar content in soda and juice may worsen diarrhea as sugar has an osmotic effect in the bowels. Caffeine in tea stimulates diuresis and may provoke tachycardia. Most cases of gastroenteritis are caused by a virus, in which antibiotics would not be effective in treatment.

A 40 year old male presents today with complaints of facial paresis and left ear pain that occurred abruptly yesterday morning. Upon exam, you notice vesicles in the left external ear canal and left-sided facial drooping when asked to smile. What is the likely treatment for this patient? a. Amoxicillin b. Prednisone c. Acyclovir d. Both B & C

d. Both B & C This patient is showing symptoms of Bell Palsy. Treatment includes corticosteroids to increase the chance of a full recovery in 9-12 months. Acyclovir is indicated when vesicles are present in the external ear canal.

Selective Serotonin Reuptake Inhibitors have the following sexual side effects except: a. Retrograde ejaculation b. Dysorgasmia c. Erectile dysfunction d. Decrease libido

d. Decrease libido Retrograde ejaculation, dysorgasmia, and erectile dysfunction are common side effects of SSRIs. Sildenafil or tadalafil can be taken 1 hour prior to having sex to lessen the side effects. Bupropion can be used to enhance arousal. Cipropheptadine can be used to help anorgasmia but is very sedating (Raj, Williams, & DeBattista, 2018).

A 9-year-old comes in with a 2-day history of nausea and vomiting and reports severe pain in epigastric area that radiates to his back. On exam, he is tender on palpation to the midabdominal area and bowel sounds are diminished. The NP explains that she would like to order to diagnostic testing but suspects acute pancreatitis. All of the following diagnoses are considered in the differential diagnoses EXCEPT: a. Peptic Ulcer Disease b. Acute Gastroenteritis c. Cholecystitis d. Diverticulitis

d. Diverticulitis : According to the 23rd edition of Current Diagnosis and Treatment of Pediatrics, Ch. 22- Liver and Pancreas, page 692, all above diagnoses (peptic ulcer disease, acute gastroenteritis, cholecystitis) are included in the differential diagnoses for acute pancreatitis. The only diagnosis not mentioned is diverticulitis. As noted in the 2018 edition of Current Diagnosis and Treatment, while nausea/vomiting is common in patients with diverticulitis, pain is described as mild to moderate and usually in left lower quadrant, low-grade fever, left lower quadrant tenderness, and a palpable mass.

Which screening is NOT done at the 6-12 week prenatal visit? a. Complete blood count b. RPR c. Rubella antibody titer d. Group B streptococcal colonization

d. Group B streptococcal colonization Group B strep is collected at 35-37 week's gestation. If the patient is positive, intrapartum antibiotics are required. Without treatment, transmission to the neonate may occur resulting in sepsis, pneumonia, and/or meningitis. Patients receive Penicillin G 5 million units IV as a loading dose then 2.5-3 million units IV every 4 hours until delivery. During the 6-12 week visit a UA, random blood glucose, CBC, serologic tests for syphilis (RPR), rubella antibody titer, varicella immunity, blood group, Rh type, antibody screening for anti-Rh, hep B surface antigen and HIV are all performed. Women at increased risk should receive chlamydia and gonorrhea testing. CDC recommends: All pregnant women under 25 years of age; Pregnant women, aged 25 and older if at increased risk; Retest during the 3rd trimester for women under 25 years of age or at risk; Pregnant women with chlamydial infection should have a test-of-cure 3-4 weeks after treatment and be retested within 3 months.

1) An infant has been diagnosed with sickle cell anemia. Which of the following is not a treatment for sickle cell in childhood? a. Daily Penicillin Prophylaxis b. Red blood cell transfusions c. Adequate Hydration d. Hemodialysis

d. Hemodialysis *** hemodialysis is not a common treatment for sickle cell crisis. Some sickle cell patients require dialysis due to renal failure from lack of oxygen but that does not usually occur in childhood.

A child presents to the clinic with abdominal pain, bloody stools, with a maculopapular and purpuric rash to the face, bilateral lower legs, and buttocks. The nurse practitioner reviews the renal ultrasound results showing a renal lesion. These findings are typical for what illness? a. Ig-A Nephropathy (Berger's Disease) b. Lupus Glomerulonephritis c. Hemolytic-Uremic Syndrome d. Henoch-Schoenlein Purpura

d. Henoch-Schoenlein Purpura Henoch-Schoenlein purpura is typically seen with type of described rash, bloody stools, and diarrhea. The renal lesion is common to Ig-A, except Ig-A usually doesn't require treatment like Henoch-Schoenlein purpura does with steroid treatment. lupus glomerulonephritis and Henoch-Schoenlein purpura both may involve joint pain, but lupus findings include a positive antinuclear antibody test. For hemolytic-uremic syndrome, a rash is not common however seizure, low platelet counts, and hypertension are common findings.

All of the following are causes of epilepsy except a. Systemic lupus erythematosus b. Alzheimer's disease c. Toxoplasmosis d. Hypercortisolism

d. Hypercortisolism Seizures can be caused by a plethora of etiologies and are grouped into three main categories: genetic, structural/metabolic, and unknown. Epilepsy may be caused by inborn disorders such as the autoimmune diagnosis of SLE. Degenerative disorders can also be responsible for seizure activity, such as Alzheimer's disease. Infectious diseases, such as central nervous system toxoplasmosis, can also cause seizure activity.

1. A 78-year-old male and his daughter present to clinic with concerns about memory loss. The daughter states that for the last 6 months her father has had trouble recalling people's names, been confused, and experiences difficulty remembering what he had to eat for breakfast. The FNP examines the patient and suggests performing a neuropsychological assessment to screen for cognitive impairment/dementia. Which of these tools are not indicated to evaluate cognitive impairment? a. Mini-Cog b. Montreal Cognitive Assessment (MoCA) c. Mini Mental State Exam (MMSE) d. PHQ-9

d. PHQ-9 * Quantifying cognitive impairment can be performed by an array of screening and diagnostic tests. Most commonly used tests for cognitive impairment include the MMSE, MoCA, and Mini-Cog. The PHQ-9 is a screening test for depression. Although this is often a component of dementia/Alzheimer's it is not a test to screen for cognitive impairment.

A four-year-old pediatric patient presents to the clinic for a hospital follow up from an admission for seizures and hemolytic uremic syndrome. The nurse practitioner knows which of the following is least likely to cause this illness? a. Shigella b. Escherichia Coli c. Pneumococcal Infections d. Pseudomonas

d. Pseudomonas Hemolytic-uremic syndrome is acute kidney failure potentially caused by Shigella, Escherichia Coli, Pneumococcal infections, certain medications, and a small percentage of genetic influence. This syndrome can lead to seizures, decreased platelets, and hypertension.

A 4-year-old female presents with a complaint of abdominal pain, fever, and urinary frequency for one day. The mother reports foul-smelling urine. The NP suspects a UTI. What is the gold standard for diagnosis? a. Urinalysis b. Positive Bagged Urine Specimen c. Renal ultrasound d. Urine Culture

d. Urine Culture "The gold standard for diagnosis remains the culture of a properly collected urine specimen."

A child presents to the clinic with warts on her fingers. The parent is asking you, the FNP, how her child got warts. Your best response is: a. Warts are spread by wet skin to skin contact. b. Warts are caused by the enterovirus c. Warts are a bacterial infection that is very contagious. d. Warts are caused by infection with the human papillomavirus (HPV)

d. Warts are caused by infection with the human papillomavirus (HPV) Warts are caused by the HPV virus. There are over 200 types of the virus, and each type is associated with different wart types. Molluscum are spread by wet skin to skin contact. Hand foot and mouth disease is caused by enterovirus. Although warts are contagious, they are viral not bacterial.

The nurse practitioner is assessing a 16-year-old female patient in clinic. The nurse practitioner becomes concerned when the patient states she sometimes feels she cannot stop eating, has been taking supplements for weight loss, and exercises daily. The patient's 24-hour diet recall only includes sodas, chips, candy, and other sweet treats. The patient is average in body weight and vital signs are within normal limits. The nurse practitioner would most likely make a diagnosis of which of the following? a. avoidant/restrictive food intake disorder b. anorexia nervosa c. binge-eating disorder d. bulimia nervosa

d. bulimia nervosa * Diagnostic criteria for bulimia nervosa includes binge eating excessive amounts of food during a normal mealtime or a sense of lack of control over eating during the episode (eg, feeling one cannot stop eating). Carbohydrates or junk food are typically included during a binge. Body weight is usually average or slightly above average in bulimic patients and there are no physical abnormalities present. Binge-eating disorder (BED) is mostly found in overweight or obese individuals and does not include the recurrent use of inappropriate compensatory behaviors. Significant weight loss is seen with anorexia nervosa and avoidant/restrictive food intake disorder.

After reviewing the LDL levels of a 53 year-old Hispanic female, the nurse practitioner informs the patient that she would like to prescribe high-intensity statin therapy with atorvastatin 80mg—a HMG-CoA reductase inhibitor. Which of the following laboratory results would prevent the nurse practitioner from starting the new medication immediately? a. elevated pancreatic enzymes b. elevated T3 and T4 c. decreased BUN and creatinine d. elevated liver function tests

d. elevated liver function tests* Manufacturers of statins recommend monitoring liver enzymes before initiating therapy. Liver failure is a serious, but rather uncommon, side effect of statins.

A patient who presents with fever, cervical lymphadenopathy, tonsillar exudate, and fine maculopapular rash most likely has: a. streptococcal pharyngitis b. secondary syphilis c. pharyngeal candidiasis d. mononucleosis

d. mononucleosis AANP sample question

Dermatological condition with vesicular rashes on an erythematous base?

herpes simplex, genital herpes Leik

Foods and supplements containing stimulants such as caffeine and ephedra are best avoided by patients with WHAT types of conditions?

hypertension, arrhythmias, high risk for MI, thyroid disease, etc. Leik

What are the Quinolones with gram-positive coverage?

levofloxacin (Levaquin); moxifloxacin (Avelox); gatifloxacin (Tequin) LEIK

Abnormal findings in Diabetic retinopathy

neovascularization, hard exudates, cotton wool spots, and microaneurysms Leik

symptoms of prostate cancer a. firm like the very tip of the nose b. rubbery c. tenderness d. induration

normal gland (firm like the very tip of the nose), rubbery (chronic prostatitis), and tenderness (acute prostatitis). induration is the only choice that suggests prostate cancer Forum

hydrocele in a newborn

observe for first 12 months because most resolves spontaneously

What is the first-line (or preferred) drug for treating strep throat?

penicillin VK PO for 10 days Leik

Best pain relief for pregnant women?

pick acetaminophen (Tylenol) instead of NSAIDs such as ibuprofen (Advil) or naproxen (Aleve, Anaprox). Leik

pinguecula vs pterygium

pinguecula = yellowish mass of conjunctiva that does not cross cornea. no trt necessary pterygium = vascularized wedge of conjunctiva that overrides cornea. excision may be required

Xanthelasma

raised yellowish plaque on eyelid caused by lipid disorder

Screening test for temporal arteritis

sed rate

What abx can you give a PCN-allergic patient who has a gram-positive infection?

the possible alternatives are macrolides or clindamycin. LEIK

How do thiazides slow down bone loss?

thiazide reduces sodium reabsorption and promotes calcium reabsorption. Thiazide also acts on bone cells directly by decreasing osteoclast differentiation.

Which type of cancer has the highest mortality in males and females: 1. Colon cancer 2. Lung cancer 3. Liver cancer 4. Skin cancer

​Answer is option 2. Lung cancer has the highest mortality.

Adverse effects of loop diuretics

■ Electrolytes (hypokalemia, hyponatremia/low sodium, low levels of chlorine). ■ Hypovolemia and hypotension (dizziness, lightheadedness). ■ Pancreatitis, jaundice, rash. ■ Ototoxicity (worsens aminoglycoside ototoxicity effect if combined). Leik

Risk factors for atrial fibrillation

■ HTN, CAD, caffeine, nicotine, hyperthyroidism, alcohol intake ("holiday heart"), heart failure, LVH, and others. ■ Paroxysmal AF (intermittent or self-terminating): Episodes terminate with 7 days or less (usually in less than 24 hours). It is usually asymptomatic. Leik

Adverse effects of thiazides

■ Hyperglycemia (careful with diabetics). ■ Elevates triglycerides and LDL (careful if preexisting hypertriglyceremia). ■ Elevates uric acid (can precipitate a gout attack). ■ Hypokalemia (muscle weakness, arrythymia).

Good alternative antibiotics for PCN-allergic patients with gram-positive bacterial infections are:

■ Macrolides such as azithromycin × 5 days (Z-Pack) or clarithromycin (Biaxin) PO BID. ■ Clindamycin (Cleocin) is also an alternative, but it is associated with slightly higher risk for C. difficile colitis. Leik

Treatment of cellulitis

■ Non-MRSA nonpurulent cellulitis: Dicloxacillin PO (orally) QID × 10 days (preferred due to high rate of beta-lactam resistance). Cephalexin QID or clindamycin TID × 10 days. ■ Penicillin allergic: Erythromycins (macrolides), second-generation cephalosporins, clindamycin. ■ Suspect MRSA: Bactrim DS one tablet BID × 10 days, Doxycycline PO BID × 10 days or clindamycin 3 to 4×/day for 10 days. ■ Td booster: If last dose was more than 5 years ago. ■ Recurrent cellulitis: Consider decolonozation. Muciprocin BID on nares × 5 to 10 days. ■ Elevate affected limb. NOTE: Follow up within 48 hours

Drugs with a narrow therapeutic index and which require regular monitoring

■ Warfarin sodium (Coumadin): monitor INR. ■ Digoxin (Lanoxin): monitor digoxin level, EKG, electrolytes (potassium, magnesium, calcium). ■ Theophylline: monitor blood levels. ■ Carbamezapine (Tegretol) and phenytoin (Dilantin): monitor blood levels. ■ Levothyroxine: monitor TSH. ■ Lithium: monitor blood levels, TSH (risk of hypothyroidism).


Conjuntos de estudio relacionados

NCLEX 4000 Questions with answers Health Assessment

View Set